Sunteți pe pagina 1din 560

AHMAD IBRAHIM SECONDARY SCHOOL

GCE O-LEVEL PRELIMINARY EXAMINATION 2018

PHYSICS
PAPER 1

6091/01
Sec 4 Express Date: 16 August 2018
Duration: 1 hour

ame ass

READ THESE INSTRUCTIONS FIRST:

ri e i so pe ci .
o o use s ap es paper c ips i i ers ue or correc io ui .
ri e our ame c ass um er a re is ra io um er o e op ica a swer s ee
pro i e .

ere are forty ues io s i is paper. swer all ues io s.


or eac ues io ere are our possi e a swers A B C a D.
oose e one ou co si er correc a recor our c oice i soft pencil o e op ica
a swer s ee pro i e .

Read the instructions on the answer sheet very carefully.

Eac correc a swer wi score o e mar . mar wi o e e uc e or a wro a swer.


rou wor i s ou e o ei is ues io paper.
e use o a appro e scie i ic ca cu a or is expec e w ere appropria e.

is ues io paper co sis s o 20 pri e pa es

3
2

1 Pe u um A ma es comp e e osci a io s i s. Pe u um B ma es comp e e


osci a io s i s. o pe u ums were isp ace a sma a e e ore eir
osci a io s.

ic o e o owi s a eme s mus e rue

A Pe u um B as a s or er perio a pe u um A.
B e s ri o pe u um B is o er a a o pe u um A.
C e mass o e o o pe u um B is sma er a a o pe u um A.
D ea e o swi o re ease or pe u um is sma er a a o pe u um A.

2 pair o er ier ca ipers is use o measure e ic ess o a coi .

ia ram s ows e rea i wi e aws c ose . ia ram s ows e rea i w e


e aws are c ose arou e coi .

cm cm

ia ram ia ram
a is e ero error a e ac ua ic ess o e coi

Zero error / cm Corrected reading / cm

A . .

B . .

C . .

D . .

4
3

3 s u e uses a microme er screw au e o measure e iame er o a a eari .


ia ram s ows e ero error o e au e a ia ram s ows e measureme
o e iame er e ore i is correc e .

ia ram ia ram

a is e rue iame er o e a eari

A . mm B . mm C . mm D . mm

4 sp ere ru s a o a smoo rai rom P o Q as s ow .

ic o e o owi rap s es represe s e aria io o e is a ce d ra e e


e sp ere wi ime t

5
4

5 ic c e acce era es rom a spee o . ms o msi . s.

a is i s a era e spee uri e our e

A . ms B . ms C . ms D . ms

6 e ia ram s ows e rap o isp aceme s a ai s ime t or a o mo i i a


s rai i e.

ic o e o owi s ows e rap o spee v a ai s ime t or is o

7 wo me a oc s are s ac e o e o op o e o er as s ow i e ia ram e ow.


e are roppe i acuum a i o e er ree u er ear s ra i a io a ie .
a is e e orce ac i o e me a oc uri e a

A B C D

6
5

8 e ia ram s ows a as rappe i e e arm o a ma ome er co ai i mercur .

e a mosp eric pressure is mm w a is e pressure o e rappe as

A mm B mm C mm D mm

9 ea er o i ui is p ace u er a e ar. e pressure o e air a o e e i ui is


re uce a some i ui e apora es. is causes e i ui o ecome co er.

oes e empera ure o e i ui a

A e air mo ecu es ow awa e i ui mo ecu es.

B e air mo ecu es coo ow e i ui .


C e i er e er mo ecu es ea e e i ui .
D ere are ewer mo ecu es o i ui i e ea er.

7
6

10 rau ic i is use o suppor a car o wei .

car

orce F

Pis o A Pis o B
. m . m

ui

Pis o A as a cross sec io a area o . m w i e pis o B as a cross sec io a


area o . m . a is e orce F ee e o suppor e wei o e car

A B C D

11 e ou e o a s ri e is sea e a air is rappe i e s ri e. e empera ure o


e air is ep co s a .

oes e pressure o e air i crease w e e pis o is pus e rom X o Y

A e air mo ecu es are mo i as er.

B e co isio e wee e air mo ecu es i creases.


C e orces e wee air mo ecu es i creases.

D e ra e o co isio o e air mo ecu es wi e wa i creases.

8
7

12 mercur i ass ermome er is s ow . a is e empera ure w e e mercur


rea is a T

A o B o C o D o

13 a er o mass . a is poure i o a i su a e me a co ai er o mass


. a . e i a empera ure o wa er is . . e speci ic ea capaci
o wa er is K w a is e speci ic ea capaci o e me a i K

A B C D

14 ic s a eme a ou a ixe mass o as is correc

A s pressure i creases a co s a empera ure e o ume ecreases.

B s pressure i creases a co s a empera ure e o ume i creases.

C s empera ure i creases a co s a pressure e o ume ecreases.

D s empera ure i creases a co s a o ume e pressure ecreases.

15 a escri es e o ume a s ape o a as or i ui a co s a empera ure

A e o ume o a as is ixe u i s s ape is o ixe .

B e o ume o a as is o ixe a i s s ape is o ixe .


C e o ume o a i ui is ixe a i s s ape is ixe .
D e o ume o a i ui is o ixe u i s s ape is ixe .

9
8

16 ir i a c ose co ai er co ai s smo e i umi a e ri i . e iewe


rou microscope ri spec s o i are see mo i a ra om.
ic s a eme is correc

A e ra om mo io o e spec s is as er i a acuum.
B e spec s mo e as er w e e air is a a i er empera ure.

C e spec s see are mo ecu es o air i rapi ra om mo io .


D e e i is ur e o e spec s s ow ow a s op mo i .

17 e ia ram s ows our i e ica ca s wi eir ou si e sur aces pai e ei er u


ac or po is e si er. Eac ca co ai s e same o ume o wa er i i ia a .
er i e mi u es i a coo room w ic ca co ai s e o es wa er

A B C D

10
9

18 e ia ram s ows a acuum as a a e ar e iew o a sec io rou e as


wa .

e mai reaso or e si eri is o re uce ea ra s er


A co uc io o .

B co uc io a co ec io .

C ra ia io o .

D ra ia io a co ec io .

19 coo ma es e pu i a e as a .

e pu i is p ace i a er o o e u i e op o ee w i e ur s row . is
e remo e rom e o e .
oes e ice cream s a co

A ir is a oo co uc or o ea a co uc s e ea awa rom e ice cream.


B ir is a poor co uc or o ea a s ows e ea rom reac i e ice cream.
C e me a is is a oo co uc or o ea a co uc s e ea awa rom e
ice cream.
D e me a is is a poor co uc or o ea a s ows e ea rom reac i e
ice cream.

11
10

20 ra o i is i ci e o o e si e o a rec a u ar ass oc . ea e o re rac io


is i e ass. e cri ica a e or i i ass is .
ic ia ram s ows e pa o is ra

21 e i ure e ow s ows e op iew o a emp room wi a er ica p a e mirror M a


e mi e. ac e s a s a poi P a oo s i o e mirror.

ic o ec cannot e see ac e i e mirror

12
11

22 e a o ec is p ace cm i ro o a co ex e s e ima e is orme cm


e i e e s.

ow wou e si e o e ima e a e ima e is a ce c a ew e e o ec is


mo e cm ur er awa rom e e s

Size of image Image distance

A ecreases ecreases

B ecreases i creases

C i creases ecreases

D i creases i creases

23 poi P is mar e o a rope e ore e rope is se o osci a e. e par icu ar


i s a ce s ow w a is e irec io o mo eme o e poi P

24 a es . s o e era e o e comp e e wa e e i a ripp e a . e wa e e


o eac wa e pro uce is . cm. a is e spee o e wa e

A . cm s B . cm s C . cm s D . cm s

13
12

25 e i ure s ows wa er ra e i rom ec io R o S i a ripp e a .

ic s a eme is correc

A e re ue c is i er i sec io S.

B e wa er is s a ower i sec io S.

C e wa er is eeper i sec io S.

D e wa es mo e s ower i sec io S.

26 a es P a Q are compo e s o e e ec roma e ic spec rum. P as a o er


wa e e a Q. ic o e o owi s a eme s is true a ou P a Q

A P is ra iowa e a Q is i rare ra ia io .

B P ca ra e as er a Qi acuum.

C Q as a ower re ue c a P.

D Q is u ra io e ra a P is ra .

14
13

27 po ice sou s i s sire w e ra e i o a emer e c . e sire pro uces wo


i ere sou s P a Q w ic are emi e a er a e .

e ia ram represe s e sou emi e e sire .

ic o e wo sou sPa Q is e so er a w ic as e ower pi c

Softer sound Sound of lower pitch


A P P

B P

C P

28 ompressio s a rare ac io s are se ou rom a ou spea er co e as i i ra es


ac war s a orwar s. e re ue c o i ra io is .

compressio is a poi P. ow muc ime e apses e ore e ex rare ac io arri es


a P

A . s B . s C s D s

15
14

29 e a i e c ar e ro is rou ear wo iso a e me a a s X a Y. X is e


ear e mome ari as s ow i e ia ram. e ro is e remo e ow wou
e a s e c ar e

X Y
A e ai e posi i e

B eu ra eu ra

C posi i e e ai e

D posi i e eu ra

30 resis or wi resis a ce R is ma e rom a e L o resis a ce wire wi a cross


sec io a area A. seco resis or wi resis a ce R is ma e rom wire o e same
ma eria wi a cross sec io a area o . A.

a e o wire is ee e or e seco resis or

A L B . L C L D . L

16
15

31 e ia ram s ows a po e ia i i er circui wi wo i e ica amps L 1 a L2.

a wi appe o e ri ess o e amps w e co ac K is mo e owar s X

lamp L 1 lamp L 2
A ri er ri er

B ri er immer

C immer ri er

D immer immer

32 e ia ram s ows e resis a ce empera ure rap o a ermis or i a circui .

a are e curre a o a ew e e empera ure o e ermis or is

Current /A Voltage / V

A . .

B . .

C . .

D . .

17
16

33 circui s X Y a Z s ow a e amps are i e ica a e use i e ica r ce s.

circui X circui Y circui Z

a is e esce i or er o resis a ce i eac circui

A X Ya Z
B X Za Y

C Y Za X

D Z Ya X

34 e ec rica ca e co ai s ree wires i e eu ra a ear . e ca e is correc


wire o a p u w ic co ai s a use. e ca e i su a io ecomes ama e a
e are me a wires are expose . i e possi e e e s ma occur.

x perso ouc es e ear wire.


x perso ouc es e eu ra wire.
x perso ouc es e i e wire.
x e i e wire ouc es e eu ra wire.
x e i e wire ouc es e ear wire.

ow ma o ese e e s wi cause e use i epu o ow

A B C D

18
17

35 P a Q represe wo para e s rai wires carr i curre s i o e pa e o e


paper. P a Q exer a orce o eac o er.

ic arrow s ows e orce o Q

36 e ia ram e ow s ows a simp e .c. mo or.

e e swi c is c ose w ic o e o owi s a eme s is are correc

curre wi ow rou e coi i e irec io WXYZ.


e coi wi ro a e i a c oc wise irec io a ou e ax e.
e sp i ri commu a or wi re erse e irec io o e curre e er .

A o B a o C a o D a

19
18

37 ma e is roppe er ica rou a copper ri .

ic o e o owi s a eme s is incorrect

A curre ows i e ri us e ore e ma e passes rou e ri .


B curre ows i e ri us a er e ma e passes rou e ri .
C e ma e s ows ow us e ore i passes rou e ri .

D e ma e acce era es us a er i passes rou e ri .

38 e ia ram s ows par o a a.c. e era or w e i s coi is i a ori o a posi io .

e rap e ow s ows e o a e ou pu p o e a ai s ime.

ic poi o e rap s ows e coi i a er ica posi io

20
19

39 i ea ra s ormer as a primar o a eo a a seco ar o a eo .

e seco ar coi is a ac e o a resis or o resis a ce .

a is e power issipa e i e resis or a e curre i e primar coi

Power / W Current / A
A .

B .

C .

D .

21
20

40 osci oscope is use o isp a e wa e orms o a er a i curre a.c. i pu .

ia ram s ows e osci oscope race pro uce e irs i pu o o a e . a


re ue c .

ia ram s ows e race pro uce e seco i pu . e co ro s o e


osci oscope are se a e same a ues.

ia ram ia ram

a is e o a ea re ue c o e seco i pu

Voltage / V Frequency / Hz
A .

B .

C .

D .

END OF PAPER
Setter: Mr Luqman

22
AHMAD IBRAHIM SECONDARY SCHOOL
GCE O-LEVEL PRELIMINARY EXAMINATION 2018

PHYSICS
PAPER 2

6091/02
Sec 4 Express Date: 14 August 2018
Duration: 1 h 45 min

ame ass

READ THESE INSTRUCTIONS FIRST

Do not open this booklet until you are told to do so.


ri e ow our ame c ass a re is er um er o is pa e a o a a i io a wri i
papers.
ri e i ar ue or ac pe .
ou ma use a pe ci or a ia rams rap s a es or rou wor i .
o o use s ap es paper c ips ue or correc io ui .

Section A
swer all ues io s.

Section B
swer a ues io s. ues io as a c oice o par s o a swer.

Information for candidates:


a i a es are remi e a all ua i a i e a swers s ou i c u e appropria e u i s.
e use o a appro e scie i ic ca cu a or is expec e w ere appropria e.
a i a es are a ise o s ow a eir wor i i a c ear a or er ma er as more
mar s are awar e or sou use o P sics a or correc a swers.

e um er o mar s is i e i rac e s a ee o eac ues io or par ues io .

FOR EXAMINER’S USE

Section A / 50

Section B / 30

TOTAL / 80

is ues io paper co sis s o 19 pri e pa es

23
Section A

swer all e ues io s i is sec io .

1 ru er a is roppe ree rom a ei o m. e a i s e rou a ime t a re ou s


er ica upwar s wi a i s maximum e oci . e maximum e oci o e a us e ore i i s
e rou or e irs ime is V.

(a) e axes e ow s e c e e oci ime rap o e a rom e poi o re ease o


e ime w e i as re ou e o i s ew maximum ei . ore air resis a ce

(b) si i orma io rom e rap e ermi e e e oci o e a us e ore i is e


rou or e irs ime.

e oci .

(c) i e isp aceme o e a a er i as re ou e o i s maximum ei .

isp aceme .

(d) ae ec a ei e oci o e a uri i s re ou .

c a ei e oci .

24
2 i . . s ows ree c i ers X Y a Z are suppor e ree ropes a passes rou ri R.

Fig. 2.1

i R is i e ui i rium u er e ac io o ree orces F x F y a F z.

raw a ec or ia ram o i Fx a a eT.

Fx .

a e ș ..

25
3 u i orm ro AB o e m wei s . is suspe e wo i e ica s ri s
a poi s X a Y as s ow i i . . . T 1 a T 2 are e e sio i e s ri s.

ri ri

T1 T2
. m

. m . m . m

A B
X Y
P

Fig. 3.1

wo wei s a P are u rom e ro a poi Aa . m rom X


respec i e .

(a) raw e wei o e ro i i . . a a e i W.


ica e c ear i s is a ce rom poi A.

(b) e ermi e T 2 e e sio o e s ri a Y.

T2 .

(c) e ce or o erwise e ermi e T 1 e e sio o e s ri a X.

T1 .

26
4 sma e p a e w ic ca carr six peop e is s ow i i . . .

Fig. 4.1

e mass o e u oa e e p a e is . is i i ia a res . e e e p a e is
a i o e wo e e i es ca exer a o a rus orce o a e ric io e wee
e w ee s a e rou is . o orces remai co s a a ese a ues uri a e
o .

(a) a cu a e e acce era io o e p a e as i s ar s o mo e.

acce era io .

(b) Exp ai w a appe s o is acce era io as e e p a e spee s up.

....

(c) e a era e acce era io uri a e o is . m s .

(i) a cu a e e ime a e e p a e wi a e o reac a a e o spee o m s.

ime .

(ii) e ermi e e mi imum e o e ru wa a is re uire or e e pa e o


a eo .

mi imum e .

27
(d) u es w e w ee s o e e p a e are o e i o e o o e e p a e a er
a eo .

5 i . . e ow s ows a o er ica ass u e wi o e e immerse i mercur a


e o er co ec e o a acuum pump a A. e u e i s i i o a e ar. i a
ope i a B a a air i e ass u e pumpe ou ia A e mercur rises o a
maximum ei o . cm a o e e is .

o acuum pump

ass u e
cm o

. cm

Y B

mercur
Fig. 5.1

(a) Exp ai w e mercur o ca rise o a maximum ei o . cm.

....

(b) e e si o mercur is m ca cu a e e pressure a Y i pasca s.

pressure .

28
(c) co ai er o air i i ia a a mosp eric pressure is co ec e oBa ea e o er a
ame as s ow i i . . .

o acuum pump

co ai er
. cm

mercur ame
Fig. 5.2

si i e ic eor o ma er exp ai w e er e ei o mercur co um rises a s


or remai s e same.

....

....

6 o co uc s a experime o e ermi e e speci ic a e ea o aporisa io o wa er.

e p aces a immersio ea er u immerse i wa er i a ope s ee co ai er. e o a e


across e immersio ea er is a e curre w ic passes rou e ea i
e eme is . . o measures e mass o wa er a er s ea oi i is ac ie e a a ai
a er a o er mi u es. e isco ers a e mass o wa er i e co ai er ecreases
. uri e mi u es.

(a) a cu a e e speci ic a e ea o aporisa io o wa er.

speci ic a e ea o aporisa io .

29
(b) s e a ue ca cu a e i a i er a e ac ua speci ic a e ea o apori a io
o wa er Exp ai w .

(c) o s rie i comme s a i is e er o use a acuum as o co ai wa er ra er


a a s ee co ai er. Exp ai w is is so.

7 co ec or iews a pos a e s amp o ei . cm rou a e s. e e s is . cm rom


e s amp a e ra io o ei o ima e o ei o o ec is . .

(a) i . . comp e e e u sca e ra ia ram o e ermi e e ima e o e s amp


posi io o e e s

ima e iewe
rom is si e o
e e s

pri cipa axis

ra

Fig. 7.1 2.0 cm


(b) a e w a is mea a ir ua ima e.

30
(c) se our rawi o e ermi e e oca e o e e s.

oca e .

(d) i . . comp e e e pa o ra a er passi rou e e s.

8 (a) ace a e ro e i e a ma e c ar e posi i e ru i i wi aco u


a copper ro e i e a ca o e c ar e is wa .

(i) Exp ai ow e ace a e ro ac uires posi i e c ar es w e ru wi aco .

(ii) Exp ai w a copper ro e i a a ca o ac uire c ar es ru i wi


aco .

31
(b) i . . s ows a i posi i e c ar e ace a e ro u ree wi a i su a i
rea . ear e me a sp ere is e rou ear i .

me a sp ere i su a i rea

posi i e c ar e
ace a e ro

Fig. 8.1

(i) aew a appe s o e i ace a e ro

(ii) raw e c ar es o e me a sp ere i i . . .

32
9 i . . s ows par o a power ra smissio s s em. E ec rici rom e power s a io
is ra smi e o e users ia ra smissio ca es.

Power s a io
e era or
e users

X Y

ra s ormer X ra smissio i e ra s ormer Y


supp ies o supp ies o
resis a ce
ra smissio i e e users
:

Fig. 9.1

(a) e power s a io is ra smi i a power o . a rom ra s ormer X o


ra s ormer Y ca cu a e e curre owi rou e ra smissio i es.

curre .

(b) a cu a e e power oss i e ra smissio i es w ic as resis a ce.

power oss .

(c) i re ere ce o our a swers rom a a exp ai w e power s a io oes o


ra smi e same power o e ouses a

....

33
Section B

swer all e ues io s rom is sec io .


swer o o e o e wo a er a i e ues io s i Q12.

10 ome cou ries o o a e e ou supp o wa er rom rai or rom ri ers. e scie is s i ese
cou ries are exp ori ew wa s o e i wa er. a a a o e scie is su es s ma i use o
ice er s ou i e a ic cea o o ai wa er. ce er s w ic are ma e rom res wa er
ca e owe o a por i e cou r . ce e arri e e are a owe o me ei er i e su or
e er rom a oca power s a io .

ase o e a e . ou are re uire o e a ua e e easi i i o owi a ice er o a por


i a a a o o ai res wa er.

is a ce e wee e ice er a e por i a a a . x m

era e owi spee . ms

E ec i e sur ace area o ice er expose o e su . x m

u s ra ia io a e Ear s sur ace m

ass o ice er . x

peci ic a e ea o usio o ice . x

E ec rica power ou pu rom oca power s a io

Table 10.1

(a) Exp ai w a is mea e s a eme ice has a specific latent heat of fusion of 3.4 x
105 J/kg.

..

..

(b) a is e ime a e o ow e ice er o e por i a a a

ime a e .

34
(c) a cu a e e o a amou o so ar e er a sor e e ice er w i e i is owe o
e por .

so ar e er .

(d) (i) Es ima e e mass o ice me e e su as e ice er is owe o e por .

mass o ice .

(ii) a e a assump io a ou a e ma e i (d)(i).

(e) ce e ice er reac es e por i ca e me e ei er i e su or e er rom


a oca power s a io . ic is a as er me o o me e ice uppor our a swer
wi appropria e ca cu a io s.

...

...

...

(f) u es a possi e e iro me a pro em o usi is me o o o ai res wa er.

..

35
11 (a) i . . s ows a simp e se up a ca e use o e ec seismic wa es rom
ear ua es. e se up co sis s o a ar ma e suspe e rom a spri a i rom
a me a ro . e me a ro ra smi s i ra io s rom e Ear a e ma e mo es
i a ou o e coi w e ere is a ear ua e. e coi is co ec e o a ca o e
ra osci oscope c.r.o. a mo i ors e e.m. across e coi .

Fig. 11.1 Fig. 11.2

i . . s ows e race a was isp a e o e c.r.o. uri a par icu ar ear ua e.


Eac comp e e osci a io o e same ma i u e represe s o e remor.

(i) escri e a exp ai ow a race s ow o e c.r.o. i i . . is o ai e w e


ere is a ear ua e.

(ii) i . . i ica e e irec io o e curre i e coi w e e sou po e


o e ma e is mo i i o e coi .

(b) ou pu o a e o . rom a e era or is co ec e o e primar coi o a s ep


up ra s ormer wi a ur s ra io o . e curre i e seco ar coi is . m .
e ra s ormer is e icie .

(i) ae e me a use or e core o a ra s ormer.

36
(ii) a cu a e e curre i e primar coi .

curre .

(iii) a e wo reaso s w a pica ra s ormer is o e icie .

37
12 EITHER

s u e ma es a . a er co ec i wo ce s o e ec romo i e orce e.m. . . i


para e . e a er a amme er wi i ere ra es a ree i ere resis ors are use o se
up e circui s ow i i . . .

Fig. 12.1

(a) a e a exp ai o e a a a e o usi wo ce s i para e ra er a usi a si e


. ce .

..

..

..

(b) e o a resis a ce o e circui is . :.

a cu a e e resis a ce o X.

resis a ce o X .

(c) (i) e ermi e e rea i o e amme er.

rea i .

38
(ii) u es a sui a e ra e or e amme er.

(d) ae e po e ia i ere ce p. . across e


(i) . resis or a

p. . .

(ii) . resis or.

p. . .

(e) e s u e se s up a seco circui usi a aria e .c. power supp a amme er a


a me a i ame amp. e circui is s ow i i . . .

Fig. 12.2

e .c. power supp is se o a e amme er rea i is . . e su e


c a es e e.m. . o e .c. power supp o . . e amp ims a e amme er
rea i c a es.

(i) aea exp ai w a appe s o e resis a ce o e i ame amp.

(ii) a e w e er e ew amme er rea i is ess a e ua o or rea er a .


.

39
12 OR

(a) a par icu ar i experime a ra o i is passe rou wa er i o air as s ow


i i . . .

Fig. 12.3

Exp ai w e ra o i c a es i s irec io w e i emer es rom wa er as s ow .

..

..

..

(b) e experime i a is repea e usi a semicircu ar ass oc as s ow i


i . . . e re rac i e i ex o ass is . .

Fig. 12.4

(i) Exp ai w e ra o i oes o c a e irec io w e i e ers e ass.

40
(ii) Exp ai w e ra o i oes o emer e rom e s rai e e AB o e
ass oc . ow re e a wor i .

(iii) i . . raw accura e e comp e e pa or e ra o i u i i


emer es rom e ass oc a ai .

(iv) e spee o i i air is . m s. a cu a e e spee o i i e


ass oc .

spee .

END OF PAPER
Setter: Mr Luqman

41
42
Answers to 2018 PRELIM 4E Physics 6091
Paper 1

1 11 21 31
31
2 12 22 32
3 13 23 33
3
4 14 24 34
5 15 25
25 35
35
6 16 26
26 36
36

43
7 17 27
27 37
37
8 18 28
28 38
9 19
19 29
29 39
10 20
20 30
30 40
Paper 2
Section A

QN SOLUTION MARK
MARKER’S
KER’S REPORT
REPORT

1 (a) orrec s ape e ore re ou


orrec s ape a er re ou

44
(b) a u

x x
x x

ms

(c) e ou ei x x m
isp aceme o e a m

(d) a e i e oci ms

27
X=2
272
72 N
ș accep s o

accep s ra es rom ɽ =3
37
37°

o

arrows poi s i e Y =450 N


correc irec io s

ca e mar s o i e

45
osu e sw o i e
a sca e 1:50 +/- 10N o pro i e e
orces a are represe e Z = 360 N
correc a accura e . E se o
mar e e i e sca e is correc
corr
co rrec
ec .

3 (a) ow wa
war
ar arrow ra
raww i e mi e o e ro . i ica e or
wei . us i iica
ca e is a ce e. . . m rom poi

(b) a i mome
mome s a ou a e ui i rium
x . x . x . x .
awar
a waar mar i ere are correc ca cu a io o mome s
. . s
(c) oa pwar s orce oa ow war s orce
. .

4 (a) ma
xa
a . ms

(b) cce era io wi ecrease i i ecomes ero


esu a orce wi ecrease ecause air resis a ce ass i crease
crre
eas
as e

cce era io ecomes ero w e orwar orce e u


ua aiir
a
a air
resis a ce.
(ci)
a u
.
s
(cii)
is a ce rea u er pee ime rap

46
is
is m
(d)
o pro i e a s ream i e o
ce
or er o re uce air resis a ce

5 (a) i is ope o e a mosp sp ere.


ere
e re. i ce a mosp eric pressure is
cm e mer
mercur
ercu
curr ca rrise
ise
is e o a maximum
maximum ei o cm.

co e
e u e is co ec
c e o a acuum is mea s a ere is o
as pressure ac
ac i o e op sur ace o i ui i e u e ere ore
merc
me r ur wi
e mercur ris
w riseisee o a maximum o cm .

(b) P p
. x x
. x Pa
(c) ei i creases as pressure o mercur sur ace i creases
creases.
s.

e empera ure i creases a era e .e. o mo ecu e


ess i crccreases.
rea
ase
ses.
o ecu es mo e as er a i e mercur sur ace wi ea
e
rrea er o
a er rce
orce
a a i er re ue c .

causi orce per u i area or pressure o i cr


crease.
reea
asse
e.

6 (a) m
m
x . x x . x
or

(b) es i is i er.
u ro
ur ou i s
ere is heat lost o e ssurrou cco
o uc io co ec io
uc
a ra ia io . e amou o e ea ecei
ec ei e
a rrecei e wa er or oi i
is us smaller a e cca
a cu a e a ue.ue

47
(c) u
e use o acuum ass re ucec s e
uces ea
a os o e surrou i s
co uc io
o co ec io
io a rara ia io .
7 (a) position of the lens

image viewed
from this side of
1
the lens
1

ray B

2.0 cm

Fig. 6.1 2.0 cm

(b) ir ua ima e is a ima e a


a ca o orm
rme
e orme
e o a scree .

48
(c) . cm

(d) correc rawi usi co cep ra a rom


rom o o om o o ec
ima e
ori i a e rom o om o ima

8 (ai) e electrons are ra a s erre


erre
re rom a oms o e ace a e ro o e
co ue o ri
u rric
c io o ruu i .
ce a e ro
ro oses es e ec
ec ro s a ecome posi i e c ar e .

(aii) opper iiss a co


c nd
duc
ucto
t r.
conductor. e ac uire c ar es ru i wi e
e
euu ra i e e a .

(bi) e acce a e ro is a rac e


ace o e sp ere. mo es c oser o e
sp e re.
re
ere.

(bii) e a i e c ar es o e ri si e o sp ere
9 (a) urre Power o a e
.

(b) Power oss . x . x


.

(c) e o a e is wi e i .
e is curre ows rou e ra smissio ca
ca e o
e ea e era e wi e er er
i . is resu s i e er os
o ss
oss
a mo e ar oss.

Section B

QN SOLUTION MARKER’S REPORT

49
10 (a) . x o erma e er is re uire uire o c a e o ice
icce e
rom so i o i ui s a e or ice ers
rsa
ersa wi o
a wi ouu ac a ei
empera ure.

(b) ime
me re uiire
r
re uire . x x .
. x s

(c) E ecc i e so pow


owe
so ar power
er recei e
. x x
. x

rom
o P E
rom
E . x x . x
. x
(di) rom m
m . x . x
. x ec i e

(dii) ere is o ea ai e surrou i e e er


supp ie e su is a sor e e ice er .

(e) aeo ea i e su . x x . x
aeo ea i e power s a io x

si e er rom e power s a io is a more e icie wa


wa o
me e ice.

(f) uri
e ice a me s uri e owi pro
oce
cess
owi processss wi
wi co ri u e o
risi sea e e s w ic ca cause
cau
a se oo i i ow w i areas.

50
oss o a i a s

11 (ai) uri a ear ua e e ma m e m


mo
o es i a ou o coi
pro uci a c a e i ma e ic ux i i e coi us
uci
c
i uci a em a e so
so e oi .

e s aw w e ire
iirec
r c io o e em c a es w e e
ma e m o es i a ou o e coi
mo
ce
e c e a a er a i race is pro uce .

ara
a ra a s aw e ma i u e o e i uce em is
p op
pr
propor
opor io a o e ra e o c a e o ma e ic ux i a e
e ce a ar er remor wi pro uce a race wi a i er
amp i u e.
(aii)

(bi) o iro
(bii) s x .

51
. x pp s s
. x . x p x .
p .
(biii) uce e ccurre
urre s are
ur are orme
orm
o rmee i e core o e
ra s ormer.
or e
ere
re iis
s e
eaa os
oss s uue o e resis a ce i e
wires.. ere
e is ma e ic ux ux ea a e e wee e primar
a seco ar coi coi . a w
wo
12E (a) o eo
e cece s as o er
eas
e
easo
aso o Eac E c ce
Ea ce supp ies a e curre ee e .
e circui
cirrcu
cuii co i ues o wor i o e ce is a .
eas
easoso e circui is s i a c ose circui .
ce ca e rep ace wi ou swi c i o .
easo e circui is s i a c ose circui .
(b) oa o circui o . oa o . i para e
. . .
. .
. . x
.
. s. .
(ci)
. .
. s. .
(cii) o . accep i . e.c. i e
d(i) p. . .
d(ii) p. . .
(ei) e resis a ce o e i ame amp ecreases.

i ess o a e across e i ame amp


amp
mp a e cce
e ess
ess
s
curre rou e amp e empera ure
e o e i ame
am
me
e ampam
mp
ecreases.

(eii) e ew amme er rea i is ess a . .


12O (a) ir is op ica ess e se a wa er.
pee o i i creases as i e s rom wa err o air
ar
ai

52
ausi i o e awaaw rom
m orma

mar s or a poi s
mar or poi s

(bi) e o i ci e ce iss o. i ci e ra
a ies
ies a o e orma .

(bii) si c
c si
s .
. o

c e c
e o i ci cee rea er a cri ica a e o a i er a
re
e ec iio
o occurs.
occu
oc curs.

i mo
m i rom op ica e ser me ium ass o op ica
ess
ess e se me ium air .
(biii) o a poi o i ci e ce e ec e ra s rai ou i o
air.

(c) c
. .
. x m s

53
54
Bukit Batok Secondary School
%(285%(67
O-LEVEL PRELIMINARY EXAMINTION 2018
%8.,7%$72. SECONDARY FOUR EXRESS
6(&21'$5<6&+22/

PHYSICS 6091 / 01
Paper 1 Multiple Choice 21 August 2018
1115 ņ 1215 hrs
1 hour

Additional Materials: Multiple Choice Answer Sheet

READ THESE INSTRUCTIONS FIRST

Write in soft pencil.


Do not use staples, paper clips, highlighters, glue or correction fluid.
Write your name, class, and class register number on the answer sheet in the spaces provided unless
this has been done for you.

There are forty questions on this paper. Answer all questions. For each question, there are four
possible answers A, B, C and D.
Choose one you consider correct and record your choice in soft pencil on the separate Answer Sheet.

Read the instructions on the Answer Sheet very carefully.

Each correct answer will score one mark. A mark will not be deducted for a wrong answer.
Any rough working should be done in this booklet.
The use of an approved scientific calculator is expected, where appropriate.

This document consists of 15 printed pages (including this cover page)

55
2018 / O-Level Prelims / Sec 4E / Physics 6091 / Paper 1

1 Which of the following quantities is a base quantity?

A charge
B energy
C force
D temperature

2 A student wanted to measure the diameter of a pen.

Which steps provide most accuracy in the measurement?

A Take average values of the diameter using a ruler.


B Take average values of the diameter using vernier calipers with zero error.
C Take average values of the diameter using vernier calipers without zero error.
D Take one value of the diameter using a micrometer with zero error.

3 The distance-time graph below shows the distance travelled by a moving car. P represents the
area under the graph.

distance / m

time /s

Which of the following expressions would be used to determine the average speed of the car?

A P/t
B d2 / t
C (d2 + d1) / t
D (d2 ņ d1) / t

4 A bus was travelling at a speed of 20 m/s. When the bus was 50 m from a traffic light, the light
turned red. The bus driver reacted immediately and applied a constant braking force for 5.0 s.
The bus decelerated at 4.0 m/s2.

Which of the following correctly describes the motion of the bus?

A The bus stopped past the traffic light.


B The bus stopped at the traffic light.
C The bus stopped before the traffic light.
D The bus continued to move past the traffic light with decreasing speed.

$SSO\3DVW.QRZOHGJHWR1HZ6LWXDWLRQV Page 2

56
2018 / O-Level Prelims / Sec 4E / Physics 6091 / Paper 1

5 A toy rocket is accelerated vertically upwards. The velocity-time graph below shows the
relationship between the velocity of the rocket and time.

v / m sņ1

time / s

Which of the following statements is/are correct?


(1) The rocket reaches the highest position at instant X.
(2) The resultant force acting on the rocket is zero at instant Y.
(3) The rocket is still in the air at instant Z.

A (1) only
B (3) only
C (1) and (2) only
D (2) and (3) only

6 A pendulum bob of mass 2.0 kg is pulled horizontally to the left by a 30 N force as shown.

Which will be the approximate final tension in the string when the bob is in equilibrium?

A 30 N
B 36 N
C 40 N
D 45 N

7 A constant force of 40 N acts on an object of mass 2.0 kg. The object moves along a rough
horizontal surface with an acceleration of 5.0 m/s2.

What is the average frictional force acting on the object?

A 8.0 N
B 10 N
C 30 N
D 50 N

$SSO\3DVW.QRZOHGJHWR1HZ6LWXD Page 3

57
2018 / O-Level Prelims / Sec 4E / Physics 6091 / Paper 1

8 Which property of an object resists a change in the state of rest or motion of the object?

A acceleration
B density
C mass
D volume

9 A bottle full of mercury has a mass of 730 g. When the same bottle is filled with an unknown
liquid P, its mass is 100 g. The mass of the empty bottle is 50 g.

Calculate the density of the unknown liquid P. (Take density of mercury to be 13.6 g/cm3)

A 1.0 g/cm3
B 2.0 g/cm3
C 7.0 g/cm3
D 14.4 g/cm3

10 A uniform rectangular board (8.0 m x 2.0 m), pivoted at its centre X, is acted on by three forces
on the edges.

10 N

5.0 N

What is the size of force F such that the board remains in equilibrium?

A 40 N
B 45 N
C 85 N
D 90 N

11 A non-uniform object is placed on an inclined plane as shown below.

If the object is just about to topple, which position is the centre of gravity?

$SSO\3DVW.QRZOHGJHWR1HZ6LWXD Page 4

58
2018 / O-Level Prelims / Sec 4E / Physics 6091 / Paper 1

12 A student left two identical, heavy, stone blocks resting on soft earth.
One is vertical and the other is horizontal as shown in the diagram below.

Which row correctly compares the force and the pressure that the two blocks exert on
the earth?

force pressure
A same different
B same same
C different different
D different same

13 One end of an oil-filled manometer is connected to a pump. The other end is open to the
atmosphere.

Which length is used when calculating the difference between pressure of the air in the pump
and atmospheric pressure?

to pump
A

C
B
oil

14 A block of wood is pulled along a horizontal bench at a constant speed of 6.0 m/s by a force of
8.0 N.

6.0 m/s

8.0 N

How much work is done in 5.0 s against the frictional force?

A 40 J
B 48 J
C 240 J
D 480 J

$SSO\3DVW.QRZOHGJHWR1HZ6LWXD Page 5

59
2018 / O-Level Prelims / Sec 4E / Physics 6091 / Paper 1

15 The diagram below shows a motor having a power input of 500 W.

Rope pulled
down Weight
600 N

Power input
500 W 20 s
10 m
taken
motor

It is used to lift a load weighing 600 N through a vertical height of 10 m in 20 s.

What is the useful power output of the motor?

A 300 W
B 390 W
C 500 W
D 800 W

16 When a tennis ball drops onto a hard and smooth horizontal surface, it bounces up and down in
the air. The height of each bounce gradually reduces until the ball stops moving.

Which of the following statements is true?

A The kinetic energy of the ball is constant.


B The potential energy of the ball is constant.
C The sum of the kinetic energy and potential energy of the ball is constant.
D The sum of the kinetic energy and potential energy of the ball is not constant.

17 Gas inside a cylinder is cooled slowly to a lower temperature. The pressure inside the cylinder
remains constant as the piston moves inwards.

direction of motion
gas

How do the speed of the particles and their rate of collisions with the cylinder and piston
compare with their initial values at the higher temperature?

average speed rate of collision


A lower reduced
B lower increased
C same same
D same reduced

$SSO\3DVW.QRZOHGJHWR1HZ6LWXD Page 6

60
2018 / O-Level Prelims / Sec 4E / Physics 6091 / Paper 1

18 The figure below shows a rug and a tiled floor. The rug has been on the floor for a long time.

Which statement(s) correctly explain(s) why the floor feels colder than the rug?

(1) The floor is at a lower temperature than the rug.


(2) The floor is a better conductor of heat than the rug.
(3) The floor has a smaller specific heat capacity than the rug.

A (1) only
B (2) only
C (2) and (3)
D (1), (2) and (3)

19 The diagram shows a beaker of water placed near a burning candle.

beaker

burning
candle water

How does thermal energy from the candle reach the water in the beaker?

A conduction, followed by convection


B radiation, followed by conduction
C convection, followed by conduction
D radiation, followed by convection

$SSO\3DVW.QRZOHGJHWR1HZ6LWXD Page 7

61
2018 / O-Level Prelims / Sec 4E / Physics 6091 / Paper 1

20 The diagram shows the rise in temperature of 2.0 kg of a substance, X. The substance is initially
in solid state and it was heated uniformly at the rate of 2000 J/min.

temperature / °C

time / min

Which of the following sets of data about X is correct?

specific heat capacity of solid X specific latent heat of fusion of X


in J / (kg °C) in J / kg
A 8000 6000
B 1330 6000
C 4000 3000
D 1330 3000

21 The water taken from two buckets is mixed together. One bucket contains 5.0 kg of water at
20 °C and the other contains 1.0 kg of water at 80 °C.

What is the final temperature of the mixture, assuming no heat is lost to the surroundings?

A 30 °C
B 50 °C
C 60 °C
D 70 °C

22 Which of the following changes in physical property cannot be used for temperature
measurement?

A e.m.f. of a battery
B electrical resistance of a solid
C pressure of a gas
D volume of a liquid

$SSO\3DVW.QRZOHGJHWR1HZ6LWXD Page 8

62
2018 / O-Level Prelims / Sec 4E / Physics 6091 / Paper 1

23 The diagram shows circular wavefronts moving from X to Z as seen from the top.

The distance between Y and Z is 1.2 m and the frequency of the dipper at X is set at 15 Hz.

What is the speed of the wave?

A 4.5 m/s
B 6.0 m/s
C 12.5 m/s
D 18.0 m/s

24 A student makes three statements.

(1) All electromagnetic waves can travel in a vacuum and in air.


(2) All waves obey the laws of reflection but some waves do not obey the laws of refraction.
(3) Sound is a longitudinal wave which travels in a direction parallel to the direction of
vibrations.

Which statements is/are not correct?

A Statement 1
B Statement 2
C Statements 1 & 2
D Statements 2 & 3

25 Statements 1 and 2 are about signals passing through an optical fibre of refractive index of 1.5.

Statement 1: The speed of the signal in the optical fibre is 2.0 x 108 m/s
Statement 2: There is less signal loss in the optical fibre than in a copper cable.

Which statements are correct?

A statement 1 only
B statement 2 only
C statement 1 and 2
D neither of the statements

26 Which of the following is arranged in order of decreasing wavelength?

A microwaves, X-rays, visible light


B radio waves, ultraviolet rays, infra-red radiation
C radio waves, visible light, gamma rays
D ultraviolet rays, visible light, infra-red radiation

$SSO\3DVW.QRZOHGJHWR1HZ6LWXD Page 9

63
2018 / O-Level Prelims / Sec 4E / Physics 6091 / Paper 1

27 Which electromagnetic wave is most likely to cause structural damage to living cells and tissue?

A infra-red radiation
B microwaves
C radiowaves
D ultra-violet radiation

28 The diagram shows waveforms produced by a flute (Y) and turning fork (Z) played by two
students.
Pressure variation

time

How does the loudness and pitch of the sound from the turning fork Z compare to flute Y?

A The loudness of Y is lower but has the same pitch as compared to Z.


B Both Y and Z have the same pitch and loudness.
C The loudness of Y is higher and the pitch is lower as compared to Z.
D The loudness of Y is the same and the pitch is higher as compared to Z.

29 A student draws three rays of light from point P through a converging lens.
Each point labelled F is a principal focus of the lens.

ray X

ray Z ray Y

Which of the rays is/are drawn correctly?

A ray Y only
B ray Z only
C ray X and Y
D ray X and Z

$SSO\3DVW.QRZOHGJHWR1HZ6LWXD Page 10

64
2018 / O-Level Prelims / Sec 4E / Physics 6091 / Paper 1

30 The rays of light from a ray-box pass through three lenses placed at positions 1, 2 and 3.

ray box

position 1 position 2 position 3

What type of lens is used at each position?

position 1 position 2 position 3


A converging converging converging
B converging converging diverging
C diverging converging diverging
D diverging diverging converging

31 A positively charged rod is held close to an earthed metal sphere.

sphere

rod

earth

Which of the following describes the charge on the metal sphere?

A It is negative because electrons are attracted towards the rod.


B It is neutral because electrons are attracted towards the rod and protons are repelled.
C It is neutral because it is earthed.
D It is positive because protons are repelled by the rod.

$SSO\3DVW.QRZOHGJHWR1HZ6LWXD Page 11

65
2018 / O-Level Prelims / Sec 4E / Physics 6091 / Paper 1

32 The figure below represents the electric field lines in the vicinity of two isolated electric charges,
P and Q.

Which statement identifies the charges P and Q?

A Both P and Q are negative.


B Both P and Q are positive.
C P is positive and Q is negative.
D P is negative and Q is positive.

33 A circuit is set up in the diagram below.

12 V

2.0 Ÿ 2.0 Ÿ

3.0 Ÿ

2.0 Ÿ

What is the ammeter reading in the circuit?

A 0.50 A
B 0.67 A
C 1.5 A
D 2.0 A

$SSO\3DVW.QRZOHGJHWR1HZ6LWXD Page 12

66
2018 / O-Level Prelims / Sec 4E / Physics 6091 / Paper 1

34 Which one of the following electrical appliances is correctly wired to a three-pin plug?

35 The diagram shows three bars placed in a line. X and Y are both magnets. Z is soft iron.

What are the magnetic forces on X and Z due to magnet Y?

force on X force on Z
A attraction attraction
B attraction repulsion
C repulsion attraction
D repulsion repulsion

$SSO\3DVW.QRZOHGJHWR1HZ6LWXD Page 13

67
2018 / O-Level Prelims / Sec 4E / Physics 6091 / Paper 1

36 Four magnetic compasses are placed near a bar magnet as shown in the figure below.

Which compass is faulty?

37 The following figure shows a beam of electrons entering a magnetic field going into the paper.

How will the beam of electrons be deflected?

A downwards
B into the paper
C upwards
D out of the paper

38 The diagram shows the trace obtained on the screen of an oscilloscope when a given signal is
applied to the input terminals.

The time-base is set at 2.0 ms / div and the y-gain is set at 2.0 V / div.

Which of the following correctly represents the peak voltage and frequency of the signal?

peak voltage / V frequency / Hz


A 4.0 83.3
B 4.0 125
C 8.0 83.3
D 8.0 125

$SSO\3DVW.QRZOHGJHWR1HZ6LWXD Page 14

68
2018 / O-Level Prelims / Sec 4E / Physics 6091 / Paper 1

39 Three identical filament lamps, X, Y and Z, are connected to an iron core with multiple coils.
The resistance of each lamp is 4.5 ȍ and each requires a current of 2.0 A to light up normally.

What can be observed about the brightness of the three lamps?

lamp X lamp Y lamp Z


A dimmer than normal normal brightness brighter than normal
B brighter than normal normal brightness dimmer than normal
C not lit normal brightness not lit
D not lit not lit not lit

40 In the graph shown, the solid curve shows how the e.m.f. produced by a simple a.c. generator
varies with time. The dashed (dotted) curve is the output from the same generator after a
modification has been made to the generator.

Which modification was made to produce the new output shown?

A The thickness of the coil was doubled.


B A split-ring commutator was added.
C The number of turns in the coil was doubled.
D The speed of rotation of the coil was doubled.

------ END OF PAPER ------

$SSO\3DVW.QRZOHGJHWR1HZ6LWXD Page 15

69
ass e is er um er ame

Bukit Batok Secondary School


O-LEVEL PRELIMINARY EXAMINATION 2018
BE OUR BEST

BUKIT BATOK
SECONDARY FOUR EXPRESS
SECONDARY SCHOOL

PHYSICS 6091 / 02
Paper eor u us
rs
our mi u es

a i a es a swer o e ues io Paper.

i io a a eria s oa i io a ma eria s ee e

READ THESE INSTRUCTIONS FIRST

ri e our ame c ass a c ass re is er um er o a e wor ou a i .


ri e i ar ue or ac pe o o si es o e paper.
ou ma use a so pe ci or a ia rams rap s or rou wor i .
o o use s ap es paper c ips i i ers ue or correc io ui .

Section A
swer all ues io s.

Section B
swer all ues io s. ues io 13 as a c oice o par s o a swer.

a i a es are remi e a all ua i a i e a swers s ou i c u e appropria e u i s.


e use o a appro e scie i ic ca cu a or is expec e w ere appropria e.
a i a es are a ise o s ow a eir wor i i a c ear a or er ma er as more mar s are
awar e or sou use o p sics a or correc a swers.

a e e ra i a io a ie s re g o e w ere ee e .

e e o exami a io as e a separa e a swer paper o e ues io Paper.


e um er o mar s is i e i rac e s a e e o eac ues io or par ues io .

For Examiner’s Use


Section A
Section B
Total

is ocume co sis s o 19 pri e pa es i c u i is co er pa e .

70
e e Pre ims ec E P sics Paper
Section A 50 marks
swer all e ues io s i is sec io .

1 a e e uses a exercise mac i e as s ow i i . . .

exercise
machine

track

Fig. 1.1

e a e e mo es orwar a ac war s a o e rac . s e pu s e a e e mo es


ac war s a e mac i e isp a s e orce exer e e a e. e e o e rac e
s ops pu i a e re ur s o e posi io s ow . is is o e comp e e mo eme .

(a) accura e a ue or e a era e power or o e comp e e mo eme is o ai e .

escri e w a measureme s are a e a ow e are use o i e a era e power or


o e comp e e mo eme .

... ... .

... ... .

... ... .

... ... .

... ... .

... ... .

(b) orces ac o ea e e as e exercises. e i a ac io reac io pair o orces.

... ... .

... ... .

Apply Past Knowledge to New Situati Pa e 2


71
e e Pre ims ec E P sics Paper
2 su e sa s ear ee eo aci . e rows a a upwar s as s ow i i . . .

highest point of
motion

path of
ball

Fig. 2.1

e isp aceme ime rap or e irs . s o mo io is s ow i i . . . ir resis a ce is


e i ei e irs . s o mo io .

Fig. 2.1

(a) Es ima e e a era e e oci o e a rom . s o . s

a era e e oci .. ..

(b) e a co i ues o a a er . s. e e ec o air resis a ce ecomes si i ica a e a


e e ua reac es ermi a e oci .

escri e ow e e oci a acce era io o e a c a es as i reac es ermi a e oci .

... ... .

... ... .

... ... .

Apply Past Knowledge to New Situati Pa e 3


72
e e Pre ims ec E P sics Paper
3 i . . s ows a s a io ar pis o i a c i er. co sis s o a pis o o o e e a is sea e o
e o er e .

piston gas cylinder Sealed end

Fig. 3.1

e as i ec i er exer s e same pressure o e pis o as i oes o e sea e e .

(a) si i eas a ou mo ecu es exp ai w e pressures are e same.

... ... .

... ... .

... ... .

(b) e pis o is e i p ace a e c i er is coo e . empera ure o e c i er a as


roppe .

si i eas a ou mo ecu es s a e a exp ai ec a e i pressure o e as.

... ... .

... ... .

... ... .

... ... .

Apply Past Knowledge to New Situati Pa e 4


73
e e Pre ims ec E P sics Paper
4 i . . s ows s eam rom a oi er passi rou a ur i e co ec e o a e era or.
eam passes rou e ur i e i e coi e copper u e a co e ses i e co e ser.
e i er a e er o e seawa er rises.

turbine

steam in generator

coiled
copper tube condenser

seawater in seawater out


Fig. 4.1

(a) ae e e ec s o co e sa io o s eam o e mo ecu es o e seawa er.

... ... .

... ... .

... ... .

... ... .

(b) e seawa er e ers e co e ser a a empera ure o a ea es a a empera ure o


. a cer ai ime o erma e er passes i o e seawa er.

e speci ic ea capaci o seawa er is . .

a cu a e e mass o seawa er a e ers e co e ser i is ime.

mass .. ..

Apply Past Knowledge to New Situati Pa e 5


74
e e Pre ims ec E P sics Paper
5 i . . raw o sca e s ows ra s o i ra e i i wa er rom a i source O.

air

water

Fig. 5.1 (to scale)

(a) (i) easure a wri e ow e si es o a es A a B.

a eA .. a eB ..

(ii) e ce e ermi e e re rac i e i ex o wa er.

re rac i e i ex .. ..

(b) Exp ai w e i ra oes not escape rom e wa er sur ace a poi C.

... ... .

... ... .

... ... .

Apply Past Knowledge to New Situati Pa e 6


75
e e Pre ims ec E P sics Paper
6 ome e ec rica compo e s are easi ama e i e ec ric c ar e is p ace o em. e are o e
s ore p aci em i co ac wi a co uc or.

plastic electrical component

metal legs
Fig. 6.1

(a) e e compo e s ow i i . . is ru e wi a woo e c o e me a e s ecome


e a i e c ar e .

Exp ai ow is appe s.

... ... .

... ... .

(b) i . . s ows e e a i e c ar e me a e s p ace ear a piece o u c ar e a umi ium oi


w ic res s o a i su a or.

plastic

metal leg

aluminium foil

insulator
Fig. 6.2

Exp ai w e a umi ium oi is a rac e oa s ic s o e me a e s.

... ... .

... ... .

... ... .

... ... .

Apply Past Knowledge to New Situati Pa e 7


76
e e Pre ims ec E P sics Paper
7 i . . s ows a coi o wire co ec e exi e ea s o a swi c a a a er .

coil

Fig. 7.1

e coi is p ace e wee e po es o a perma e ma e a is ree o ur a ou e axis.


e e swi c is c ose orces ue o e curre ac o e si es o e coi .
e coi s ar s o ur .

(a) (i) i . . raw a arrow o s ow e irec io o e orce o e coi ex o e


po e o ma e .

(ii) Exp ai ow ou o ai e our a swer.

... ... .

... ... .

... ... .

... ... .

(b) e coi s ops w e i is er ica . Exp ai w e ur i e ec o e orces is ero a is


posi io .

... ... .

... ... .

(c) or er or e coi o ro a e co i uous a sp i ri commu a or is co ec e e wee e


a er a e coi .

Exp ai ow e sp i ri commu a or e a es e coi o ro a e co i uous .

... ... .

... ... .

... ... .

... ... .

Apply Past Knowledge to New Situati Pa e 8


77
e e Pre ims ec E P sics Paper
8 s u e co s ruc s a mo e o a circui rea er usi a u ma e i e s ee core as s ow i
i . . . e is a ce rom e s i spri o e pi o is cm. e iro arm ca mo e ree a ou
e pi o .

fixed spring
contacts
connection
pivot
25 cm
terminal P

terminal Q
iron arm
coil

Unmagnetised
steel core

Fig. 8.1

(a) curre owi rom ermi a P o ermi a Q causes e co ac s o separa e.


Exp ai w .

... ... .

... ... .

... ... .

... ... .

(b) esu e i s a is mo e ca o e use o ce.

u es a exp ai one reaso w .

... ... .

... ... .

... ... .

(c) u es a exp ai one mo i ica io suc a e circui rea er wi e ac i a e o wor a a


muc ar er curre .

... ... .

... ... .

... ... .

... ... .

Apply Past Knowledge to New Situati Pa e 9


78
e e Pre ims ec E P sics Paper
9 i . . s ows a circui co ai i a i ame amp a a power supp .

X Y
Fig. 9.1

e aria e resis or XY is ma e rom a o resis a ce me a wire. e si i co ac C mo es


ao e wire rom X o Y.

e wire XY o e m s aw.

(a) a e w a appe s o e rea i s o e ree me ers as mo es rom o .


i e umerica a ues w ere possi e.

o me er ..

... ... .

o me er ... . ..

... ... .

amme er ... . ..

... ... .

(b) e aria e resis or is a us e so a e po e ia i ere ce across e amp is . .


e curre owi rou e amp is m .

e ermi e e amou o c ar e a ows rou e amp i . mi u es.

c ar e .. ..

Apply Past Knowledge to New Situati Pa e 10


79
e e Pre ims ec E P sics Paper
10 s rai e o copper wire ies ori o a e wee e po es o a s ape ma e .
i . . s ows e wo e s o e wire co ec e o a er se si i e ce re ero amme er.

copper wire

sensitive centre-zero ammeter


Fig. 10.1

e copper wire is mo e upwar s uic e wee e wo ma e ic po es. e ee e o e


amme er e ec s mome ari .

(a) (i) Exp ai w e ee e o e amme er e ec s.

.. ... ... .

.. ... ...

(ii) raw a arrow o i . . o s ow e irec io o curre owi i e par o copper


wire e wee e wo ma e ic po es.

(b) e wire is ow mo e ow war s s ow e wee e wo ma e ic po es.

aew a appe s o e ee e o e amme er.

... ... .

... ... .

Apply Past Knowledge to New Situati Pa e 11


80
e e Pre ims ec E P sics Paper
Section B 30 marks
swer all e ues io s i is sec io .
swer o o e o e wo a er a i e ues io s i Question 13.

11 e o resis i e wire is cu i o wo pieces E a F. Eac piece o resis i e wire is c ampe i


ur e wee wo me a c ips as s ow i i . . . e e o wire e wee e c ips is . cm
or wire E a . cm or wire F.

metal clip metal clip metal clip metal clip


resistive wire F

resistive wire E
Fig. 11.1

(a) e po e ia i ere ce p. . across eac wire is s ow i crease . e p. . is measure a


arious a ues o curre . i . . s ows e rea i s o ai e .

wire E wire F
curre p. . p. .

. . .
. . .
. . .
. . .
. . .
. . .
. . .
. . not measurable
. not measurable
Fig. 11.2

(i) si a a rom i . . escri e e re a io s ip e wee e curre a e p. .


across E or i curre s.

.. ... ... .

.. ... ...

(ii) i e a e circui is i wor i co i io s su es w e p. . is not measura e or


wire F a a i curre .

.. ... ... .

.. ... ... .

.. ... ...

(iii) e ermi e e resis a ce o e resis i e wire E w e e curre is .

resis a ce .. ..

Apply Past Knowledge to New Situati Pa e 12


81
e e Pre ims ec E P sics Paper
(iv) raw a a e a sui a e e ec ric circui ia ram o e experime se up o e e
rea i s o o e o e resis i e wires i i . . .

(v) o e experime o e wires a a . r ce were co ec e i series.


e uce e curre owi rou e wire. Exp ai our a swer.

.. ... ... .

.. ... ... .

.. ... ... .

.. ... ...

(b) e experime is repea e wi a s ro wi owi o er e wires e use o a a e a .

i . . s ows e ew rea i s o ai e a se ec e curre s.

wire E wire F
curre p. . p. .
. . .
. . .
Fig. 11.3

(i) u es w e a ues o e p. . a e same curre i i . . are ower a a i


i . . .

.. ... ... .

.. ... ... .

.. ... ...

(ii) u es o e o er i ere ce a is see or rea i s o p. . a a ues o curre rea er


a . as compare o ose i i . . .

.. ... ... .

.. ... ...
Apply Past Knowledge to New Situati Pa e 13
82
e e Pre ims ec E P sics Paper
12 i . . s ows e asic s ruc ure o a ra s ormer.

iron core

230 V

primary coil Secondary coil


Fig. 12.1

a er a i o a eo is app ie o e primar coi a a o a e is i uce i e


seco ar coi . e seco ar coi is co ec e o a amp.

(a) escri e w a is mea a alternating voltage.

... ... .

... ... .

(b) u es a reaso or usi a iro core i e ra s ormer.

... ... .

... ... .

(c) e primar coi as ur s.

a cu a e e sma es um er o comp e e ur s i e seco ar coi a wou i ea


i uce o a e o a eas i e seco ar coi .

um er o ur s

Apply Past Knowledge to New Situati Pa e 14


83
e e Pre ims ec E P sics Paper
(d) e ra s ormer o e s e pri cip e o co ser a io o e er . s u e e ermi es e i pu
power a e ou pu power o e ra s ormer a ca cu a es e e icie c o e ra s ormer.

(i) ae e pri cip e o co ser a io o e er .

.. ... ... .

.. ... ... .

.. ... ... .

.. ... ...

(ii) e amp is ra e . . a cu a e e curre i e primar coi w e e amp


wor s orma .

curre

(iii) rea i e primar coi as a curre o . .


a cu a e e e icie c o e ra s ormer.

e icie c

Apply Past Knowledge to New Situati Pa e 15


84
e e Pre ims ec E P sics Paper
13 EITHER

a io a spri oar i er rai s re u ar . e poo e c im e up a i o s airs wi a o a


er ica ei o . m o reac a spri oar a e op i seco s. i . . s ows im
asce i i e air uri a ump.

7.0 m 30 cm
5.2 m

Fig. 13.1 Fig. 13.2

(a) escri e e mai e er c a ew e e i er c im e up e i o s eps.

... ... .

... ... .

(b) i e a e i er s wei is i is a era e power uri e c im .

a era e power

(c) e i er use e e as ic spri oar o prope imse a is ce re o ra i reac e a


maximum ei o . m rom e sur ace o e wa er e ore p u i ow .

i . . s ows e spri oar i ac io w e i was use o prope e i er upwar s a a


cer ai spee . e spri oar mo e ow a er ica is a ce o cm uri is i s a
spru ac o ori o a posi io a e i er e e spri oar w e i was ori o a .

(i) e ermi e e er ica spee a w ic e i er e e spri oar .

er ica spee

Apply Past Knowledge to New Situati Pa e 16


85
e e Pre ims ec E P sics Paper

(ii) i e a era e upwar orce exer e e spri oar o prope e i er upwar s.

a era e orce

(d) i . . s ows a ipper a wi i ra e up a ow er ica as e w ee ur s crea i


wa es o a wa er sur ace. irec curre .c. mo or ri es a w ee .

Wheel connected to d.c. motor

dipper
water surface

Fig. 13.3

(i) i re ere ce o i . . exp ai w a is mea ra s erse wa e mo io .

.. ... ... .

.. ... ... .

.. ... ... .

.. ... ...

(ii) e w ee ma es re o u io s e er mi u e.

e ermi e e re ue c o e i ra io o e ipper.

re ue c

Apply Past Knowledge to New Situati Pa e 17


86
e e Pre ims ec E P sics Paper

OR

i . . s ows a e ec ric circui ma e wi a i epe e resis or a a aria e


resis or r eos a WKDWLVVHWWRNȍ7KHSRWHQWLDOGLYLGHULVFRQQHFWHGLQVHULHVZLWKD9
power supp a a o me er is co ec e across e .

LDR

12 V

rheostat

Fig. 13.4

uri a experime a i was s o e o e a e resis a ce o e was ȍ

(a) e i e electric current.

... ... .

... ... .

(b) ar ou wi a arrow o i . . ex o e power supp e irec io o co e io a


curre ow.

(c) a cu a e

(i) e curre i e circui

curre

(ii) e rea i s ow o e o me er.

rea i

Apply Past Knowledge to New Situati Pa e 18


87
e e Pre ims ec E P sics Paper

(d) escri e a exp ai ow e rea i o e o me er wou c a ew e e i i e si


ecreases.

... ... .

... ... .

... ... .

... ... .

(e) escri e a exp ai e e ec o i creasi e resis a ce o e r eos a .

... ... .

... ... .

... ... .

... ... .

------ End of Paper ------

Apply Past Knowledge to New Situati Pa e 19


88
89
Q ANSWER Suggested Reason
1 D Recall. 7 base quantities.
2 C Most accurate method (average of few readings and precise instrument)
3 D Distance travelled ÷ Time taken
4 B Area under speed-time graph = ½ (20) (5.0) = 50.0 m. He reacted immediately.
Taking into account negligible human reaction time, he would stop on the line.
5 B Y is the highest point (momentarily at rest at highest point)
To stop, speed should be zero.
6 B For right angled triangle, tension = sqrt (202 + 302) = 36.1 N
7 C Push – friction = m a and hence 40 – friction = 2.0 (5.0)
Friction = 40 – 10 = 30 N
8 C Recall. Inertia depends on mass.
9 A Volume of bottle = mass / density = 680 / 13.6 = 50 cm3
Density of P = mass / volume = 50 / 50 = 1.0 g/cm3
10 B Clockwise moments = anti-clockwise moments
10 (4.0) + 5.0 (1.0) = F (1.0) and hence F = 45 N
11 B Recall. Weight acts through corner (edge) of base.
12 A Recall. Pressure = Force / Area.
13 B Recall. Manometer.
14 C W = F d = 8.0 (6.0 x 5.0) = 240 J
15 A P = F d / t = 600 x 10 / 20 = 300 W
16 D Due to friction, energy is converted to heat and sound ndd and
anndd dissipated
dis
issipa
iss patted away.
aw
wayay.
17 B Pressure inside stays the same so when speed dro drops
ro
ops
ps tthe
hee rrate
h ate o
at off frequency
fre
requ
quen
ency
en cy increases
inc
ncreeases due to
smaller inner wall surface area.
18 C Heat is lost from feet faster on floor tile due to smallerr c and and bet
better
tteer cconductor
onducucto
uc torr of heat.
to
19 B Radiation through the air followed follllow
o ed by conduction of heat through gh gglass.
laass.
20 D Q=mc =Pt
2000 x 4.0 = 2.0 x c x (3 – 0) and hence, c = 1330 J/(kg kg °°C)
C
C)
Q = m l F = Pt
2000 x 3.0 = 2.0 x l F and hence, e l F = 30
3000
0 J/kg
21 A Heat gained
gained by colder water = heat h att lost
he los
o t by hotter
hot
otte
terr water
wate
wa t r
5.0 x c x (new temperature - 20) = 1.0 0 x c x (80 - new temperature)
(80
5T - 100 = 80 – T and hence, e, 6T
6T = 180
180 °C
°C and
and so T = 30 °C
22 A Recall. Thermometric property. properertyty..
23 B wavelength h=1 1.2/3
.2/3
.2 /3 = 0.400m
v = f = 0.40 0. x 15 15 = 6.00
6.00 m/s
m/s
24 B Statements
Statem men ents
ts 2 is incorrect.
in
nco
corr
rrec
ect.
t. It
It can
c n be refracted and reflected.
ca
25 C Speed in fibre e=2 2.0
.0 x 10
108 m/s apply n = c/v.
26 C Recall.
Reeca
callll..
27 D Recall.
Reca
caallll.. Higher
High
Higherr frequency
frequency implies higher penetrating power and larger ionising power.
28 A Recall.
Recallll.. Smaller
Sm
malalle
ler maximum value (smaller amplitude) and same period.
29 C Ray X and annd Ray Y are correct.
30 B Converge means rays go nearer after lens. Diverge means ray move farther apart after lens.
31 A Electrons are attracted up from earth. earth Electrostatic induction.
induction
32 B Recall.
33 D I = V / R = 12 / 6 = 2.0 A (effective resistance = 1 + 2 + 3 = 6 )
34 D Recall. Switch is along live wire. Earth wire touches metal casing.
35 A Unlike poles attract. Z becomes an induced magnet that is attracted to Y.
36 C Magnetic field lines go from right-hand-side pole to left-hand-side pole.
37 A Apply fleming’s left hand rule (conventional current to left)
38 B Amplitude = max value of 2 boxes vertically.
Frequency = 1 / 4 horizontal boxes.
39 C Y is powered by the d.c. cell. X and Z do not lit due to no changing magnetic field (no a.c.
supply).
40 D Halved the period implies twice the frequency.
Twice the amplitude implies twice the rate of cutting of magnetic flux (faraday’s law)

90
B BA SEC NDAR SC
E E RE NAR E A NA N
S CS A ER S ES ED ANSWERS

Q Suggested Ans e
1a x Record down force (using the force meter already on the machine)
x Measure and record down distance = length of rope pulled (using measuring tape or metre
rule)
x Measure and record down time (using stopwatch)
x Calculate Power = ( Force x distance ) / time
x Repeat for another try to get another power reading and then calculate average

KǀĞƌǁŚĞůŵŝŶŐŵĂũŽƌŝƚLJŽĨĐĂŶĚŝĚĂƚĞƐĚŝĚŶŽƚƵƐĞŝŶƐƚƌƵŵĞŶƚƐƚŽŵĂŬĞŵĞĂƐƵƌĞŵĞŶƚƐ͘
1b x Pull on earth on him downwards Pull by him on earth upwards with same size of force
x Pull on man by rope to right pull by man on rope to the left with samee si
ssize
ize
ze o
off force
No a s no d e t on ent oned

KǀĞƌǁŚĞůŵŝŶŐŵĂũŽƌŝƚLJŽĨĐĂŶĚŝĚĂƚĞƐĚŝĚŶŽƚŵĞŶƚŝŽŶĚŝƌĞĐƚŝŽŶĂŶĚ ĚŝĚ
ŝĚ ŶŽƚ
ŽƚŵĞŶ
ĞŶƚŝ
ĞŶƚŝ
ƚŝŽŶƐŝnjĞ͘
Ğ
KǀĞƌǁŚĞůŵŝŶŐŵĂũŽƌŝƚLJŽĨĐĂŶĚŝĚĂƚĞƐĚŝĚŶŽƚŵĞŶƚŝŽŶĚŝƌĞĐƚŝŽŶĂŶĚĚŝĚŶŽƚŵĞŶƚŝŽŶƐŝnjĞ͘
DĂũŽƌŝƚLJŽĨĐĂŶĚŝĚĂƚĞƐŵŝdžĞĚƵƉƚŚĞďŽĚŝĞƐŝŶǀŽůǀĞĚ͘
2a Velocity = gradient of displacement-time graph
= 0 – 1.0 / (1.0 – 0.8)
= – 5.0 m/s (accept 5.0 m/s downwards)

DĂŶLJĐĂŶĚŝĚĂƚĞƐŐŽƚƚŚĞǁƌŽŶŐĂŶƐǁĞƌŽĨϱ͘ϬŵͬƐ
DĂŶLJĐĂŶĚŝĚĂƚĞƐŐŽƚƚŚĞǁƌŽŶŐĂŶƐǁĞƌƌ ŽĨ Ĩ ϱ͘ϬŵͬͬƐƐ
2c x Velocity increases by less and less until n ill it
unt it becomes
mes constant
become
me cco
onstant
x Acceleration decreases unti
until
tiil iitt b
til becomes
ecom
om
omeess zezero
ero

>ĂƌŐĞŵĂũŽƌŝƚLJŽĨĐĂŶĚŝĚĂƚĞƐŽǀĞƌůLJǁƌŝƚĞ͘
>ĂƌŐĞŵĂũŽƌŝƚLJŽĨĐĂŶĚŝĚĂƚĞƐ ŽǀĞĞƌƌůůLJ
LJ ǁƌ
ǁƌŝƚ
ŝƚƚĞ͘
ŝƚĞ͘
Ğ͘ 
3a moving
x Randomly movin ng gas
in gas molecules
ga moleculles hit on n the h inner wall alll randomly
wal do ly with equal probability
raandom
nd
exeert a force
x These collisions exert o ce per
for perr unit
unniit area
aarrea that
hat iss constant
tha co
onnst a t all along inner wall
stan

KǀĞƌǁŚĞůŵŝŶŐŵĂũŽƌŝƚLJŽĨĐĂŶĚŝĚĂƚĞƐĂŶƐǁĞƌĞĚƐĂŵĞĨƌĞƋƵĞŶĐLJĂƐĂĐŽŶĚŝƚŝŽŶ͘
KǀĞƌǁŚǁŚŚĞů
Ğ ŵŝŶŐŶŐ ŵĂũ
ŶŐ Ă Žƌŝƚ
ŝƚƚLJŽĨ
ŽĨĨ ĐĂŶĚŝ ĚŝŝĚĂ
Ě ĂƚĞ
ƚĞƐ
Ɛ ĂŶŶƐǁ
Ɛ ĞƌĞƌĞĚ
ĞĚ ƐĂŵ
ĂŵĞ
Ğ ĨƌĞƋƵĞŶĐLJĂƐĂĐŽŶĚŝƚŝŽŶ͘
DĂũŽƌŝƚLJŽĨĐĂŶĚŝĚĂƚĞƐĂƌĞĐŽŶĨƵƐĞĚďĞƚǁĞĞŶŝŶƚĞƌŵŽůĞĐƵůĂƌĐŽůůŝƐŝŽŶƐĂŶĚĐŽůůŝƐŝŽŶƐĂŐĂŝŶƐƚŝŶŶĞƌ
DĂũŽƌŝƌŝƚLJ
ƌŝ
ŝ LJŽĨĐĂŶĚ ŶĚŝĚ
ŶĚ ĚĂƚĞƐƐ ĂƌĞƌĞĞĐŽŶĨƵƐĞĞĚ ĚďďĞĞƚƚǁ
ǁĞĞ
ĞĞŶŶ ŝŶ
ŝŶƚĞ
ƚĞƌŵŽů
ŽůůĞĐƵůĂƌĐŽůůŝƐŝŽŶƐĂŶĚĐŽůůŝƐŝŽŶƐĂŐĂŝŶƐƚŝŶŶĞƌ
Ž
ǁĂůů ƐƵƌĨĂ
ǁĂ Ĩ ĐĞƐ͘
ǁĂůůƐƵƌĨĂĐĞƐ͘
3b x SpSSpeed
eedd of
of ggas
a mo
as molecules
oleeccules d
ol decrease
ecrreeas
ec ase aand
nd
n d they move less vigorously
x They e hit it inner
hit nnerr walls
inn w lls with
wa wiith
w th small
smmaallll force
o ce and less frequently
for
x Smaller
Smmalalle
l r force exexerted
xer
erted
ted pe
te p
per
eerr un
u
unit
niit ar
area
ea on inner walls (smaller pressure)

DŽƐƚĐĂŶĚŝ
DŽƐƚĐĂŶĚŝĚĂƚĞƐĚŝĚŶŽƚŵĞŶƚŝŽŶŵŽůĞĐƵůĞƐŚŝƚƚŝŶŐƚŚĞŝŶŶĞƌǁĂůůƐůĞƐƐŚĂƌĚ;ǁŝƚŚůĞƐƐĨŽƌĐĞͿ͘
Ě ĚĂ
ĚĂƚĞƐ
ƚĞĞƐ Ěŝ
Ě Ě
ĚŶ
ŶŽ
Žƚ
ƚ ŵĞŶƚŝŽŶŵŽůĞĐƵůĞƐŚŝƚƚŝŶŐƚŚĞŝŶŶĞƌǁĂůůƐůĞƐƐŚĂƌĚ;ǁŝƚŚůĞƐƐĨŽƌĐĞͿ͘
4a x Molecule
Molecules
les off seawater
le sea
eawater roll and slide faster
x Moleculess of of seawater move more vigorously
x Molecules of seawater move farther apart from each other

>ĂƌŐĞŵĂũŽƌŝƚLJŽĨĐĂŶĚŝĚĂƚĞƐĨŽĐƵƐĞĚƚŚĞŝƌĂŶƐǁĞƌŽŶƐƚĞĂŵŵŽůĞĐƵůĞƐůŽƐŝŶŐŚĞĂƚŝŶƐƚĞĂĚŽĨ
ƐĞĂǁĂƚĞƌŵŽůĞĐƵůĞƐŐĂŝŶŝŶŐŚĞĂƚ͘^ŽŵĞĐĂŶĚŝĚĂƚĞƐǁƌŽƚĞĂďŽƵƚŚĞĂƚƚƌĂŶƐĨĞƌŵĞƚŚŽĚƐ͘
4b Q= mc
265 000 000 = m (3900) (64 – 30)
mass = 1998.5 kg = 2000 kg

DĂŶLJĐĂŶĚŝĚĂƚĞƐĐŽƵůĚŶŽƚĐŽŶǀĞƌƚϯ͘ϵŬƚŽϯϵϬϬ͘DĂŶLJĐĂŶĚŝĚĂƚĞƐŵŝƐƚŽŽŬϮϱϲĨŽƌϮϲϱ͘
5a(i) angle A = 42 ° and angle = 30 °
(nearest degree for angle)

91
Q Suggested Ans e
5a(ii) n = sin (large angle) / sin (smaller angle)
= sin A / sin
= sin (42) / sin (30)
= 1.34 (no units)

ĂŶĚŝĚĂƚĞƐǁŚŽǁƌŽƚĞƐŝŶ/ͬƐŝŶƌĐŽƵůĚŶŽƚƐĐŽƌĞĨƵůůĐƌĞĚŝƚĂƐĂŶŐůĞƌŝƐůĂƌŐĞƌƚŚĂŶĂŶŐůĞŝ͘
5b x Angle of incidence in water (optically denser medium) is larger than critical angle
x ight is travelling from optically denser medium to optically less dense medium
x Total internal reflection occurred and light reflects back into optically denser medium

dŚĞĐŽŶĚŝƚŝŽŶŽĨůŝŐŚƚƚƌLJŝŶŐƚŽŵŽǀĞĨƌŽŵŽƉƚŝĐĂůůLJĚĞŶƐĞƌƚŽŽƉƚŝĐĂůůLJůĞƐƐĚĞŶƐĞŵĞĚŝƵŵŝƐŶŽƚ
ŵĞŶƚŝŽŶĞĚ͘
ŵĞŶƚŝŽŶĞĚ͘
6a Metal legs gained (surface) electrons from woollen cloth due to friction.

>ĂƌŐĞŵĂũŽƌŝƚLJŽĨĐĂŶĚŝĚĂƚĞƐƌĞĨĞƌƌĞĚƚŽŶĞŐĂƚŝǀĞĐŚĂƌŐĞƐ͘
6b xx The force of attraction between the positive charges on Al and electrons
electr trron
ons on
ons on metals
metals legs is
larger than the force of repulsion between the electrons on Al and
an
nd eleelectrons
leecctron
ns on metal
o metettaall legs
x As the positive charges on Al are now closer to the metal legss th
x tthan
han
an tthehe elect tro
rons
electrons ns o
off Al
A
Al.l.
x Upward force of attraction is larger than downward weight of
x of Al
Al ffo
oill aass welllll
foil

sĞƌLJĨĞǁĐĂŶĚŝĚĂƚĞƐŵĞŶƚŝŽŶĞĚĂďŽƵƚƚŚĞĞdžŝƐƚĞ ƚĞĞŶĐĞŽĨ
ŶĐ
ŶРƚƚŚ
ŚĂƚ
ĂƚƚƚƌƌĂĐĐƚƚŝŽ
ŝŽŶ
ŶĂ
ĂŶ
ŶĚ
Ě ƌĞƉƵ
sĞƌLJĨĞǁĐĂŶĚŝĚĂƚĞƐŵĞŶƚŝŽŶĞĚĂďŽƵƚƚŚĞĞdžŝƐƚĞŶĐĞŽĨďŽƚŚĂƚƚƌĂĐƚŝŽŶĂŶĚƌĞƉƵůƐŝŽŶ&ĞǁĞƌ Ɖ ůƐŝŽ
ŝŽŶ
Ŷ &Ğ
&ĞǁĞ
ǁ ƌ
ĐĂŶĚŝĚĂƚĞƐǁĞŶƚŽŶƚŽŵĞŶƚŝŽŶƚŚĞǁĞŝŐŚƚŽĨƚŚĞůĨŽŝů͘
ĐĂŶĚŝĚĂƚĞƐǁĞŶƚŽŶƚŽŵĞŶƚŝŽŶƚŚĞǁĞŝŐŚƚŽĨƚŚ ŚĞĞůů ĨŽŝŝů͘
7a(i) Vertically upwards on coil next to N-pol
N-pole
olle of
o magne
magnetet

ĂŶĚŝĚĂƚĞƐĂƌĞƌĞŵŝŶĚĞĚƚŽĚƌĂǁƚŚĞĂƌƌŽǁŽŶƚŚĞĐŽŝůƵƐŝŶŐƌƵůĞƌ͘
 ŶĚŝĚ
Ă ĚĂƚƚĞƐĂƌĞ ƌĞŵŝŶ ŝŶĚĞĞĚƚƚŽĚƌĂǁƚŚ ŚĞ ĂƌƌƌŽ
ƌ ǁŽŶŽŶ
ŽŶ
ŶƚŚĞĐŽŝůƵƐŝŶŐƌƵůĞƌ͘
7a(ii) xx y FFleming’s
leemi
m ng’s lef efft ha
left hand
nd rule,
hand ee,, left
lef
eft middle
mid
mi dddle finger points in direction of current
xx eeft
ftt iindex
nddex
e finger
fiin
nge
ger points in in direction
d re
di rect
ct on of magnetic field lines from N-pole to S-pole
ctio
x
x eeft
ftt tthumb
humb points in in direction
direc
irrec
ection
tion
ti on of magnetic ( orentz) force on coil upwards
7b Force on
on co ccoil
oil aacts
c s tth
ct through
hro
hro ouggh the
th
th hee pivot (M = F d, d = 0 m, M = 0 Nm)

KǀĞƌǁŚĞůŵŝ
KǀĞƌǁŚĞůŵŝŶŐŵĂũŽƌŝƚLJŽĨĐĂŶĚŝĚĂƚĞƐĂƌĞĐŽŶĨƵƐĞĚŽǀĞƌƚŚĞĐƵƚƚŝŶŐŽĨŵĂŐŶĞƚŝĐĨůƵdžǁŝƚŚŵŽŵĞŶƚƐ
ŵŝŝŶŐ
ŵ Ő ŵĂũ
ĂũŽƌ
ũŽƌ
ŽƌŝŝƚƚLJŽĨĐĂŶĚŝĚĂƚĞƐĂƌĞĐŽŶĨƵƐĞĚŽǀĞƌƚŚĞĐƵƚƚŝŶŐŽĨŵĂŐŶĞƚŝĐ ĨůƵdžǁŝƚŚŵŽŵĞŶƚƐ
;Dс&ĚͿ͘
;Dс&ĚͿ͘
7c xx It reverses the direction of current in the coil every half a turn
x Such that the force on coil nearest to N-pole of magnet is always upwards
x
x It also prevents entangling of the external wires from power source during rotation

dŚĞŝĚĞĂŽĨĚŝƌĞĐƚŝŽŶŽĨĨŽƌĐĞďĞŝŶŐƌĞǀĞƌƐĞĚŝƐŶŽƚĐůĞĂƌůLJĞdžƉƌĞƐƐĞĚŵŽƐƚŽĨƚŚĞƚŝŵĞ͘
8a x Current in solenoid creates a strong electromagnet with N-pole at top of coil
x ottom left of iron arm is attracted downwards to the solenoid due to magnetic induction as it
is now an induced S-pole
x Anti-clockwise moment due to downward magnetic force of iron arm about pivot is more than
clockwise moment due to upward force of spring about pivot

KǀĞƌǁŚĞůŵŝŶŐŵĂũŽƌŝƚLJŽĨĐĂŶĚŝĚĂƚĞƐǁƌŽƚĞǀĞƌLJƐŝŵƉůŝƐƚŝĐĂŶƐǁĞƌƐ͕ŽŵŝƚƚŝŶŐĚĞƚĂŝůƐ͘

92
Q Suggested Ans e
8b x Steel core becomes a permanent magnet that cannot be easily demagnetised.
x Iron arm cannot move back up to close the contacts.
x Current cannot flow as contacts stay open.
8c F st o nt
Much larger current will create a much stronger electromagnet.
Hence, we need to reduce the strength of the electromagnet.

x Fewer turns of coil of solenoid so that the circuit breaker will not activate at a low current.
x arger distance ( 25 cm) of spring away from pivot for a larger moment to overcome.
9a x Voltmeter 1 will change from decrease from initial 12 V
x Voltmeter 2 will increase from initial 0 V
x Ammeter will decrease from initial value

KǀĞƌǁŚĞůŵŝŶŐŵĂũŽƌŝƚLJŽĨĐĂŶĚŝĚĂƚĞƐĚŝĚŶŽƚƐƉĞĐŝĨLJŶƵŵďĞƌƐ;ϭϮs͕ϬsͿŝŶ ƚŚĞ
ŚĞŝƌ
ŝƌ ĂŶƐ
ŶƐǁĞƌƐ͘
KǀĞƌǁŚĞůŵŝŶŐŵĂũŽƌŝƚLJŽĨĐĂŶĚŝĚĂƚĞƐĚŝĚŶŽƚƐƉĞĐŝĨLJŶƵŵďĞƌƐ;ϭϮs͕ϬsͿŝŶƚŚĞŝƌĂŶƐǁĞƌƐ͘
9b Q=It
= 0.15 (5.0 x 60)
= 45.0 C (accept 45 C)
10a(i) The copper wire experiences a temporary change in magnetic flux
flu
ux (of
(o
of magnets)
maggnets) linked
ma lin
inke
ked to
o it
it (per
second), induced an e.m.f.
10a(ii) Arrow drawn on wire in-between magnets INT paper

sĞƌLJLJ ĨĞǁ
ǁĐĂŶĚŝŝĚĂ ĚĂƚĞƐŐŽ
ĚĂƚ Žƚ
ƚ ƚŚ
ƚŚĞ ĐŽ
ƚŚĞ ŽƌƌƌĞĐ
Ğ ƚĂŶƐǁĞƌ͘
sĞƌLJĨĞǁĐĂŶĚŝĚĂƚĞƐŐŽƚƚŚĞĐŽƌƌĞĐƚĂŶƐǁĞƌ͘
10b x Sm Smaller
malle
leer sisize
ize
ze ooff momentary
mo
m omeent ary deflection
ntar
Deflection
x Defl leccttiio
onn in
n the
the opposite
th oppo
op posite direction (to the left)
11a(i) The p.d. incr
increases
ccrrea
rea
ease
s s proportionally
prop
pr o ortionally /uniformly / constantly
constantly / linearly as current increases
11a(ii) current
x High curre reen aused overheating of the wire (P = I2 R heating) and wire melted
ntt ccaused

verwhelming ma ority of candidates wrote that the voltmeter range is 0 to 3 V and hence could
not measure more than 3 V. However, under working conditions, the voltmeter used would
definitely be able to. The only exception is for a short-circuit fault (for ammeter) where the current
would rise by a lot.
11a(iii) R=V/I
= 0.23 / 1.6
= 0.144 (accept 0.14 )

93
Q Suggested Ans e
11a(iv) x Workable circuit to vary current and get p.d. (e.g. use of rheostat oror potentiometer
p teenttiometer or variable
po vari
power supply)
x Correctly drawn circuit symbols and connections with ruler eerr
x Able to measure accurately current (ammeter) in wire and nd p
nd .d
d. (voltmeter)
p.d. (vo
voltme
o eteer)r) across
acr
cross wire

A e tet e eEo eF
Do no a e tt o
ot
not eess onne
onn
nnee ted n a a e


ǀĞƌƌǁŚĞůŵŝŝŶ ŶŐ Ő ŵĂũũŽƌŝƚƚLJŽĨĐĂŶĚŝ ĚŝĚĂ
Ěŝ ĚĂƚĞĞƐ
Ɛ ĚŝŝĚŶ
ŶŽŽƚƵƐĞĂƌŚĞŽƐƚĂƚ͘ƐŝnjĞĂďůĞŶƵŵďĞƌĚŝĚŶŽƚƵƐĞƌƵů
KǀĞƌǁŚĞůŵŝŶŐŵĂũŽƌŝƚLJŽĨĐĂŶĚŝĚĂƚĞƐĚŝĚŶŽƚƵƐĞĂƌŚĞŽƐƚĂƚ͘ƐŝnjĞĂďůĞŶƵŵďĞƌĚŝĚŶŽƚƵƐĞƌƵůĞƌƚŽ
Ěƌ
ƌĂǁ
ǁƐƚƌĂŝŐŚƚůŝůŝůŝŶĞ
ŶĞƐ͘
ĚƌĂǁƐƚƌĂŝŐŚƚůŝŶĞƐ͘ĞƐƐ͘͘
11a(iv)) x Ap ppr
p oxximmaatteelly 2.0 A (s
Approximately samme current
(same curr
cur ent for series connection)
x Sum off p p.d.
.d d. in bboth wires
oth w
ot wi ire
res iin series
n ser ies is about 0.37 V + 1.19 V = approximately 1.5 V
11b(i) x StStrong
tro wind
r ngg win nd causes
c usseess ccooling,
ca oo
o oling, reducing resistance and hence, by V = IR, reducing p.d.
x Conduction
Cond
Co ducction
tion
ti n of
of heat
heat
he a away from wire also leads to lower temperature
11b(ii) x Figg 11
1
11.3
1.3
3 iss no
n
now
ow able to measure p.d. of currents previously not measurable in Fig. 11.2
x Fig. 11.3
11.
1.3 shows
ssh
hows smaller increase in p.d. as current is increased than Fig. 11.2
12a x Positive and negative terminals switches position periodically
x Size (magnitude) of voltage changes sinusoidally

>ĂƌŐĞŵĂũŽƌŝƚLJŽĨĐĂŶĚŝĚĂƚĞƐĐŽŶĨƵƐĞĚĐƵƌƌĞŶƚĂŶĚǀŽůƚĂŐĞĂŶĚŵĞŶƚŝŽŶĞĚĂďŽƵƚƚŚĞĐŚĂŶŐŝŶŐ
ĚŝƌĞĐƚŝŽŶŽĨĐƵƌƌĞŶƚĨůŽǁ͘
12b x Increase the magnetic field strength by concentrating magnetic field lines inside it
x Ensures good magnetic flux linkage between primary and secondary coil

verly broad answers such as increasing efficiency were re ected unless candidates went on to
elaborate further on why efficiency went up.
12c NS / NP = VS / VP
NS = 12 (2200) / 230
= 114.8 = 115 turns
12d(i) x Energy cannot be created nor destroyed. It changes from one form to another.
x Total energy in a closed (isolated) system is fixed (constant)

DĂŶLJĐĂŶĚŝĚĂƚĞƐĚŝĚŶŽƚŵĞŶƚŝŽŶƚŚĞϮŶĚƉŽŝŶƚŽĨƚŽƚĂůĞŶĞƌŐLJŝŶĂĐůŽƐĞĚƐLJƐƚĞŵďĞŝŶŐĐŽŶƐƚĂŶƚ͘
12d(ii) For 100 ideal transformer (zero loss),
Is Vs = Ip Vp
Ip = (1.2 x 12) / 230
= 0.0626 A (accept 0.063 A)

94
Q Suggested Ans e
12d(iii) Efficiency = useful output / total input
= Is Vs ÷ Ip Vp
= ( 1.2 x 12 ) ÷ ( 230 x 0.080 )
= 0.783 (accept 78.3 )

KǀĞƌǁŚĞůŵŝŶŐŵĂũŽƌŝƚLJŽĨĐĂŶĚŝĚĂƚĞƐĐŽŵƉĂƌĞĚƚŚĞƐĞĐŽŶĚĂƌLJĐŽŝůĐƵƌƌĞŶƚƚŽƚŚĞƉƌŝŵĂƌLJĐŽŝů
ĐƵƌƌĞŶƚ͘
E ER E ER
13a Chemical energy Æ gravitational potential energy (initial and final forms)
inetic energy Æ gravitational potential energy (during the run)
13b P=Fd/t
= ( 620 x 5.2 ) / 16
= 202 W
13c(i) Ignoring effects of air resistance,
oss in E = ain in PE
½ m v2 = m g h
V = sqrt (2 x 10 x 1.8)
= 6.00 m/s
13c(ii) Work done by springboard on man = gain in PE by man
F x d = (620 x 1.8)
F = 620 x 1.8 / 0.30
= 3720 N (accept 3700 N)

KǀĞƌǁŚĞůŵŝŶŐŵĂũŽƌŝƚLJŽĨĐĂŶĚŝĚĂƚĞƐĚŝĚŶŽƚŐĞƚƚŚŝƐĐŽƌƌĞĐƚ͘
KǀĞƌǁŚĞůŵŝŶŐŵĂũŽƌŝƚLJŽĨĐĂŶĚŝĚĂƚĞƐĚŝ ĚŝŝĚ
Ě ŶŽ ŽƚŐĞƚƚŚ ŚŝƐŝƐĐŽƌ
Ž ƌĞĐƚ͘
Žƌ
13d(i) x The water molecules vibrat vibrate vertically
te vertic
cal
a lyy up aand
p an d down,
down w ,
x Perpendicular to the propagation
propa gation of wave
p ga vee (energy)
(en
neerrrgy ffrom
ggyy) fr om lleft
e t to right
ef righ
ht
13d(ii) F = number of vibrations in 1 ssecond eco
ond
nd
= 240 / 60
= 4.0 Hz
R R
13a Rate off ffl
flow
low
o o
off el
eelectric
lec
e tric
icc cha
charge
haarrgge
13b

13c(i) I=V/R
= 12 / (3000 + 1000)
= 0.0030 A

DĂŶLJĐĂŶĚŝĚĂƚĞƐůŽƐƚŵĂƌŬƐĂƐƚŚĞLJůĞĨƚƚŚĞŝƌĂŶƐǁĞƌĂƐϬ͘ϬϬϯ͘
ĂŶĚŝĚĂƚĞƐĂƌĞƌĞŵŝŶĚĞĚƚŚĂƚϭƐ͘Ĩ͘ĂŶƐǁĞƌƐĨŽƌĐĂůĐƵůĂƚŝŽŶƐƐƵĐŚĂƐƚŚŝƐŝƐŶŽƚĂůůŽǁĞĚ͘

95
Q Suggested Ans e
13c(ii)
3c(ii) V = IR
= 1000 x 0.0030
= 3.00 V

Many candidates lost marks as they left their answer as 3 V.


ĂŶĚŝĚĂƚĞƐĂƌĞƌĞŵŝŶĚĞĚƚŚĂƚϭƐ͘Ĩ͘ĂŶƐǁĞƌƐĨŽƌĐĂůĐƵůĂƚŝŽŶƐƐƵĐŚĂƐƚŚŝƐŝƐŶŽƚĂůůŽǁĞĚ͘
ĂŶĚŝĚĂƚĞƐĂƌĞƌĞŵŝŶĚĞĚƚŚĂƚϭƐ͘Ĩ͘ĂŶƐǁĞƌƐĨŽƌĐĂ
ĐĂ
ĂůĐƵůĂƚŝŽ
ŽŶ
ŶƐƐ ƐƵĐĐŚ
Ś ĂƐƐƚŚ
ƚŚŝƐƐ ŝƐ
Ɛ ŶŽƚĂůůůŽ
ůŽǁĞ
ǁĞĚ͘
Ě͘͘
Ě
13d
3d x Voltmeter reading will slowly increase.
ோಽವೃ
x ecause ܸ௢ ൌ ൈ ܸ௦ . Wh
W
When
hen
e light intensity
int
n en
nsity decreases,
dec
ecrreeaasses, ܴ௅஽ோ஽ோ increases
஽ோ increeas
ases
es and
es and Voltage
ோೝ೓೐೚ೞ೟ೌ೟ ାோಽವೃ
will increase accordingly to thee formula
fo
orrmu
ula

ĂŶĚŝĚĂƚĞƐǁŚŽĚŝĚŶŽƚƵƐĞƚŚĞ
ĂŶĚŝĚĂƚĞƐǁŚŽĚŝĚŶŽƚƵƐĞƚŚĞƉŽƚĞŶƚŝĂůĚŝǀŝĚĞƌƉƌŝŶĐŝƉůĞŵŽƐƚůLJŐŽƚŝƚǁƌŽŶŐĂƐƚŚĞLJǁƌŽŶŐůLJ
Ś ƉŽƚĞŶƚŝĂ ŝŝĂ
ĂůůĚŝ
ĚŝǀŝŝĚĞ
Ě ƌƉƌ
ƉƌŝŶ
Ɖƌ ŝ ĐŝŝƉů
ƉůĞŵŽ
ŵŽ
ŵŽƐƚ
ŽƐƐƚƚůLJŐŽƚŝƚǁƌ
ǁƌŽŶŐĂƐƚŚĞLJǁƌŽŶŐůLJ
ǁƌ
ĂƐƐƵŵĞĚƚŚĂƚƚŚĞĐƵƌƌĞŶƚǁŽƵ ƵůĚ
ĚƐƚĂLJ
ĂƐƐƵŵĞĚƚŚĂƚƚŚĞĐƵƌƌĞŶƚǁŽƵůĚƐƚĂLJĐŽŶƐƚĂŶƚ͘ LJ ĐĐŽ
LJ ŽŶƐŶƐƚĂƚƚĂ
ĂŶƚ
ĂŶƚƚ͘
13e
3e x Fo g t s tua
tuat
at onss t e d a oss DR R s no s a e
x Fo da s tuat on o
onss t e d a o ss DR
oss DR s n noo s a e as a e
x It will make
maa e the circuit
ciiirrcu
uitit less
lesss sensitive/needs
s ns
sensit
itive/neeed eds more
more
mo re cchanges
h nges in light intensity before voltmeter
ha
reading
read
a in
ad ng ch
change
han
a ggee
x byy inc
increasing
nccreasingg ܴ௥௛௘
n ௥௛
௛ ௢௦ ௦ ௔௧௧ , ܴ௅஽ோ m
௢௦௧
௥௛௘௢௦௧௔௧ usst cch
ust
must hange
change ge even
ge even more before the voltage will make the
co
orrres
e pondingg change.
corresponding c aan
ch n
nge
ggee.

KǀĞƌ
ĞĞƌƌǁŚ
ƌ ŚĞůŵŝŶŐ Ő ŵĂũ
ĂũŽƌŝƚLJ
LJLJŽĨ
ŽĨ ĐĂ
ĂŶ
ŶĚŝŝĚĂ
ĚĂƚĞ
ƚĞƐĚŝĚŶŽƚŬŶŽǁŚŽǁƚŽĂƉƉƌŽĂĐŚƚŚŝƐƋƵĞƐƚŝŽŶ͕ ĨŽĐƵƐŝŶŐƚŚĞŝƌ
KǀĞƌǁŚĞůŵŝŶŐŵĂũŽƌŝƚLJŽĨĐĂŶĚŝĚĂƚĞƐĚŝĚŶŽƚŬŶŽǁŚŽǁƚŽĂƉƉƌŽĂĐŚƚŚŝƐƋƵĞƐƚŝŽŶ͕ĨŽĐƵƐŝŶŐƚŚĞŝƌ
ĂŶƐǁĞƌĞĞƌƌ ŽŶ ƚŚĞĚĞĐƌĞĂ
ĞĞĂ
ĂƐĞĞ ŝŶ ǀǀŽ
Žůƚ
ůƚŵĞƚĞƌƌĞĂĚŝŶŐĨŽƌĂƉĂƌƚŝĐƵůĂƌŝŶƐƚĂŶĐĞŽŶůLJ͘
ĂŶƐǁĞƌŽŶƚŚĞĚĞĐƌĞĂƐĞŝŶǀŽůƚŵĞƚĞƌƌĞĂĚŝŶŐĨŽƌĂƉĂƌƚŝĐƵůĂƌŝŶƐƚĂŶĐĞŽŶůLJ͘

96
97
ec P sics u e is u

40 marks
swer a ues io s i is sec io

1. e i ear is e i e as e is a ce i ra e s i ear. ic o e o owi


is e eares es ima e o i ear i i ame ers m
A B C D

2. e o owi ia ram s ows e rea i o a microme er screw au e.

a is e rea i o e microme er screw au e


A . mm B . mm C . cm D . cm

3. cu e o mass . wi si es . m o as a cu e o si es . m cu rom i s
cor er as s ow . a is e e si o e remai i por io
A m B m
C m D m

4. The overall stopping distance of a car consists of a ‘thinking distance’ or e ri er


o reac and the ‘braking distance’ o s op is car. ri er ri i a m s oo
. s o reac a a ur er . s o s op is car. a is is o era s oppi
is a ce
A m B m C m D m

Pre imi ar Exami a io P sics ec our Express 2

98
5. e o owi rap s ows e spee ime rap o a o .

ic o e o owi rap s s ows e is a ce ime rap o e o

s s

A B

s s

C D

6. a ro i across a ie wi s ow ow a e e ua s op ecause
A i er ia wi cause a o ec s o remai i a s a e o res .
B ere is o e orce ac i o e a .
C ere is a orce a ac s i e irec io opposi e i s mo io .
D e a as o e er si ce ere is o wor o eo e a .

7. car ra e i a a co s a e oci o m s e cou ers a o a resis i e


orce o . ssumi ere are o o er ori o a orces ac i o e car
w ic o ese re a io s ips escri es e ri i orce pro i e ee i e
A B C D

8. e pe e o mass m is ire er ica upwar s a reac es a ei o m.


a is e o a e er a e i es poi
A B . C D

Pre imi ar Exami a io P sics ec our Express 3

99
9. ec icia ries o oose a u wi a spa er u e
u oes o ur . e as e e spa er a o e e
ori o a e e as s ow . ic o e o owi
me o s is o a o ica a emp spa er
A se a o er spa er. ap
Horizontal
B ri e spa er earer e ori o a e e . level
C pp more orce. u
D Pus e u ur er i o e ap.

10. rec a u ar ox o ime sio s . m . m . m wei s . a is e


mi imum pressure i exer s o e sur ace i res s o
A . Pa B . Pa C . Pa D . Pa

11. ic o e o owi s a eme s is rue

A e orce a pis o is .
B Pis o K wi mo e a o er is a ce a pis o .
C e pressure a pis o K a pis o is e same.
D e pressure a pis o K is ower a a pis o .

12. ma ies o a e o ee es. a appe s i e um er o ee es is ou e


orce o ee e orce o e ma Pressure a co ac
A emai s e same ou e emai s e same
B a e emai s e same a e
C emai s e same ou e ou e
D a e emai s e same emai s e same

Pre imi ar Exami a io P sics ec our Express 4

100
13. umi a e smo e par ic es suspe e i air are iewe rou a microscope.
e appear o mo e ra om . ic o e o owi es escri es e
co ersio a ra s er o e er a a es p ace
A Ki e ic e er of air molecules → Kinetic energy of smoke particles
B Po e ia e er o air mo ecu es → Kinetic energy of smoke particles
C Heat energy from source → Kinetic energy of smoke particles
D Light energy from source → Kinetic energy of smoke particles

14. ic o e o owi s a eme s is rue w e e empera ure o a so i is raise


A e mo ecu es expa a e so i occupies a rea er o ume
B e mo ecu es i e so i s ar o mo e arou
C e mass o e so i i creases as e o ume i creases
D ea ra e s o a par s o e so i i e orm o i e ic e er o e mo ecu es

15. as i e process o co e sa io wi
A o i e o or a e i a ea ecause ere is o c a ei empera ure.
B i eo ea ecause i s mo ecu es are osi i e ic e er .
C i eo ea ecause i ermo ecu ar orces are ormi .
D a ei ea i or er o rea e i ermo ecu ar orces.

16. owi across e sur ace o a spoo o o soup wi cause i o coo mai
ecause
A s i air is a poor co uc or u mo i air is oo co uc or.
B co ec io ca o occur wi ou owi .
C owi across e sur ace i creases e sur ace area or ra ia io .
D owi across e sur ace a ows more e apora io o a e p ace.

17. e i erior w ic is ouc i e wa er o a acuum as esi e o o o


wa er is s i ecause
A s i sur aces are poor a sor ers o ra ia io .
B s i sur aces are oo a sor ers o ra ia io .
C s i sur aces are poor emi ers o ra ia io .
D s i sur aces are oo emi ers o ra ia io .

Pre imi ar Exami a io P sics ec our Express 5

101
18. e e co ai i o a oi i i ui is p ace o a a a ce. e
a a ce rea s a er mi u es. a is e a ue o e speci ic a e ea o
apourisa io o e i ui
A . B . C . D .

19. a i orma io ca ou co c u e rom e rap escri i a wa e


isp aceme

is a ce

A e amp i u e is . B e amp i u e is .
C e perio is . D e perio is .

20. ic o e o owi ca e use o ca cu a e e perio o a wa e


A re ue c i i e wa e spee
B re ue c i i e e wa e e
C wa e e mu ip ie e re ue c
D wa e e i i e e wa e spee

21. i e a e cri ica a e o a me ium is w a is e re rac i e i ex


A . B . C . D .

22. o ec is p ace cm rom a e s o oca e cm. ic o e o owi


es escri es e proper o e ima e
A rea i er e imi is e B rea i er e ma i ie
C ir ua upri ma i ie D ir ua upri imi is e

Pre imi ar Exami a io P sics ec our Express 6

102
23. e re rac i e i ex o wa er is . . a is e spee o i i wa er
A . x ms B . x ms C . x ms D . x ms

24. o s ou s o a mou ai a ears e ec o rom e eares ei ouri


mou ai a er . s. i e a e spee o sou i air is ms ow ar is e
ei ouri mou ai rom e o
A m B m C m D m

25. e o owi rap escri es a o i u i a wa e wi e e i e as e posi i e


irec io . ic is a re io o compressio
Displacement / m
B D
A
C

Position / m

26. ic o e o owi me o s ca e use o es w e er a u ow ma eria


is a ma e
I i ou w e er a compass ee e is e ec e w e a wire carr i a
curre is wou arou i.
II i ou w e er a or po e o a perma e ma e wi a rac .
III i ou w e er a ou po e o a perma e ma e wi repe .
A a o B a o C a o D o

27. e ia ram s ows wo c ar es. w ic irec io wi e e ec ric ie ac

Pre imi ar Exami a io P sics ec our Express 7

103
28. e ia ram s ows wo i su a e me a sp eres P a . e s eps are i or er
Step 1: ri e c ar e ro c ose o P
Step 2: ome ari ear e P
Step 3: epara e P rom
Step 4: emo e e c ar e ro
a are e c ar es o P a
p io A B C D
ar e o P Posi i e o c ar e e ai e e ai e
ar e o Posi i e Posi i e o c ar e e ai e

29. circui is se up as s ow . e ce as a em o . a e resis a ce o is


5.0 Ω.

a are e rea i so e amme er a o me er w e e oc e is


a us e o i e mi imum resis a ce a e swi c is c ose
p io A B C D
ea i o . oo s o u sca e e ec io
ea i o . . .

30. ic o e o owi es escri es e c arac eris ics o a ermis or as curre


i creases
p io A B C D
o a e creases creases ecreases ecreases
esis a ce creases ecreases creases ecreases

31. wire as resis a ce . wire a as o wi wice e iame er ma e o e


same ma eria wi a e resis a ce
A B C D

Pre imi ar Exami a io P sics ec our Express 8

104
32. wo i e ica ce s are co ec e o wo circui s. is ou a o circui s a e
e same curre owi i em. a is e a ue o

A 1.5 Ω B 5.0 Ω C .0 Ω D .0 Ω

33. e mos sui a e use ra i or a ea er mar e . is


A B C D

34. e cos o a u i o e ec rici is ce s. a is e cos o e eares


ce o ur o a . compu er or mi u es
A . B . C . D .

35. curre o ows i e i e wire o a soc e w e e app ia ce is u c io i


orma . ic o e o owi is rue
A curre o ows i e eu ra wire
B curre o ows i e ear wire
C curre o ess a ows i e eu ra wire
D curre o ess a ows i e ear wire

36. is co ec e i series wi a ixe resis or


= 5.0 Ω. The input voltage is 6 V and the
ou pu o a e is . ic is e i e p sica
co i io o e surrou i
A ri
B ar
C o
D co

Pre imi ar Exami a io P sics ec our Express 9

105
37. ic o e o owi es escri es w a ma e wi a rac a piece o so iro
A e piece o so iro ecomes a i uce ma e.
B e piece o so iro ecomes a emporar ma e.
C e piece o so iro ecomes a perma e ma e.
D i uce curre wi ow i e piece o so iro .

38. eam o e ec ro experie ces a ma e ic ie rom e op o e o om o e


pa e. orce o e ri o e pa e ac s o e e ec ro eam. a is e
irec io w ic e e ec ro eam ra e s a w ic ru e e ermi es e irec io
p io A B C D
irec io o e o e pa e u o e u o e
pa e pa e pa e
Rule: Fleming’s e i e i
a ue

39. a.c. i pu o is co ec e o e primar coi o a i ea ra s ormer. e


ou pu curre is . ic o e o owi is a possi e com i a io o e i pu
curre a ou pu o a e
p io A B C D
pu urre
u pu o a e

40. ac e era or pro uces a ou pu o a e

o a e as s ow i e ia ram.

ic o e o owi es escri es e
c a es i e e era or is ur e wice
as as . s ime

p io A B C D
u pu o a e ou es a es ou es c a e
Perio ou es ou es a es ou es
----------------------------------------- END OF PAPER -------------------------------------------------

Pre imi ar Exami a io P sics ec our Express 10

106
Section A (50 marks)

Answer all the questions in the space provided.

1. car escri es a i ear mo io represe e e rap s ow i i . . .

v / ms-1

t/s

Fig.1.1

(a) (i) escri e e mo io o e car rom o s.


…………………………………………………………………………………
…………………………………………………………………………………
…………………………………………………………………………………

(ii) a is e a ue o e re ar a io o e car rom o s


…………………………………………………………………………………
…………………………………………………………………………………

(b) i e oa isp aceme ra e e e car or e w o e our e .

(c) ec e isp aceme ime rap or e car’s mo io .


ica e a re e a a ues.

Pre imi ar Exami a io P sics ec our Express

107
2. u i orm ro PQ o e . cm a wei . is p ace o e pi o as s ow
i i . . e ow. spri a a ce is a ac e o e o er e o e ro . oa o
. is p ace . cm rom e spri a a ce.

Fig. 2.1

(a) a is e rea i o e spri a a ce i or er or e ro o a a ce


ori o a

(b) e ermi e e ma iu ea irec io o e reac io orce o e pi o .

(c) e . wei is ra ua mo e a o e ro owar s P e ro ei


ep ori o a s a e a exp ai ec a ei e ma i u e o T.
……………………………………………………………………………………………
……………………………………………………………………………………………
……………………………………………………………………………………………

Pre imi ar Exami a io P sics ec our Express

108
3. . a s ar i rom posi io A w ic is . m a o e e rou s i es ow
rom a i c i e wi a i i ia spee o vo m s as s ow . ric io o e rou i c i e
pro uces . o ea e er . e a ea es e i c i e a posi io B ra e i
er ica upwar a reac es a ei o . m a o e e oor posi io C e ore
a i er ica ow .

A
v0

(a) ae e Pri cip e o o ser a io o E er .


……………………………………………………………………………………………
……………………………………………………………………………………………
……………………………………………………………………………………………

(b) a is are e e er e er ies a e a possesses a posi io A


…………………………………………………………………………………………

(c) a cu a e e ra i a io a po e ia e er a posi io C.

(d) a cu a e e i i ia spee v0 a posi io A.

(e) a e o e assump io or our ca cu a io i (d).


………………………………………………………………………………………

Pre imi ar Exami a io P sics ec our Express

109
4. i . . s ows e p a iew o a is a co ai i o e o is .
e ia ram is raw full scale.
a

wa er
si e B
is

si e A
Fig. 4.1

E e

o ca see wo ima es o e is w e e oo s rom e posi io s ow .


i . . s ows a ra o i rom e is a is re rac e a si e B o e a .
e ra e ers e e e as s ow .

(a) Measure e a e o i ci e ce a re rac io a use ea es o e ermi e


e re rac i e i ex o e wa er i e a .

On Fig.4.1

(b) (i) s e c a seco ray o ee o raw o sca e rom e is o ee e


a is re rac e a si e A o e a

(ii) s ow e posi io s o e two images o e is .

Pre imi ar Exami a io P sics ec our Express

110
5. (a) Exp ai wri i a ou mo ecu es ow e air i si e a car re exer s
a pressure o e wa s o e re.
……………………………………………………………………………………..
……………………………………………………………………………………..
……………………………………………………………………………………..
……………………………………………………………………………………..
(b) esse c ose a pis o co ai e a co s a mass o as. Keepi e
empera ure o e as co s a wei s are p ace o op o e pis o w ic
re uces e o ume o e as.

omp e e e a e e ow usi e wor s increases decreases or no effect


o escri e e c a es a a e occurre .

Property of the gas Change that has occurred


um er o mo ecu es i e er cm

re ue c o co isio s o e as mo ecu es
wi e pis o

era e i e ic e er o e as mo ecu es

Pressure o e as

6. i perspex a is p ace ear a i posi i e c ar e me a ome i a a e


raa e era or. e a swi s awa rom e posi i e c ar e me a ome a
remai s s a io ar a posi io X. i . .

me a ome
X

a er ra
e era or

Fig. 6.1

(a) Exp ai w e perspex a mo es awa rom e me a ome.


…………………………………………………………………………………………
…………………………………………………………………………………………
Pre imi ar Exami a io P sics ec our Express

111
(b) e perspex a as a mass o . .

e i s a w ere e a is s a io ar a X ere is a ori o a e ec ric


orce E . ac i o e ri a e sio T i e s ri a e wei W
i . .

FE 0.15 N

Fig. 6.2

usi a sca e rawi e ermi e e e sio T a ea eT a e


s ri ma es wi e er ica . a e .

Pre imi ar Exami a io P sics ec our Express

112
7. (a) i . . s ows a e ec ric circui powere a . a er o e i i e
i er a resis a ce.

. : . :

E
12.0 V V
. : . :

S
A

Fig. 7.1
e ermi e e amme er a e o me er rea i sw e

(i) e swi c S is ope

(ii) e swi c S is c ose .

Pre imi ar Exami a io P sics ec our Express

113
(b) e same power source is ow co ec e o a po e ia i i er co sis i o a
a a resis or. i . . .

i emi i io e is a input transducer w ose resis a ce ca


c a e accor i o e amou o i a i o i.

10 k :

12.0 V

LDR V

Fig. 7.2

(i) Explain the word ‘input transducer’.


……………………………………………………………………………….
……………………………………………………………………………….

(ii) a is e resis a ce o e w e e o me er rea s


.

Pre imi ar Exami a io P sics ec our Express

114
8. pupi ma es a simp e .c mo or as s ow i i . . usi some commo ma eria s
a co ec e o a . a er .

Fig. 8.1

e e ame e copper wire is a i su a e wire wi part of its insulation remo e .


e e s o e coi are p ace o e ar e paper c ip.

e e power source is ur e o e coi is i e a slight push a e coi e i s


o spin.

(a)(i) is e coi i e a slight push


………………………………………………………………………………………

(ii) Exp ai w e coi s ar s o ro a e co i uous .


…………………………………………………………………………………………
…………………………………………………………………………………………
…………………………………………………………………………………………
…………………………………………………………………………………………

(b) a s ro er power source is use s a e i s e ec o e ro a io o e coi .


………………………………………………………………………………………..

(c) a is e purpose o e ir ma e i si e e cup


………………………………………………………………………………………..

Pre imi ar Exami a io P sics ec our Express

115
9. i . . s ows e s ruc ure o a ra s ormer w ic is use i e ra smissio o
e ec rica power rou e ca es.

oi um er o ur s

Fig. 9.1 Table 1

e i eer is assi e o ui a s ep ow ra s ormer or s eppi ow e o a e


rom . o i e su s a io o a ousi es a e. e as e c oice o usi
our pes o coi s wi i ere um er o ur s as s ow i a e a o e.

(a) ase o a e se ec e mos sui a e pair o coi s or ma i e primar coi


a seco ar coi o e ra s ormer. Exp ai our c oice.
…………….…………………………………………………………………………………..
………………………………………………………………………………………………...
…………………………………………………………………………………………………

(b) ssume a e ra s ormer is e icie a e power ou pu is


e ermi e e curre owi i e primar coi .

(c) aea exp ai one ea ure a ca impro e e e icie c o is ra s ormer.


…………….…………………………………………………………………………………..
………………………………………………………………………………………………...
…………………………………………………………………………………………………

END OF SECTION A-

Pre imi ar Exami a io P sics ec our Express

116
Section B 30 marks

swer a e ues io s rom is sec io . ues io as a c oice o sec io o a swer.

10. Fig. 10.1 s ows e rau ic ra i s s em or a car rom e ra e pe a o e


ra i iscs o e w ee .
Pis o P
Fig 10.1

Pis o Q

orce is app ie ow war s o e ra e pe a i or er o s ow ow e w ee s o


e car.

(a) si Fig. 10.1 exp ai c ear ow a orce app ie o e ra e pe a ca s ow


ow a mo i car.

…………………………………………………………………………………………………....

….…………………………………………………………………………………………………

.……………………………………………………………………………………………………

.……………………………………………………………………………………………………

(b) e sur ace area o pis o P i co ac wi e ra e ui a e mas er


c i er is . x m a e area o pis o Q o e s a e c i er is
. x m.

(i) Exp ai w e area o pis o P as o e sma er a pis o Q.

……………………………………………………………………………………

…….………………………………………………………………………………

Pre imi ar Exami a io P sics ec our Express

117
(ii) i e orce exer e o Pis o Q w e a orce o is exer e o
e ra e pe a .

(iii) pis o P mo es ow cm w e e ra e pe a is epresse


ca cu a e e is a ce mo e pis o Q.

(c) or er o e sure a e ra i s s em u c io s proper air ca o e


rappe i e ra e ui . Exp ai c ear ow rappe air i e ra i ui
ca a ec e per orma ce o e rau ic ra i s s em.

……………………………………………………………………………………………

……………………………………………………………………………………………

(d) e e roa is we a su e ar ra i w e e car is mo i a a i


spee ca cause e w ee s o s op ro a i i s a a e car wi s i s i e
u co ro a .

(i) Exp ai w a as mo i car s i s o e we roa w e e ra e is


su e presse er ar a e w ee s s op ro a i .

……………………………………………………………………………………

..………………………………………………………………………………….

……………………………………………………………………………………

(ii) o re uce e possi i i o a car s i i o a we sur ace e w ee s o


e car a e specia esi e rea s as s ow i Fig.10.2. u es
ow ese rea s are a e o re uce e c a ces o e car s i i o
a we sur ace.

re rea s
Fig. 10.2

………..…………………………………………………………………………………

……………………………………………………………………….………………....

Pre imi ar Exami a io P sics ec our Express

118
11. (a) o pure su s a ce X was ea e u i orm rom i s so i s a e u i i reac es
e aseous s a e. e empera ure o X was a e i i er a s o . mi u es a
are a u a e as s ow i Fig.11.1. ssume a e ea supp ie was co s a
a o ea was os uri e ea i process.

e i poi o pure X . Time Temperature


oi i poi o pure X . / min / C
ime w e X e a me i . mi u es . .
ime a e or a o so i X o me . mi u es . .
ime w e X e a oi i . mi u es . .
peci ic a e ea o apori a io o X . . .
Power . .
. .

Fig. 11.1
empera ure

.
ime mi
.

(i) a a i e a a o ai e a usi e i e i orma io po e ea i


cur e o pure su s a ce X i e ri i es pro i e .

(ii) a cu a e e speci ic ea capaci o e so i X.

Pre imi ar Exami a io P sics ec our Express

119
(b) Fig. 11.2 e ow s ows e coo i cur e rap s o wo pure i ui s Y a Z o e
same mass.
empera ure
T1

Fig. 11.2

ime

(i) ic su s a ce Y or Z as a ower me i poi

……………………………………………………………………………………………

(ii) ic su s a ce Y or Z as a rea er speci ic ea capaci i e i ui


s a e Exp ai our a swer c ear .

……………………………………………………………………………………………

……………………………………………………………………………………………

(iii) ic su s a ce Y or Z as a rea er speci ic a e ea o usio Exp ai


our a swer c ear .

…………………………..………………………………………………………………

……………………………………………………………………………………………………

Pre imi ar Exami a io P sics ec our Express

120
EITHER

12A.(a) Fig. 12.1 s ows a o ec AB ear a i co er i e s. e pri cipa oci o


e e s are a F a F’

Fig. 12.1

(i) mea s o a accura e rawi raw ra s o i e posi io s o e


ima es o e poi s A a B.

(ii) o ec AB is rou c oser a c oser o e co er i e s u i a is a ce


ess a o e oca e escri e c ear e c a es o e ima e o AB.

……………………………………………………………………………………………

………………………………………………………………………………………….

(b) Fig. 12.2 s ows a sca e rawi o a o ec PQ a i s ima e P’Q’ a er


passi rou a i co er i e s. oca i e posi io o e co er i
e s a rawi ra s o e ia ram i e oca e o e co er i
e s.

P’

Q’
Q

Fig. 12.2

Focal length = ………………………..


Pre imi ar Exami a io P sics ec our Express

121
(c) i ra s passi i o a e e a u er o wo re rac io s o ce as e pass rou
e cor ea a a o er as e pass rou e e s o e e e. Fig.12.3 s ows ow
i ra s pass rou ee e a a e ima e o a o ec is orme a e ac o
e e e re i a or a i i i ua wi per ec e esi . or a s or si e perso e
ima e o a is a o ec is orme i ro o e re i a.

Fig. 12.3

(i) e wa o correc s or si e ess is o use a pair o spec ac es. ic


pe o spec ac e e s co er i or i er i wou e sui a e o correc
s or si e ess Exp ai our a swer c ear .

…………………………………………………………………………………………….

….…………………………………………………………………………………………

………………………………………………………………………………………….…

(ii) o er me o o correc s or sightedness is by performing a ‘lasik surgery’


w ic remo es a sma por io o issue i e cor ea o ma e e cor ea ess
rou e . u es ow e ess rou e cor ea i front of the eye’s lens can
ep o correc s or si e ess.

……………………………………………………………………………………………

……………………………………………………………………………………………

……………………………………………………………………………………………

Pre imi ar Exami a io P sics ec our Express

122
OR

12B. (a) Fig. 12.4 s ows a so e oi wi a a er a i curre a.c supp coi e arou a
so iro core. a umi ium ri is p ace rou e so iro a res s o e
solenoid. When the a.c supply is turned on, the ring ‘floats’ above the solenoid as
s ow i Fig. 12.5.

o iro core

umi ium ri

Fig. 12.4
Fig. 12.5

(i) Explain clearly why the aluminium ring ‘floats’ when the a.c supply is ur e o .

…………………………………………………………………………………………....

.……………………………………………………………………………………………

.……………………………………………………………………………………………

.……………………………………………………………………………………………

.……………………………………………………………………………………………

….…………………………………………………………………………………………

(ii) e a.c supp is ow rep ace a .c supp w a wi e o ser e


a er e supp is ur e o

……………………………………………………………………………………………

……………………………………………………………………………………………

Pre imi ar Exami a io P sics ec our Express

123
(iii) e so e oi as a a.c supp u e a umi ium ri is rep ace wi
a ‘C’ s ape ri i s ea as s ow i Fig. 12.6. e e supp is
ur e o e shaped ring does not ‘float’ upwards but continued to
remai a res o e so e oi i s ea . Exp ai e reaso w is
appe s.

Fig. 12.6

…………………………………………………………………………………………....

.……………………………………………………………………………………………

.……………………………………………………………………………………………

…………………………………………………………………………………………….

…………………………………………………………………………………………….

(b) Fig.12.7 s ows a simp e a.c e era or w ic as a re ue c o a


pea o a e .

Fig. 12.7

Pre imi ar Exami a io P sics ec our Express

124
(i) ec e rap o e o a e pro uce a ai s ime or wo comp e e c c es
e ow. a e e posi io o e coi o e a i Fig. 12.7 w e ime s.

(ii) e spee o ro a io is reduced by imes e ori i a spee s e c e


ew rap o e o a e pro uce o e same axis a o e. a e is ew
rap wi (ii).

(iii) Exp ai c ear e i ere ces e wee e rap or b(i) a b(ii).

…………………………………………………………………………………………....

.……………………………………………………………………………………………

.……………………………………………………………………………………………

End of Section B—

Pre imi ar Exami a io P sics ec our Express

125
126
CCHY 2018 Pure Physics Prelim Exam Mark Scheme

Paper 1

D C C D D D A A
B C B A B D B A
C B A C B C B A
C D D B C C A C
B B C D D B A C

Paper 2
Note: 1 mark will be deducted for not expressing numerical value to 2/3 sig. fig on 1
occasion. 2 marks to be deducted for more than 1 occasion.
50% mark for each part of qn will be deducted for missing or wrong “unit”.

Section A
Qn no. Suggested Solution Marks
a i e car ra e e a co s a spee a m s rom o s
e ece era es u i orm o s op rom s o s
a remai s a io ar a res or a ur er s
re erses c a e irec io a acce era es u i orm or m
ii Ei er rap ica me o
e ar a io r ra ie . m
mss
r usi ormu a
a –u – . ms
or r . ms
o mar awar e i wor i is o s ow
o a isp aceme
is a ce mo e uri e irs s iss a ce m
moo e uri
s x x x
m
c s/m

¾ o a constant slope or irs


ow s up o m
¾ a reducing gradient or e ex s o m
¾ a horizontal line graph e wee – sa m
¾ a i creasi a ecreasi curve or s owi e as s
1 mark will be deducted for not stating / labeling the axes.

127
a
Let the spring balance reading (or tension) be T
To balance about (pivot) P, e mome a ou P
o a a ic oc wise mome o a c oc wise mome
x x x
7.0 N

Either
e e reac io orce a e pi o e .
i ce e orce o mo i a a a ce
e ce o a upwar orce o a ow war orce

3.0 N
irec io o is er ica upward

OR using POM and take moment about the spring


ng position
posit
itiion

c a i u e si e o e spri a a ce rea i decreases


de
eccrre
eaas
se
es

e o a c oc wise mome as ecrease ass e c oc oc w


wise
ise
se m
mome
omee
e wei a ou P as ecrease wi e re
re uc io
o i e
perpe icu ar is a ce.
o mai ai e ui i rium e a ic oc wise mome m spri musmus a so
ecrease propor io a e .
s mome orce x perpe icu
ic ar
ar iis
s a ce a e is a ce is
co s a e spri orce mu
mus ec
ecre
ecrease
r asee o co
comp
compe
mpee sa e e re uc io
i e mome
mom
o e .

a o a e er is a wa s co ser ser e remai


rema
re maii u c a e E er ca o
e crea e or es ro e e ca o e co er e rom o e orm o
o er orm s
i e ic a ra
r i a io a po e ia e er
c Ep m . x x
65
ssume
me o e er iis s oss a o a e er is co ser e
EP a rric
ric iio
o o a e er a PE KE
. . . x x .
12.4 m/s

OR
EK a or o e a ai s ric io Ep ai
. . . x x – .
12.4 m/s

e ere is e i i e oss o e er ue o sou ea e er o ase

a si e Pri cip e o e ersi i i o i


By measurement

128
‘i i air ‘r i wa er r

wa er si i si r si si 1.31 or e wee . o .

re rac e ra

si e A

and I2 ar
I1 and are
a e e irs a seco ima es o e is
ow correc re rrac
ac e ra rom si e o e e
ii s ow ccorrec
orre
or recc pposi
osii io s o e ima es I1 and I2
os
a air mooeecu
cu e ess mo i ra om a om ar i co i i wi e
re wa a s a re ou i o .
exer a orcorce
rce
eo e u i area of wall’s surface.
is pro uces a pressure as pressure is orce per u i area

Increase increases no effect increases


are correc
a perspex a as ee c ar e wi i uce posi i e c ar es o i s
sur ace a oms ear o e a e raa e era or
s like charges repel e c ar e a wi e repe e o e
c ar e me a ome.

129
as a wei o x . .

ca e cm o . or ess

FE 0.15 N

W 0.05 N

FE a W are correc s ow o ma iu ea irec io


irec o

orrec triangle s ow or para e o ram


ram s o
ra ow
owi
wi e
resultant o a

orrec a T a ues . T

(Deduct 1 mark each for for not


no
ot expressing
e pressing
ex ng T toto 2 or 3 sf / not
labeling the forces on thethe scale
sca
cale
l drawing/
dra
rawi
wing
ng// not
not indicating
in the
diire
r ction of the force(s)
direction fforce(s
s)

a i ope o me er rea i is 0 as ere


ere is
is o curre .
om i e resis a ce :
.
mme er
m er re
reaa s 1.2
1.2 A
ii c ose com
co i e resis
res
esis
is a ce
ce x :
.
mme e
mme er ow rea s .5 A

p. across
acrooss : . x .
e ce p. across para e e wor –
urre rou :
P. across :
P x .
o me er ow rea s 4.5 V
i e ice a co er s o er orm o e er s o e ec rica e er .

ii si po e ia i i er
s p. D a co s a
e ce :
2.0 k:
:

130
Alternatively
e x
x x
x x
x
x 2.0 k:
:

a i o o ercome i er ia o e coi so a i ca s ar o ur e a e e
co uc i e ame e par o e wire o e i co ac wi paper c ip
o a ow curre o passp i o e coi .
ii e e ec ric curre ows i o e coi ia e paper c ip sa rom
ri o e assume e coi is er ica as s ow i e ia ramm

Explanation of the force set up


i se s up a ma e ic ie a e o om coi w ic
i erac wi e ma e ic ie o e perm
perma
rma
a e ma es
e ow wi a or po e up

e e resu a ie pro uces a orc o


orce
rrc
cce
ep pu
pus
us i e
bottom coil (using Fleming’s LHR)
LHR
HR
R) n
ne
near
e
earar tthe
ar he
h eb
bottom
o to
ot tomm ta
tape
pe
w ic ur s e coi .
is causes e co uc i e ame e copperr w wire
irre o
ro a e.

er a i e w e e coi is s i isp ace


e oo e
si e e curre p pro
ro
o uc
uces
ces a ma a e ic ic po e i e coi
w ic wi cause
se e coio o ur u as i iss repe e
ma e po e. i iss a racc e e coi
coi
o wi o ur a
ou a e o isp
sp ace
e ccoi
oi o e o er si e

Explanation
Expl
Explan
anatio
on for
for continuous
cont
con inuous rotation

o ccurre
urre
ur re o
ows
w i o e coi w e
ws e i su a e par o
e ame
ame e wire
wire is i co ac wi e paper c ip a
e ce
ce o more ma e ic orce.

u i er ia wi co i ue o ro a e e coi u i e
co uc i e ame e copper wire co ec s up e circui
a ai .
is a ai se up a orce pus i e coi i a ai
repea i e c c e a causi e coi o co i ue o
ur .

e ro a io spee wi i crease.
Mention ‘ increased rotation’ – zero mark
c o secure e ma e s s ro i e same posi io o op o e
ase o o er a rac i em pre e ma e a rac i e coi
a o ei.

131
a oi s M. Primar coi s a coi K : secondary coil
ompari e o a es o primar coi o seco ar coi
ep ow ra io

e ce e coi s mus e s ep ow o e same ra io o


ompari e ur ra io
i.e oi oi K
If working is not clearly shown, award maximum 1 mark)
pu power x
si P
6.1 A
c Any one of the following…..
ami a i e iro core wi re uce e power oss ue o ea a
pro uce i uce curre ow as e curre i e core
cor
oree i se .

si ow resis a ce primar a seco ar coi s wi mi iimi


mi e e
amou o ea pro uce i e coi s.

o i crease e ma e ic ux i a e e wee e primar


prriim
maar a
seco ar coi s usi a so ma e ic maa eria
erria
e ia iro
ro core
iro co
c e o i
orre
them up…
Section B
a orce exer e o e ra e pe a acts on the surface
su
urface area off Pi
Pis
Piston
ston P in
contact with the oil i e mas er c i er to create
creat
a e a pressure
pressurre
is pressure in the oil is transmitted to all parts of tthe
he oil

i ce oil is incompressible t is creates


crea
e te
tes a force
forc
ce pressing
pres
pressi
s ng on the disc
pads o e w ee s. Friction between
bet
etwe
ween
en theed
disc
isc
isc pads
pads and the wheels s ows
e car ow .

i i ce e pressure acting in n tthe


he lliquid
iq
qui
uidd is tthe
he s
same
ame throughout
small area at Piston P would
wou
ouldld require
requiire a smaller force exer e to
produce a larg
larger
ger fforce
orce a
or att P
Piston
isto
istonn Q.

ii orce exer e o pis


pis o P x P
x . x . x
1800 N
Pressure
Press
sur
ure
e o piss o P . x Pa i o e a o e wro

iii ssumi o e er oss


P x P x
x
0.4 cm
c i ce air is compressible
Pressure exerted at the master cylinder will not be fully transmitted o
e isc ra es esu i i a greater force required at brake pedal to
obtain the same force on the disc brake or e i ea s s em
Force on Piston Q is smaller
i e as mo i car as high inertia
a wet road there is less friction between the wheels and the road

132
e w ee s su e s ops ur i e forward force is greater than the
resistive force

ii e rea s allows water to pass through e sur ace o e re


is increases the friction between the car and the road surface o
pre e s i i .
a i

correc
mar

eac correc par s ape o e rap wi correc a ee a ues


es or axe
axes
es correc
o a par s mar s
o . mi
. mi o . mi
. mi o mi
correc
mi o mi
mi o mi
mar s
ii ea e er supp ie power x ime
x . x
5k
15
ure
empera ur re c a e –
15 C
ea capaci
apacci o so
cap so i ea e er supp ie
mass x emper
emperae a ur
ureec a e
x
500 /kg C

i u s a cce
e

ii u s a ce Y
e su ec e o e same cooling condition e fall in temperature for
substance Y is slower than substance Z
is i ica es a a higher amount of energy needs to be lost
su s a ce compare o or e same amount of fall in temperature.
iii u s a ce Z
or e same mass same period of time
u s a ce a es a longer time to change state
ica i a higher amount of latent heat needs to be lost
compare o o c a e rom i ui o so i s a e

133
Ei er
a i

orrec pair of rays rom o e re rac e rou e e s a passi


pas
assi
si
e o er s rai rou op ica ce re
orrec pair of rays rom o e re rac e rou e e s a passipassi
e o er s rai rou op ica ce re
orrec smaller straight image raw
Arrowheads drawn or e er i ra a imag
imagege la
llabelled
abe
bell
lled
ed w
with
wi
ith
th A’
A’ and
a d B’
an B
o arrow ea s or i comp e e arrow ea s – mii us
us ma m
mar
ar
ima e is o s rai – mi us mar

ii s e o ec is rou earer o e e s owar s o e oca e is a ce


ima e ecomes magnified u remain inverted and re rreal
al
e e o ec is ess a o e oca e is a cce
e rrom e e s e
ima e ecomes ma i ie Upri
Upright
right a virtual..

P’

a
Q
Q’
Q

orrec i e passi rou op o o ec a ima e o oca e posi io o


e s a orrec i e rom o ec o e s com i e wi i e

oca e e wee . o . cm

c i Diverging e s.
i er i e s wi spread the incoming rays e ore i reac es e e s
e more i er e ra s e eri e e s wi e focused at a further
distance in the eye o o e re i a

ii e ra s e er e ess rou e cor ea i undergoes lesser


refraction/less converging
is causes e esser re rac e ra s o e focused at a further distance in

134
the eye a er passi rou e e s.

a i e e supp is ur e o a changing magnetic field is pro uce


arou e so e oi
e c a i ma e ic ux ma e ic ie i es cu i e a umi ium ring
induces an emf on the ring
Lenz’s law e i uce em o e ri is suc a e magnetic field
induced around the aluminum ring opposes the magnetic field of the
solenoid a pro uce i
Like poles wi exis e wee e a umi ium ri a e so e oi
repe e ri upwar s si ce like poles repel

ii e ri wi move upwards momentarily a su se ue falls back


ba
ack
down and rest o op o e so e oi .
iii e s ape ri does not allow current to pass around the alum aluminium
uminiu
ium
m
continuously.
is does not allow any induced current magnetic force/
force/field
e/fi
fiel
eldd to
o be
produced arou e c s ape ri . e ce e ri wi remai
rem
emai
ai a resres o e
op par o e so e oi .

i Em

. .

. . ime

orrec si e ccur
ur e s a
arr i rrom
om max
orrec max. em mi em a perio

ii orrec si e cur
ur e o e s ar i rom max
orrec ma
max.
x eem
m
mi em
perio . s

iii ro a io imes s ower wou resu i


a Output e.m.f 9.0 V w ic is imes esser a i i ia
Period becomes 0.022 s si ce re ue c ecomes
c slower rotation causes lesser e.m.f to be induced i e e era or
and it the period for each oscillation is longer.

135
136
COMMONWEALTH SECONDARY SCHOOL

PRELIMINARY EXAMINATION 2018

PHYSICS (6091/1)

ame ass

SECONDARY FOUR EXPRESS 17 September 2018


PAPER 1 1 hour
1200 h – 1300 h

READ THESE INSTRUCTIONS FIRST

ri e our ame i ex um er a c ass o e ues io paper a a separa e a swer


s ee s use .

ere are forty ues io s i is paper. swer all ues io s. or eac ues io ere are
our possi e a swers A B C a D. oose e one ou co si er correc a recor our
c oice i soft pencil o e .

Read very carefully the instructions on the OTAS.

INFORMATION FOR CANDIDATES

Eac correc a swer wi score o e mar . mar wi o e e uc e or a wro a swer.

rou wor i s ou e o ei is oo e .

a e e ra i a io a ie s re o Ear g o e .

is paper co sis s o 19 pri e pa es i c u i e co er pa e.


ur o er

137
1 e rap s ows e spee ime rap o a pe u um o osci a i rom a poi .

.
a is e perio o e osci a io o e pe u um o

A s B . s C . s D . s

2 e o owi ia ram s ows e ic ess o me a coi s usi a microme er.

a is e ic ess o o e me a coi

A . mm B . mm C . mm D . mm

3 car ra e s a constant speed rou a e .

ic o e o owi s a eme s a ou e mo io o e car is not correc

A e car is acce era i .


B e e oci o e car is u i orm.
C e isp aceme o e car i creases.
D e is a ce co ere per u i ime e car is co s a .

138
4 s o e is row er ica upwar s a a s ac o i s s ar i poi a s ops.

a i e oci i e upwar irec io as posi i e w ic o e o owi


e oci ime rap s is correc

A B

C
D

5 ome s u e s wa o ca cu a e e e si o pure copper. e measure e


mass a o ume o i ere samp es o pure copper.

ic o e o owi mass o ume rap s s ows eir resu s

A B

C D

139
6 roc e as a mass o . e orce pro uce ee i eo e roc e is
i e irec io a ai s e orce o ra i .

a is e acce era io o e roc e

A ms
B ms
C ms
D ms

7 An aircraft heads north-east at 400 km/h. The wind is blowing towards the
north-west at 100 km/h.

Which vector diagram represents the correct way to obtain the resultant velocity of
the aircraft?

A B

C D

140
8 e ia ram e ow s ows a ca o so ri a a ce o i s e e.

ere is e i e posi io o e ce re o ra i

XA

B X XC

X
D

9 orce F ac s o a s ape o ec pi o e a poi Pa i ere posi io s.

ic o e o owi ia rams s ows e posi io w ere orce F is app ie a


wou resu i e owes ma i u e o mome a ou pi o P

A B

C
D

141
10 orce o is app ie o a ox o mo e i up e ramp as s ow . e ric io
ac i o e ox is .

m m

m
ow muc wor is o e a ai s ric io

A
B
C
D

11 mo e par ic es i a ra spare ox are o ser e usi a microscope. sma


poi o i is see o mo e arou as s ow .

a oes is experime emo s ra e a ou air mo ecu es

A e are i co i uous ra om mo io .
B e ca e see rou a microscope.
C e mo e more uic w e e are ea e .
D e mo e ecause o co isio s wi smo e par ic es.

142
12 sma cor is ixe wi wax o a me a p a e. e ec ric ea er is p ace c ose o
e p a e.

me a p a e
e ec ric
ea er
cor

wax

er some ime e wax me s a e cor rops o .

ow oes ea reac e wax

A co uc io o
B co uc io a co ec io
C ra ia io a co uc io
D ra ia io a co ec io

13 e ia ram s ows a e ec ric ea er ei use o ea a ea er o wa er a a


ea er o oi or se era mi u es. o ea ers are i e ica i si e.

e empera ure o e wa er a e empera ure o e oi i crease co s a . e


rise i empera ure o e oi is muc rea er a a o e wa er.

ic o e o owi exp ai s e o ser a io

A e oi as a i er oi i poi a wa er.
B e oi as a i er ea capaci a wa er.
C e oi as a ower oi i poi a wa er.
D e oi as a ower ea capaci a wa er.

143
14 piece o wire as a e ec rica resis a ce o . : i me i ice a . :i oi i
wa er.

a is e resis a ce a assumi a resis a ce c a es u i orm wi


empera ure

A . :
B . :
C . :
D . :

15 e o owi s eps are use o co s ruc a ca i ra e a ermome er i e e sius


sca e. rra e em i e correc or er.

easure e a ue o e ermome ric proper a s eam poi .


oose a appropria e ermome ric proper .
easure e a ue o e ermome ric proper a ice poi .
i i e e empera ure ra e e wee e wo ixe poi s i o e ua par s.

A
B
C
D

16 ic o e o owi ra ia rams is correc

o ec o ec

A B

e e e e

o ec o ec

C D

e e e e

144
17 e ia ram s ows i ra e i rou a me ium. e i reac es e
ou ar wi a acuum as s ow . e i emer es ra e i ao e sur ace.

a is e re rac i e i ex o e me ium

ୱ୧୬ ଷ଴ι ୱ୧୬ ଺଴ι


A B
ୱ୧୬ ଽ଴ι ୱ୧୬ ଽ଴ι
ୱ୧୬ ଽ଴ι ୱ୧୬ ଽ଴ι
C D
ୱ୧୬ ଷ଴ι ୱ୧୬ ଺଴ι

18 i eam is i ci e i o a semi circu ar ass oc a re rac e ou as s ow .


rap o si a a ai s si b is p o e as s ow .

si a
b
.

si b
. .

a is e cri ica a eo e ass

A B
C D

145
19 e ia ram s ows a sec io o a wa e mo io . e par ic e a posi io x mo es i
e irec io o e arrow s ow .

ic o e o owi par ic es a e a e e posi io s A B C a D is i correc

Direction
Direction of
of wave
wave velocity
velocity
A C

x D

20 e wo rap s s ow e ow re er o e same wa e.

isp aceme m isp aceme m

is a ce m
ime ms

a is e spee o e wa e

A . ms
B . ms
C ms
D ms

146
21 e ia ram s ows wa e ro s mo i rom e o ri as see rom a o e a
ripp e a .

ic o e o owi s ows e correc ep o e wa er i e a as see


rom e cross sec io o e a

A B

C D

22 e ow are ree s a eme s a ou e ec roma e ic ra ia io .

ƒ icrowa es ma cause e io isa io o ce s.

ƒ a io wa es are use i ca cer ra io erap .

ƒ ra io e ra ia io is use i remo e co ro s or e e isio se s.

ow ma o e s a eme s are correc

A
B
C
D

23 a io wa es isi e i a ra s are a par o e e ec roma e ic spec rum.

a is e correc or er o i creasi wa e e

s or es o es

A ra io wa es isi e i ra s
B ra io wa es ra s isi e i
C ra s ra io wa es isi e i
D ra s isi e i ra io wa es

147
24 e ia ram s ows a ou spea er a is pro uci a co i uous sou wa e o
re ue c i air.

ic ia ram es s ows ow e sou causes a mo ecu e a P o mo e uri



s
ଶ଴଴

25 e spee o a sou wa e is re uce a w e i passes rom me ium o


me ium .

ic s a eme e ow escri es ec a ei e sou wa e correc

A e re ue c is re uce a .
B e wa ee is re uce a .
C e re ue c ecomes wice i s i i ia a ue.
D e wa ee ecomes wice i s i i ia a ue.

26 e ia ram wo copper ca s a wi ou er sur ace o i ere ex ure are


i e wi same amou o wa er a room empera ure a ea e ea ers o e
same power

ic o e o owi s a eme s is correc

A a er i oi s as er ecause u sur ace is a oo a sor er o ra ia io .


B a er i oi s as er ecause po is e c rome sur ace is a poor a sor er o
ra ia io .
C a er i oi s as er ecause po is e c rome sur ace is a poor emi er o
ra ia io .
D a er i o ca s a e e same e o ime o oi ecause e ex ure o
ou er sur ace wi o a ec e ra e o e er a sor e e wa er.

148
27 ic o e o owi s a eme s is rue

A me a s are ma e ic ma eria s.
B e ec rica co uc ors are ma e ic ma eria s.
C ma eria s a ca e e ec rica c ar e are ma e ic ma eria s.
D ma eria s a ca a ec e irec io o a compass ee e are ma e ic
ma eria s.

28 i u c ar e co uc i a is mo e owar s e posi i e p a e.

ic ia ram correc s ows e c ar es o e a us a er i as ouc e


e posi i e p a e

29 e e ec romo i e orce o a ce is . .

ic o e o owi s a eme s a ou e ce is correc

A e ce ca supp . o c ar e per seco .


B e ce ca supp . o e ec rica power per seco .
C e ce ca supp . o e er per cou om o c ar e.
D e ce ca supp . o e er per cou om o c ar e per seco .

149
30 a is e curre I
I2 I1

A . m
B m
C m
D m

31 e ia ram s ows a a er a ixe resis or a amme er a a aria e resis or


co ec e i series. o me er is co ec e across e ixe resis or.

e resis a ce o e aria e resis or is re uce .

ic o e o owi correc escri es ec a es i e rea i so e


amme er a e o me er

ammeter voltmeter
A i creases i creases
B i creases ecreases
C ecreases i creases
D ecreases ecreases

150
32 e s eps a e o c ar e a co uc or are s ow e ow.

P ace e co uc or o e c ar e o a i su a i sa .
emo e e c ar e ro .
P ace a posi i e c ar e ro ear e sur ace o e co uc or.
o ec e co uc or o e rou .
emo e e rou wire.

ic o e o owi s ows e correc se ue ce o e e s

A
B
C
D

33 ree i e ica i u s L1 L2 a L3 a a resis or R are co ec e as s ow i


e ia ram.

ow wi e ri ess o amps L1 L2 a L3 c a ew e e swi c S is ope e

151
34 e ia ram e ow s ows e simp e circui o a e ec ric a . ic o e
o owi ac io s wi ow e use

I Poi A ouc es poi C.


II Poi B ouc es poi C.
III Poi A ouc es poi B.

A Io
B I a II o
C II a III o
D I II a III

35 e ia ram s ows a rass ro suppor e o wo copper rai s w ic are


co ec e o a a er . e po e o a ma e is p ace e ea e rai s.

w ic irec io oes e rass ro experie ce a orce

152
3 wo ma e s are roppe er ica rom e same ei .

P as ic
pipe

e ma e a roppe rou e copper pipe oo a o er ime o rop rou


e pipe compare o e o er o e a roppe rou e p as ic pipe.

ic o e o owi is e correc exp a a io

A urre is i uce i e copper pipe.


B opper pipe is a o ma e ic ma eria .
C a e ism is i uce i e copper pipe.
D P as ic is a e er co uc or o e ec rici .

37 e ia ram e ow s ows a e era or ur i i e a i c oc wise irec io .


ax e o ro a io

K
L

ic row is correc

direction of current flow through position of coil when current


the coil at the position shown through the coil is maximum
A oKo o ori o a
B oKo o er ica
C o oKo ori o a
D o oKo er ica

153
38 ar ma e is mo e s ow i o a coi o wire w ic is co ec e o a osci oscope.

e race o e osci oscope is s ow e ow

e ma e is e mo e ac rom e coi a a rea er spee .

ic race s ows is
A B C D

39 e ia ram i us ra es e race o ai e o e scree o a osci oscope


w e a i e si a is app ie o e i pu ermi a s.

i isio s

i isio s
e ime ase is se o . ms i a e o a e se si i i is . i .
ic o e o owi correc represe s e pea o a e a re ue c o e
si a

Pea o a e re ue c
A . .
B .
C . .
D .

154
40 ree i e ica i ame amps a are co ec e o a ra s ormer wi
multiple coils. The resistance of each lamp is 4.5 Ω and each requires a current of
. o i up orma .

a ca e o ser e a ou e ri ess o e ree amps

amp amp amp


A immer a orma orma ri ess ri er a orma
B ri er a orma orma ri ess immer a orma
C o i orma ri ess o i
D o i o i o i

155
COMMONWEALTH SECONDARY SCHOOL

PRELIMINARY EXAMINATION 2018

PHYSICS (6091/2)

ame ass

SECONDARY FOUR EXPRESS 14 September 2018


PAPER 2 1 h and 45 minutes
0800 h – 0945 h

READ THESE INSTRUCTIONS FIRST

ri e our ame i ex um er a c ass o e ues io paper a a separa e a swer


s ee s use .
ri e i ar ue or ac pe .

Section A (50 marks)


swer a ues io s.
ri e our a swers i e spaces pro i e o e ues io paper.

Section B (30 marks)


swer a ree ues io s.
ues io as a c oice o par s o a swer.

INFORMATION FOR CANDIDATES


e um er o mar s is i e i rac e s a e e o eac ues io or par ues io .
a i a es are remi e a a ua i a i e a swers s ou i c u e appropria e u i s.
a i a es are a ise o s ow a eir wor i i a c ear a or er ma er as more
mar s are awar e or sou use o p sics a or correc a swers.

a e e ra i a io a ie s re g o Ear o e .

ee o e exami a io e sure a ou a e su mi e a our wor .

For Examiner’s Use

Paper 2
80

Parent’s/Guardian’s Signature

is paper co sis s o 20 pri e pa es i c u i e co er pa e.


ur o er

156
1 s i er umps rom a i a i u e e icop er as s ow i Fig. 1.1.

Fig. 1.1

(a) Exp ai w e acce era io o es i er

(i) is . ms a e s ar o e ump

…………………………………………………………………………………

…………………………………………………………………………………

(ii) ecreases wi ime.

…………………………………………………………………………………

…………………………………………………………………………………

…………………………………………………………………………………

(b) o e poi uri e i e e acce era io o e s i er was . ms . e


s i er a is e uipme a e a o a mass o .

e ermi e e o a resis i e orce ac i o es i er a a poi .

total resistive force = ………….

157
2 (a) e i e gravitational field strength.

………………………………………………………………………………………….

………………………………………………………………………………………….

(b) s o e is re ease rom res a a u ow ei o e Ear . reac es


e rou a er . s. ssume a air resis a ce is e i i e.

(i) a cu a e e e oci o eso ea . s.

Velocity = …………………

(ii) e space e ow s e c e e oci ime rap o e s o e. ica e


ime . s a i s correspo i e oci o e axes.

(iii) e ermi e e ei rom w ere e s o e is roppe .

Height = ………………….

158
3 ossi ue s wi e e ua ru ou . is as e o scie is s oo i or a er a i e
sources o e er . i a s ream s s ems use e i e ic e er o seawa er o
e era e e ec rica e er uri e i comi a ou oi i es.

Fig. 3.1 e ow s ows a wi ur i e s s em i w ic owi seawa er ur s e ur i e


a es.

Fig. 3.1

e opera i . x o seawa er ra e i a a spee o . ms passes


rou eac ur i e e er seco . Eac ur i e e era es . x o e ec rica
e er .

(a) e i e power.

…………………………………………………………………………………………

(b) e i pu power o eac ur i e is e i e ic e er o e seawa er a ows


rou eac ur i e i o e seco .

a cu a e e i pu power o eac ur i e.

pu power ……………

(c) a cu a e e perce a e e icie c o eac ur i e.

Perce a e e icie c =………….

(d) u es o e a a a e o i a s ream s s ems o er co e io a wi


arms.

…………………………………………………………………………………………

…………………………………………………………………………………………

159
4 Fig. 4.1 s ows a er ar e p a e mirror i c i e a o e ori o a e ea a
pa er o e i cei i o a a .

Fig. 4.1

e mirror is se o a s a imme ia e e ow e ce re C o e pa er . R a S
are wo ra s o i rom C a s ri e e mirror.

e ia ram is raw o sca e.

(a) Fig. 4.1 mar wi a cross x e posi io o I e ima e o C. a e our


cross I.

(b) omp e e e i ra s o s ow ow a sRa S ra e a er e s ri e e


mirror.

160
5 Fig. 5.1 s ows ra s rom a is a o ec reac i a co er i e s wi a oca e
o . cm.

Fig. 5.1

(a) a e w a is mea e p rase “the focal length of a converging lens is


6.0 cm”.

………..……………………………………………………….………………...

………..……………………………………………………….………………...

………..……………………………………………………….………………...

(b) omp e e e ra ia ram i Fig. 5.1 o sca e o s ow ow e co er i


e s orms a ima e.

(c) escri e e e ec i a o e ima e w e a o e e s is ei cu awa .

………..………………………………………..…………………….………………...

………..………………………………………..…………………….………………...

161
6 Fig. 6.1 s ows wo a me a p a es posi io e ori o a o e a o e e o er. e
posi i e ermi a o a i o a e supp u i is co ec e o e o om p a e a e
e a i e ermi a is co ec e o e op p a e.

Fig. 6.1

e i o a e supp is swi c e o .

(a) Fig. 6.1 raw e s ape a e irec io o e e ec ric ie pro uce


e ori o a p a es.

(b) sma oi rop e is p ace e wee e wo me a p a es. e oi rop e


remai s s a io ar i mi air.

(i) ae e c ar e o e oi rop e .

…………………………………………………………………………………

(ii) Explain, using your understanding of Newton’s laws of motion, how the
oi rop e ca remai s a io ar i mi air.

…………………………………………………………………………………

…………………………………………………………………………

…………………………………………………………………………

…………………………………………………………………………

…………………………………………………………………………

162
7 amp is opera e rom a .c supp . e ri ess o e amp is o e arie
co i uous o er a wi e ra e. is is ma e possi e usi a aria e resis or
AB o maximum resis a ce . : wi s i i co ac X. wo circui s or ac ie i
e esire resu s are su es e a s ow i Fig 7.1 a Fig. 7.2.

Fig. 7.1 Fig. 7.2

(a) a cu a e e resis a ce o e amp.

Resistance = …………….

(b) a cu a e e maximum a mi imum curre owi rou e amp i

(i) circui i Fig. 7.1

Maximum current = ……………...

Minimum current = ……………....

(ii) circui i Fig. 7.2.

Maximum current = ……………...

Minimum current = ……………....

163
8 Fig. 8.1 s ows e si e iew o wa er wa es orme i a ripp e a .

is a ce m

. . . . . . .

Fig. 8.1

(a) e ermi e e wa e e o e wa e.

a ee ………….

(b) i e a e perio o e wa e is . s e ermi e e spee o e wa e.

pee o e wa e ……….….

(c) sma piece o s ro oam is oa i a A o e wa e a . m ori o a


rom e ori i . a e a exp ai w a wou e e ori o a is a ce
o e s ro oam rom e ori i a er . s.

………………………………………………………………………………………….

………………………………………………………………………………………….

………………………………………………………………………………………….

………………………………………………………………………………………….

164
9 e ia ram e ow s ows e scree o a ca o e ra osci oscope . e ime
ase is se a . ms mm a e e o e ime ase sweep MN is mm.

M X Y N

mm

(a) a cu a e e ime ura io represe e MN.

Time = ……………………….

(b) ra ar si a se rom a ra ar s a io o a is a aircra is isp a e o e


a Xa e si a recei e ac rom e aircra is isp a e a Y. e
is a ce XY is mm.

i e a e spee o e ra io wa e is . x ms ca cu a e e is a ce o
e aircra rom e ra ar s a io .

Distance = ……………………….

(c) e si a isp a e a Y is wea er a a a X. a e a reaso w is is so.

……………………………………………………………………………………………..

……………………………………………………………………………………………..

……………………………………………………………………………………………..

165
Section B
swer all e ues io s rom is sec io .
swer o o e o e wo a er a i e ues io s i Question 12.

10 our ra s ormers A B C a D are ei i es i a e . or eac ra s ormer e


i pu o a e is c a e a e ou pu o a e measure eac ime. e resu s or
eac ra s ormer are s ow e rap s i Fig. 10.1.

. . . . . .

Fig. 10.1

166
e o e ra s ormers is e use o i up a amp rom a power supp
as s ow i Fig. 10.2.

Fig. 10.2
(a) Exp ai ow a curre i e primar coi pro uces a ou pu o a e i e
seco ar coi .

…………………………………………………………………………………………

…………………………………………………………………………………………

…………………………………………………………………………………………

(b) escri e e purpose o e iro core.

…………………………………………………………………………………………

…………………………………………………………………………………………

(c) se e aao e rap s i Fig. 10.1 o a swer e o owi ues io s.


(i) a e a exp ai w ic ra s ormer A B C or D wou e use o
i e amp o orma ri ess rom a supp as s ow i
Fig. 10.2.
…………………………………………………………………………………

………………………………………………………………………………….

167
(ii) ra s ormer C co ai s ur s o i s primar coi . a cu a e e
um er o ur s o i s seco ar coi .

Number of turns = …………….….


(iii) ra s ormer A as a curre o . i e primar coi . a cu a e
e curre i e seco ar coi .

Current = ………………………….

(iv) a e a exp ai w ic ra s ormer A B C or D is not sui a e o e


use or e ra smissio a is ri u io o e er rom power s a io s
o ra smissio ca es.

…………………………………………………………………………………

…………………………………………………………………………………

…………………………………………………………………………………

168
11 (a) Fig. 11.1 s ows a ma ome er w ic co ai s mercur ei use o measure e
pressure rom a as supp . e e o e u e is co ec e o e as supp
a e o er e is ope .

Fig. 11.1

(i) Exp ai ow e as mo ecu es i e ma ome er crea e a pressure.

……………………………………………………………………………………...

……………………………………………………………………………………...

……………………………………………………………………………………...

(ii) ar a a e o e ri im o e ma ome er i Fig. 11.1 a poi Qi


e u e w ic is a e same pressure as poi P.

(iii) i e a e e si o mercur is m a a e a mosp eric


pressure is mm ca cu a e e pressure o e as supp i Pa.

Pressure ……...........

(iv) a e ow e i ui e e s i e u e wi c a e i a ma ome er u e o a
ar er ore i er cross sec io a area is use . e o a amou o
mercur is co s a .

……………………………………………………………………………………...

……………………………………………………………………………………...

……………………………………………………………………………………...

169
(b) Fig. 11.2 s ows a o er ica ass u e wi o e e immerse i mercur
a e o er co ec e o a acuum pump a A. e ass u e i s i i o
a e ar. i a ope i a B a a air i e ass u e pumpe ou rou
A e mercur rises o a maximum ei o mm a o e e e e o mercur
i e is .

o acuum pump A

ass u e

mm mm

e ar

Y
B

ercur i e is

Fig. 11.2

ic c e pump is ow a ac e o ope i Ba air is ei pumpe i o e


e ar.

a e a exp ai ow e mercur co um i e ass u e wou e a e as


air is pumpe i o e e ar e ic c e pump.

……………………………………………………………………………………..…...

…..……………………………………………………………………………………...

…..……………………………………………………………………………………...

170
12 EITHER

(a) e eri aria wa s o o a mi or sur er o a o . e s eri i es er


i s rume s comprisi o a sca pe a a emos a immersi em i
. o oi i wa er or mi u es. e e uic ra s ers e
i s rume s o a we i su a e ra co ai i o s eri i e wa er a room
empera ure o
w ic u co ers e i s rume s. er a ew mi u es
e i s rume s a wa er reac e same empera ure y o .

e mass o e sca pe is a e mass o e emos a is . o are


ma e rom s ee wi a speci ic ea capaci o K. e speci ic ea
capaci o wa er is K.

(i) e ermi e i erms o y e

1. ea os e sca pe a emos a .

Heat lost = ………………………………

2. ea ai e e s eri i e wa er.

Heat gained = ………………………….

(ii) rom (a)(i) wri e a e ua io re a i e ea exc a e e wee e


sca pe emos a a wa er.

…………………………………………………………………………………

…………………………………………………………………………………

(iii) e ce or o erwise e ermi e y.

y = ………………………

171
(b) e appara us is se up o e ermi e e speci ic a e ea o usio o ice as
s ow i Fig. 12.1.

E ec ro ic a a ce rea i
rus e ice a o

ea er B
ea er 250
swi c e
swi c e on 200
off
175
150 A
100

50

0 20 ime s
E ec ro ic a a ce E ec ro ic a a ce

Setup A Setup B

Fig. 12.1 Fig. 12.2

o se ups use simi ar appara us a ma eria s. e ea er i A is swi c e


o w i e e ea er i B is swi c e o . e a a ce rea i s are recor e a
re u ar ime i er a s a e resu s are p o e a ai s ime as s ow i Fig.
12.2.

(i) a e w a is mea e specific latent heat of fusion o ice.

……………………………………………………………………………….

……………………………………………………………………………….

……………………………………………………………………………….

……………………………………………………………………………….

(ii) e ermi e e mass o ice me e e ea er i e irs s.

Mass of ice = ………....….

172
(iii) e er is ei supp ie a a ra e o s ca cu a e e speci ic
ae ea o usio o ice assumi a a e e er re ease rom
e ea er is a sor e e ice.

Specific latent heat of fusion = ………………….

173
12 OR

(a) o i es i a e a a er o roc u er rou a exp osio is ri ere o e sur ace o


e Ear . Fig. 12.3 e ow s ows e arra eme .

Ear

Fig. 12.3
ou wa e rom e exp osio ma ra e o e e ec or rou air path 1 or
rou Ear path 2 . ca a so e ra smi e i o e a er o roc path 4 wi
par o wa e ei re ec e a e ou ar e wee e Ear a e a er o roc
as i ica e path 3.

e ime a e or e sou o reac e e ec or is s ow i Table 12.4.

Pa 1 2 3
ime a e i seco s or sou o
. . .
ra e rom e source o e e ec or

Table 12.4
(i) Exp ai w sou wa e a es e s or es ime o reac e e ec or a o
path 2.

…………………………………………………………………………….……………

………………………………………………………………………………………….

(ii) i e a e spee o sou i air is ms ca cu a e e is a ce e wee


e source o sou a e e ec or.

is a ce

(iii) se e a swer i par (ii) o ca cu a e e spee o sou i Ear .

pee o sou

174
(iv) e erri o path 4 exp ai ow e spee o sou c a es w e i ra e s
rom Ear o e a er o roc .

……………………………………………………………………………….…………

………………………………………………………………………………………….

(b) Fig. 12.5 s ows u rasou ei use osu a u or a .

Fig. 12.5

(i) Exp ai ow e i ra io s o e source pro uce wa es o u rasou a


su es ow ese wa es are ra smi e rou e o issue o e
recei er.

………………………………………………………………………………………

………………………………………………………………………………………

………………………………………………………………………………………

………………………………………………………………………………………

………………………………………………………………………………………

(ii) rasou use i me ici e as a re ue c w ic is imes i er a


e maximum re ue c a ca e ear uma s.

Es ima e e re ue c a mi e use or u rasou i me ici e.

re ue c

175
176
COMMONWEALTH SECONDAR
SECONDARY
RY S
SCHOOL
CHOOL
EXPR
RESS PHYSICS
SECONDARY FOUR EXPRESS PHYSICCS
PRELIMINARY EXAM
EXAMINATION
MINATION 2018
2018
ANSWERR KEY
KEY

1 D 11 A 21 A 31 A
2 B 12 C 22 A 32 B
3 B 13 D 23 D 33 B
4 D 14 A 24 A 34 C
5 D 15 B 25 B 35 B
6 B 16 B 26 C 36 A
7 A 17 C 27 D 37 C
8 B 18 B 2
28 D 38 C
9 B 19 B 2
29 C 39 B
10 C 20
0 D 30 C 40 C

177
COMMONWEALTH SECONDARY SCHOOL
SECONDARY FOUR EXPRESS PHYSICS
PRELIMINARY EXAMINATION 2018
MARK SCHEME

Section A (50 marks)

o e uc o e mar per ues io or errors i si i ica i ures.

Qn Answers Mark
1ai Air resistance is negligible
r o air resis a ce or resis i e orce
r eo orce ac i is e wei or ra i a io a orce or ra i
r esu a orce is ue o e wei or ra i a io a orce

1aii Air resistance increases as spee i creases

Resultant force decreases


ccep ecreases

1b esu a orce ma
x . ms
675 N
o o i e e irs mar i s u e pu s res
resu
su a orce a
as
s e resis i e
orce.

o a resis i e orce ei – resu


re u a orce
orc
ce

230 N s

r esu a orce ma
x . ms

680 N s

o a resis
re i e o
orc
orce
rcee ei – resu a orce

220
22 0N s

o a mar or 5

2a Gravitational force per unit mass.

178
Qn Answers Mark
2bi a u
10 ms-2 (v – 0 ms-1)/2.0s

20 ms-1

2bii Straight line rap wi positive gradient starting from orig


origin.
gin
in.

orrec a ues o velocity o 20 ms-1 a corresponding time


me o 2.
2.00s
c ear i ica e o rap . Axes labelled with units.

v/ms-1

t/s
.
2biii rea u er e rap x 2.
2.00 s x 20 m/s
m/s

20 m

e ei rom
rom w ic e a is roppe is m.
e.c. . ro
rom i iiii

Examin
Examiners’
Exam iner
e s’
s comments:
com
omments:
e era
era we o e apar or some s u e s w o s i s e c e rap
wi w wro
ro i i ia spee o m s.
ese s u e s ee o u er i e e p rase suc as “released from rest”
i e ues io s em o remi emse es.

o a mar or 7

3a Rate of work done or Rate of energy conversion


x
3b (9.7 X 105 kgs-1) (3.0 ms-1)2

4.4 X 106 /s or W s
3c (1.2 x 106 / 4.37 x 106) x 100
.
28 s
e.c. . rom

r (1.2 x 106 / 4.4 x 106) x 100


.
27 s

179
Qn Answers Mark
3d o e o owi

o ere is curre or wa er mo i rou ou e a u


o so or wi owi i e correc irec io .
o No noise pollution or i a e er .
o Do not require large clearance of land space.
o oes o o s ruc e i pa o birds

o a mar or 6

4a

ma e oca e correctly and


an
nd accu
accurately
c ra
r tely (+/
(+/-
+/- 1 mm
mm)) a a e e as I.

4b e ecc e ra s raw correc


corrre
recc accep
acce
c p ei
ei er
er ex e sio o I or rom
measureme o a e o i ci e cece a e o re ec io o
.

e mar or eac ra .

o a ma
marr or 3

5a e distance
dist
distan
ance
ce be
betw
between
twee
e n the optical centre ce re o co er i e s e s
axxis an
axis andd Pr
P
Principal
inci
incipa
p l Focus oca Poi is 6.0 cm.

5b ra is raw parallel o e o er wo ra s a passing through the


optical centre undeviated.

ra s brought to a focus o e oca p a e. Don’t accep ima e


orme a Pri cipa ocus .

a 6.0 cm . cm away from the lens

180
Qn Answers Mark

5c ma e ecomes less bright or dimmer.

Examiners’ comments:
a s u e s ou a e ima e wwii e a e or or ima
ima e wi
isappear or ecome ur.
u e s ee o ow a w e e a e e s is cu
u a e ra s a
use o e co er e is a e o e e s wi o o er reac e
ima e ra s i ci e o e o er a is o a ece e . i cece ess i ra s
reac e ima e e ima e wi e ess ri .

o a mar or 5

6a ape Vertical straight


ht lines
lin
ines
s are
are raw
raw rom o e p a e o a o er
wi equal spacing e wee
wee em.
em mm i spaci is a owe

irec io
o e arro
arrows
ows poi
poi rom o om p a e o op p a e

6bi Positive.
Posi
siti
tive
v .

6bii e el
elec
electrostatics
ectrros
osta
tati
tics or e ec ric orce o a rac io rom e op p a e a
repu sio
repu s rom e o om p a e is acting upwards o
rom e oi rop

a is equal to the weight o e oi rop w ic ac s ow war s.

s ere is no resultant force ac i o e oi rop e oi rop remai s


a res .

o a mar or 7

7a P
(12 V)2 / 24 W

6.0 Ω
7bi
aximum curre

2.0 A

181
Qn Answers Mark

i imum curre

1.0 A

7bii
aximum curre

2.0 A

i imum curre accep i wor i is o wri e

0A

o a mar or 6

8a λ = 0.550 m – . m
0.400 m

8b V=λ
0.400 m/0.33 s

1.2 ms-1
8c e piece o ro oam wou e 0.
0.500
.500 m orii o a
or rom e ori i .

is iss ecause e wa e iss a tr


transverse
tran
a svere se
s wave
wav
ave or e mo eme o
s ro oam is perpe icu ar o e irec iire
recc io o e wa e

eac par o e wa e only ly oscillates


osc
scilla
ate
tes
s up and down a o e a
ri or e ro oam
o i ra es
es perpe icu ar o e irec io o e
wa e
r wa
a es
es ra
ra s er e er
er wi ou ra s erri ma er e ce i s
ori o a is a ce
ce wou
wou remai co s a .

o a ma
marr or 6

9a mmm x . ms mm
20 ms or 0.020 s or 2.0 x 10-2 s

9b
଴Ǥ଴ଵଶ ଵଶ௠௦
i i e ime e. . ଶ
or ଶ

. x msx . s

1 800 000 m or 1.8 x 106 m

9c E er is absorbed e surrou i air.

us less energy is received e ra ar.

182
Qn Answers Mark

o a mar or 5

10a e current i e primar coi is alternating changing

e magnetic flux in the secondary coil is changing co i uous


e ce i uci a em i e seco ar coi .

b o concentrate the magnetic field lines

ci B ecause e the output


p voltage
g is 12.0 V when the input
p voltage
g is
3.00 V. as s ow o e rap

cii Ns/50 12.0 V/6.00


V .00 V
V/6

s 100

ciii 48.0 V/ 6.00 V 0.500 A/Is

s 0.0625 A s
civ D ecause i is a step-down ra s ormer orr current
cu
curr
rren
en
ntt is
is stepped
ste
tepp
pped
ed u
upp

resu i i a higer power loss in the cables.

o a mar or 10

11ai e as mo ecu es are mo


movi
moving
ing rrandomly
ando
doml
mly y a continuously a i
spee .
ccep constant and random
rando
d m or co
cons
constant
nsta
tant
nt random

e collide with the wal


walls
alls
ls o
off th
the
he c
container/mercury
ontainer/mercury us exer i a
force o e wa
wa s mercur
mercu
curr . e orce per u i area is e pressure.

11aii

183
Qn Answers Mark
11aiii mm – mm 80 mm or Pressure difference is 80 mmHg

P mm mm
mm

P = ρgh
0.840 m x 13600 kgm-3 x 10 Nkg-1

110 000 Pa s or 114 000 Pa s or 1.14 x 105 Pa


11aiv e i ui levels i both sides o e u e wi fall.

owe er e difference i e wo e e s wi remain unchanged.

11b s air is pumpe i o e e ar e pressure of the bell ar increases


increas
asees
a o e a mosp eric pressure.

s suc mercur wou e orce i o e ass u e ma i e


mercury column taller/longer.
taller/longer
r .

o a mar or 10

12 ea os sca pe a emos a
EITHER (0.050 kg + 0.070 kg) x (450 /kgK) x (100oC -y)
y
ai1 . x x . x xy
(5 400- 54 y)
y or 54(100 –y)

ai2 ea ai e e s eri i e wa e er mcθ


mcθ
. x Kx y o
840y - 23 520) or 840(y
840(0(y
y --28)
28))
28

aii Heat lost


los
ostt by scalpel
scalp
pel and
and hemostat
hem
emostat heat gained by water

r 54(100 –
–y)
y) 840(y
840(
840(yy -28)

aiii y
y y
y
y 32 s

bi peci ic a e ea o usio o ice is e amount of thermal energy


required

o c a e unit mass of ice rom i s solid to liquid state wi ou a c a e


i empera ure.

184
Qn Answers Mark
bii ass o ice me e e ea er a o e i e irs s –

25 g

biii ae ea ai e ice ea supp ie ea er


peci ic a e ea o usio P m

400 /s 20s) / 25g

320 /g

o a mar or E E 10

12 OR
ou ra e s faster in solid ear or rou compare o air.
airr.
ai
ai
aii
is a ce 330 ms-1 x 0.100 s
33 m or 33.0 m
aiii
pee 33 m /0.020 s or 33.0 m /0.020 s
1700 ms-1 s or 1650 ms-1 s

aiv
e speed of sound increases. e re rac e wa e bend
bends
ds aw
away from
the normal.
bi Pro uc io o u rasou
Vibration of the source a i re ue c o a ro
r
alongg the direction of the
e pr
propa
prop
opa o o e wave
a a io w ve
wa
or o i u i a wa e pro u uces
ces u rras
rasou
asou
ou .

ra smissio rou o issues


iiss
ssue
u s
e molecu
molecules
c le
es o e bobody
dy tissue
tis
issue vibrate ac war a orwar in the
direction
directio parallel
ion pa
ara
rallel o e ir ec iio o e propa a io o sound or wave.
irec

r
e i ra
ra io s o e source resu s i e orma io o a series o
compressions
co
omp
mpre ess
ssio ns (or
ions o region of high pressure) and rarefactions (or
(or
region
regi
gion
on ooff lo
loww pressure) i e body tissue.

ra smissio o e recei er
e e u rasou i s e e ser par o e issue i is reflected
rou e o issue a is e ec e e receiver (or o ac o
e receiver .

bii 2 MHz or 2 x 106 Hz or 2000 kHz

o a mar or 10

185
186
a i a e ame ass ex o

DUNMAN SECONDARY SCHOOL


Where…… discernment, discipline, daring,
determination & duty become a part of life.

PRELIMINARY EXAMINATION 2018


SEC 4 EXPRESS
PHYSICS (REVISED) 6091 PAPER 1

1 hour 7 August 2018


0800 0900 h Tuesday

i io a a eria s u ip e oice swer ee

READ THESE INSTRUCTIONS FIRST

ri e i so pe ci .
o o use s ap es paper c ips i i er ue or correc io ui .
ri e our ame a i ex um er a c ass o e swer ee i e spaces pro i e u ess
is as ee o e or ou.

ere are forty ues io s o is paper. swer all ues io s. or eac ues io ere are our
possi e a swers A B C a D.
oose e one ou co si er correc a recor our c oice i soft pencil o e separa e
swer ee .

Read the instructions on the Answer Sheet very carefully.

Eac correc a swer wi score o e mar . mar wi o e e uc e or a wro a swer.


rou wor i s ou e o ei is oo e .
e use o a appro e scie i ic ca cu a or is expec e w ere appropria e.

is ues io paper co sis s o 17 pri e pa es i c u i e co er pa e.

187
2

1 e ia ram e ow s ows a pair o er ier ca ipers se o measure a me a pipe. i e a


e pipe as a i er a iame er o . mm w a is e ic ess o e wa o e me a
pipe

A . mm B . mm C . mm D . mm

2 e o owi s a eme s escri e arious mo io s.

. p a e ies ue or or m.
. ouris ra e s m o a our e .
. s ai craw s a mm s i a s rai i e owar s a carro .
. ru er s a era e spee i a race arou a rac is m s.

ic s a eme s escri e ec or ua i ies

A a B a C a D a

3 e e oci ime rap or a mo i o ec is s ow e ow.

spee ms

N P Q

M O

ime s

w ic re io is e acce era io u i orm

A M oN B N oO C O oP D P oQ

Turn over

188
3

4 rap o resu a orce o a mo i car a ai s ime is p o e as s ow e ow.

s
ic o e o owi e oci ime rap s s ows e e oci o e car

A B
ms ms

s s
C D
ms ms

s s

5 orces ac o a o ec . e o ec is i e ui i rium w ic o e o owi


co i io s is are re uire

e orces a e e same ma i u e.
e orces are opposi e i irec io .
e orces are i e same irec io .
e orces are o e same pe.

A a o
B a o
C a o
D a o

Turn over

189
4

6 c c is is ri i a a s ea spee o a e e roa .

ccor i o ew o s ir aw o mo io w a is e e ua a opposi e orce o e


ac war pus o e ac w ee o e roa

A e e sio i e c c e c ai
B e o a air resis a ce a ric io a orce
C e orce exer e e c c is o e pe a s
D e orwar pus o e roa o e ac w ee

7 ome s u e s measure e masses a o umes o i ere si e samp es o a pe o


woo . ic rap s ows eir resu s

A B
mass mass

o ume o ume

C D
mass mass

o ume o ume

8 e ia ram e ow s ows a u i orm me re ru e o wei . suspe e wo spri


a a ces. e e sio s o e spri a a ces are . a . respec i e .

e ermi e X e is a ce e wee e wo spri s.

A . m B . m C . m D . m

Turn over

190
5

9 a a uses paper c ips o a a ce a o ir o is i er as s ow .

o ir

paper c ips

a is e e ec o e paper c ips

A e e p o raise e ce re o ra i a o e is i er.
B e e p o raise e ce re o ra i o is i er.
C e e p o ower e ce re o ra i o e ow is i er.
D e o o a ec e ce re o ra i u i crease e wei .

10 e ia ram e ow s ows a simp e rau ic ac . ic o e o owi c a es wi


e a e ea ier oa s o e i e
a e

pis o

pis o rau ic ui

iame er o iame er o
A ou e remai s e same
B ou e a e
C remai s e same a e
D a e ou e

Turn over

191
6

11 e ia ram s ows a ai ei oc e i o a piece o woo wi a orce o .

mm

mm
woo

mm

a is e pressure o e woo

A Pa
B Pa
C Pa
D Pa

12 ma ome er is use oi ica e e pressure i a s ee esse as s ow i e ia ram


e ow.

i ui
s ee ma ome er
esse

ic op io escri es e pressure i e esse as i ica e e i ui ma ome er

A is ero.
B is e wee ero a a mosp eric pressure.
C is e ua o a mosp eric pressure.
D is rea er a a mosp eric pressure.

Turn over

192
7

13 orce o . exer e o a o ec causes i o acce era e rom a e oci o ms o


msi s.

e ermi e e power o is orce.

A . B . C D

14 co um o as is s ow compresse as s ow i e ia ram.

pis o

as

ic o e o owi rap s correc s ows e re a io e wee e as pressure P a


e e o e as co um

A B
P P

C D
P P

1 1

15 e ou er sur ace o a e ec ric e e is a wa s ep po is e a s i . a is e purpose


o is

A o s op e er oss co uc io
B o e a poor emi er so as o re uce e er oss
C o re uce e er oss erma co uc io
D o re ec a ra ia io i ci e o i s ou er sur ace

Turn over

193
8

16 e ia ram e ow s ows wo ermome ers a e a e same is a ce awa rom


a i e u se ur er. e u o ermome er is wrappe wi a si er oi w i e e
u o ermome er is wrappe wi a ac paper.

si er oi ac paper

ermome er ermome er

u se ur er

ic o e o owi rap s s ow e correc c a e i empera ure o e wo


ermome ers

A B

empera ure empera ure

ime ime

C D

empera ure empera ure

ime ime

17 e ca i ra i a ermocoup e ermome er o e o i s u c io s is immerse i me i


ice w i e e o er is ep i oi i wa er. e curre rea i o e micro amme er is
. e o u c io is immerse i su s a ce a e rea i is w a is
e empera ure o su s a ce

A o B o C o D o

Turn over

194
9

18 e same ua i o erma e er is app ie o our i ere oc s.


e empera ure rise pro uce is s ow o eac oc .
ic oc as e i es ea capaci

A B

empera ure
empera ure rise o o
rise o o

C D

empera ure empera ure


rise o o rise o o

19 e rap e ow re ers o a experime i w ic a i i ia so i specime o i ro e


a sor s ea a a co s a ra e. o i i ro e as a speci ic ea capaci o
. K a me s a K.

ic o e o owi are e speci ic a e ea o usio a speci ic ea capaci o


i ui i ro e respec i e

speci ic a e ea o usio speci ic ea capaci o i ui i ro e


A . x . x K
B . x . x K
C . x . x K
D . x . x K

Turn over

195
10

20 u s a ce K co sis s o par ic es a are c ose o e er a si i pas eac o er


ra om . e a era e spee o e par ic es i su s a ce K is a so ra ua i creasi .

ic o e o owi es escri es e su s a ce

A u s a ce K is a i ui oi i .
B u s a ce K is a i ui ei ea e .
C u s a ce K is a so i me i .
D u s a ce K is a so i ei ea e .

21 perso s a s a poi as s ow i e ia ram e ow.

mirror

ic o e pi s wi e perso e a e o see i e mirror

A pi s a
B pi s a
C pi s a
D pi s a

Turn over

196
11

22 i . s ows a semicircu ar ass s a wi ce re a cri ica a eo o.

a appe s w e a ra o i perpe icu ar o e iame er P is i ci e a

A e i ra emer es as .
B o a i er a re ec io occurs a .
C e i ra emer es a some poi e wee a P.
D o a i er a re ec io occurs a some poi e wee a P.

23 wo para e ra s o i a passes rou a co er i e s as s ow e ow.


ic poi wi ra P pass rou

A C D

24 pe e is roppe i o s i wa er so a e circu ar wa e ro s are see o ra e ou war s


wi a spee o . e wa e e is w a is e ime a e or e is ur a ce a o
reac

A B C D

Turn over

197
12

25 e rap s ows ow e isp aceme o a par ic e i a wa e aries wi ime.

isp aceme cm

ime s

ic o e o owi is correc

A e wa e as a amp i u e o cm a ca e ei er ra s erse or o iu i a.
B e wa e as a amp i u e o cm a ca e ei er ra s erse or o iu i a.
C e wa e as a amp i u e o cm a ca o e ra s erse.
D e wa e as a amp i u e o cm a ca o e ra s erse.

26 e ia ram e ow s ows i ere re io s o e e ec roma e ic spec rum.

ra io isi e amma
wa es i ra s

ic o e s a eme s e ow is rue o e ra ia io ou i re io

A as a i er spee i ass a i air.


B as a ower re ue c a ra io wa es.
C ca e use i a remo e co ro er.
D as a sma er wa e e a amma ra s.

27 rare ra ia io as wa e e s e wee m o . mm.

a is e maximum re ue c o i rare ra ia io i acuum

A .
B .
C .
D .

Turn over

198
13

28 a e orms are s ow o a ca o e ra osci oscope or a ue a a ui ar. e


osci oscope se i s are e same or o wa e orms.

ue ui ar
a is are e simi ari ies e wee e wo sou s

A e ou ess o
B e pi c o
C e ou ess a e pi c
D e a e o simi ari ies

29 e a i e c ar e ro is rou ear co uc or w ic is ear e mome ari as


s ow e ow.

co uc or
a
co uc or

i su a i
sa

er e ro is remo e w a are e resu i c ar es o co uc or a Pa

P
A posi i e posi i e
B posi i e e ai e
C e ai e posi i e
D e ai e e ai e

30 c ar e c ou carr i a c ar e o passes a i s c ar e o e ear rou


i i . e i i as s or . ms.
a is e i i curre

A B . C . D .

Turn over

199
14

31 ic o e o owi are correc

e ec ro wi acce era e w e p ace i a e ec ric ie .


e ec ro wi a wa s mo e owar s a posi i e c ar e o ec i a s rai i e.
e irec io o a e ec ric ie a a poi is e irec io o e orce o a sma
posi i e c ar e p aces a e poi .

A a o
B a o
C a o
D a

32 wire o e m as a resis a ce o . seco wire ma e o e same ma eria


as ou e e resis a ce a a e cross sec io a ra ius.

a is e e o e seco wire

A m B m C m D m

33 e ia ram s ows a circui co ai i i e resis ors co ec e o a a er .

w ic resis or is e curre e sma es

C D

Turn over

200
15

34 e circui s ows a po e ia i i er w ic co sis s o a ixe resis or a a i epe e


resis or. e po e ia i i er is use o swi c o a amp w e i e s ar .

i . .
K

e resis a ce o e ixe resis or is . e po e ia i ere ce across K is .


w e e amp is swi c e o .
a is e resis a ce o e i epe e resis or w e e amp is swi c e o

A . B . C D

35 E ec rica app ia ces a e o a e a power ra i s as is e e ow. ic as e owes


e ec rica resis a ce

app ia ce o a e power
A e e
B air r er
C e e isio
D was i mac i e

36 e ia ram s ows e irec io o e compass ee e w e p ace ear wo ar ma e s.

a is e i e po es a a

po e a po e a
A or ou
B or or
C ou or
D ou ou

Turn over

201
16

37 e i pu ermi a s o a ca o e ra osci oscope are co ec e o a supp o pea a ue


. a o re ue c . e ime ase is se as ms per i isio a e ai a
. per i isio .

ic race is o ai e

A B

C D

38 p i ri commu a ors ca e ou i a .c. mo or.


a is e purpose o e sp i ri commu a or

A a ows e coi o ro a e i e same irec io eepi e irec io o e curre


rou e coi co s a e er a c c e.
B a ows e coi o ro a e i e same irec io re ersi e irec io o e curre
rou e coi e er a c c e.
C a ows e coi o c a e i s irec io o ro a io e er a c c e.
D pro uces a rea er ur i e ec ecomi ma e ica i uce .

39 e ia ram e ow s ows a wire carr i a curre e wee wo perma e ma e s.

S N S N

curre carr i
wire

a wou e e irec io o e orce ac i o e wire

A o e e
B o e ri
C i o e pa e
D ou o e pa e

Turn over

202
17

40 e ra io o um er o ur s i e seco ar coi o a i e primar coi o a ra s ormer


is .

e ermi e e primar o a ei e seco ar o a e is .

A B C D

- - - End of Paper 1 - - -

Turn over

203
Candidate Name: Class: Index No:

DUNMAN SECONDARY SCHOOL


Where…… discernment, discipline, daring,
determination & duty become a part of life.

PRELIMINARY EXAMINATION 2018


SEC 4 EXPRESS
PHYSICS (REVISED) 6091 PAPER 2

1 hour 45 minutes 31 uly 2018


1115 1300h Tuesday

a i a es a swer o e ues io Paper.


o i io a a eria s are re uire .

READ THESE INSTRUCTIONS FIRST

ri e our ame a i ex um er a c ass o a e wor ou a i .


ri e i ar ue or ac pe o is ues io paper.
ou ma use a so pe ci or a ia rams rap s or rou wor i .
o o use s ap es paper c ips i i ers ue or correc io ui .

Section A
swer all ues io s.

Section B
swer all ues io s. ues io as a c oice o par s o a swer.
ri e our a swers i e spaces pro i e o e ues io paper.

a i a es are remi e a a ua i a i e a swers s ou i c u e appropria e u i s.


e use o a appro e scie i ic ca cu a or is expec e w ere appropria e.
a i a es are a ise o s ow a eir wor i i a c ear a or er ma er as more mar s
are awar e or sou use o P sics a or correc a swers.

e e o e exami a io as e a our wor secure o e er.


e um er o mar s is i e i rac e s a e e o eac ues io or par ues io .

is ues io paper co sis s o 20 pri e pa es i c u i e co er pa e.

Turn over
204
2

Section A
swer a e ues io s i is sec io i e spaces pro i e .
e o a mar or is sec io is .

1 (a) omp e e e a e. i e e missi pre ixes s m o s a a ue.

pre ix s m o a ue
mi i m
io

(b) er i e a o e ec or ua i ies i e is e ow.

force energy distance weight acceleration

(c) i . . s ows a ri suppor e wo s ri s a a rom a eam.

. .

Fig. 1.1
(i) e space e ow raw a a e e ia ram o s ow e resu a o e wo
e sio s i e wo s ri s. e ermi e e si e o e resu a orce a e
irec io e wee e resu a orce a e ori o a .

resu a orce ..
irec io ..

Turn over
205
3

(ii) ae e wei o e ri .

wei ..

2 i . s ows a o u i orm p a . m o a wei s . pri a a ces


a are a ac e o e p a a a is a ce o . m rom eac e as s ow i
i . . .

Fig. 2.1

e epa is ori o a spri a a ce recor s .

(a) a cu a e e rea i o spri a a ce .

rea i ..

(b) i . .
(i) i ica e c ear wi a cross X a i e posi io or e ce re o ra i
o e p a . a e i C.G’.

(ii) mar a a e a e orces wi sui a e orce arrows ac i o e


pa .

Turn over
206
4

(c) a i mome a ou spri ca cu a e e is a ce o e ce re o ra i rom


ee o epa .

is a ce ..

(d) Exp ai w is e e sio i spri o co si ere or e ca cu a io i par (c)

.. .
...
...

3 i . s ows a wa er wa e i a ripp e a a t s. e wa e as a spee o . cm s


a X. e wa er wa es crosses a ou ar AB w ere e is a ce e wee cres s c a es
rom . cm o . cm.
A
X

Y
B
Fig. 3.1 o raw o sca e

(a) a cu a e e perio o e wa e e ore i crosses e ou ar AB.

perio o wa e ..

Turn over
207
5

(b) a cu a e e spee o e wa e a poi Y.

spee o wa e ..

(c) e amp i u e o e wa e remai s co s a a . cm.


(i) a cu a e e a era e spee o e par ic e a .

a era e spee o e par ic e a ..

(ii) ae e i s a a eous spee o e par ic e a .

i s a a eous spee o e par ic e a ..

(iii) i . . raw e wa e or ime t . w ere is e perio o e


wa e.

Turn over
208
6

4 (a) i . . s ows a ra o i rom e op o a o ec P passi rou wo ass


prisms.

Fig. 4.1

(i) e spee o i as i ra e s rom P o is . x msa e re rac i e


i ex o e prism ass is . . a cu a e e spee o i i e prism.

spee ..

(ii) a cu a e e cri ica a e.

cri ica a e ..

(iii) Exp ai w e ra re ec s rou a a oes o pass ou o e


prism a .

.
.
. ...

Turn over
209
7

(iv) raw a seco i ra rom o e o ec P o s ow a e ima e


see e e e is upri .

(b) i . . s ows ra o i rom e op o a o ec passi rou a e s.

Fig. 4.2

(i) a e w e er e e s is co er i or i er i .

........

(ii) raw a o er i ra rom e o ec so a e ima e o e o ec ca


e oca e . a e e ima e I.

(iii) oca e e pri cipa ocus o e e s rawi a o er i ra rom


e o ec . a e e pri cipa ocus F.

Turn over
210
8

5 i . . is a rap o curre a ai s po e ia i ere ce p. . or a e o me a wire.

curre
.

p. .
p

Fig. 5.1

(a) e me a wire o e s m s aw. ae m s aw i wor s.

.. .
.. .
...

(b) a cu a e e resis a ce o e me a wire.

resis a ce ..

(c) ew wire is ma e rom e same me a as e ori i a wire. e ew wire is a


e e o e ori i a wire. e iame er o e ew wire is a a o e ori i a
wire.

(i) a cu a e e resis a ce o e ew wire.

resis a ce ..

Turn over
211
9

(ii) i . . raw a i e o s ow ow e curre aries wi p. . or e


ew wire.

6 i . . s ows a iro ri suspe e a rea . ere is a ar ma e c ose o e ri .


e iro ri is a rac e o e ma e .

rea
ar ma e

iro ri

Fig. 6.1

i . . s ows a rass ri suspe e a rea c ose o a ar ma e.

rea

ar ma e

rass ri

Fig. 6.2

(a) Exp ai w e rass ri is not a rac e o e ma e w ie e iro ri is


a rac e o e ma e .

.. .
.. .
.. .
.. .

Turn over
212
10

(b) e e po e o e ar ma e i i . . is mo e uic owar s e rass


ri ere is a i uce curre i e ri a e ri mo es awa rom e ar
ma e.

(i) Exp ai w a curre is i uce i e rass ri .

.
.
.
.

(ii) Exp ai w e rass ri mo es awa rom e ma e.

.
.
.
.

7 i . . represe s e asic s ruc ure o a ra s ormer.

iro core

primar coi seco ar coi

Fig. 7.1
a er a i o a eo is app ie o e primar coi a a o a e is i uce i e
seco ar coi .

(a) (i) escri e w a is mea a alternating o a e.

.
.
.
.

Turn over
213
11

(ii) Exp ai w a iro core is use i s ea o a s ee core.

.
.
.
.

(b) e primar coi as ur s.


a cu a e e sma es um er o comp e e ur i e seco ar coi a wou i e
a i uce o a e o a eas . i e seco ar coi .

um er o ur s ..

(c) e curre i e primar coi is . . e curre i e seco ar coi is .


a e ou pu o a e is . .
a cu a e e e icie c o e ra s ormer.

e icie c ..

End of Section A

Turn over
214
12

Candidate Name: Class: Index No:

Section B
swer all e ues io s rom is sec io .
swer o o eo e wo a er a i e ues io s i Q10.
e o a mar or is sec io is .

8 ea e ar ic e e ow a a swer e ues io s.
ersea ca e s s ems ra spor e ep o e co ersa io s
e ep o e co ersa io s are carrie across e ocea s o e wor as rie pu ses o
i i ca e a co ai air i i re op ic s ra s. ese s ra s are ma e rom
ass co ere aca i a is pro ec e a ou er casi as s ow i i . . .
e spee o i i c a i is rea er a i e core o a i er a re ec io occurs
a a e i is e co i e o e core.

Fig. 8.1
e ra smissio o i is o e icie as i passes ow e i re si ce i is
a sor e impuri ies i e ass. i ra e s rou m o ass e o
o e si a arri es a e o er e . er o is a ces e i si a as o
e oos e a u erwa er repea ers a are powere a e ec ric curre se a o
a me a s ea i si e e ca e. e repea ers co ec e i series wi e same
curre o . rou eac a a po e ia i ere ce o across eac repea er.
a pica m u ersea ca e ere are repea ers. Eac i ome er o e
me a s ea as a resis a ce o . a some o e e er pro i e e supp
is os as erma e er ea i e s ea .

Fig. 8.2
E PP E
Turn over

215
13

(a) aew repea ers are ecessar a o a u ersea ca e.

..... .. .. ..
... .. ..

(b) e i e critical angle.

..... .. .. ..
... .. ..

(c) Exp ai w e ass s ra s a e o e co ere wi a ca i as s ow i


i . . .

..... .. .. ..
..... .. .. ..
..... .. .. ..
... .. ..

(d) e ermi e or a mu ersea ca e


(i) e o a po e ia i ere ce across a e repea ers

o a po e ia i ere ce ..

(ii) e o a po e ia i ere ce across e resis a ce o e me a s ea

o a po e ia i ere ce ..

Turn over
216
14

(iii) e po e ia i ere ce pro i e e power supp

o a po e ia i ere ce ..

(iv) e erma e er ea os rom eac i ome er o e me a s ea i


o e a .

erma e er oss ..

Turn over
217
15

9 (a) uc ear om es as ee co uc e o a remo e is a a o ser e rom


a s ip w ic is m awa . e uc ear om emi s o i a sou w e
exp o e .

(i) i e a e spee o sou a i are ms a . ms


respec i e ca cu a e e ime a e i seco s or e i a sou
o ra e rom e om o e s ip.

ime a e or sou ..
ime a e or i ..

(ii) e ce or o erwise ca cu a e e ime i er a e wee e o ser er


seei a eari e exp osio .

ime i er a ..

(iii) Exp ai a c a es o our a swer o e ime i er a i (ii) i e om


exp o e i e sea a o ser e rom a su mari e. ssume a e
is a ce rom e om a o ser er is s i m.

.
.
.
.

Turn over
218
16

(b) uc ear proo se sor is p ace a m awa rom e om si e. e se sor


e ec s e amou o i i e e iro me a se s e si a o a
osci oscope. e isp a o e osci oscope is as s ow e ow.

e amou o i is measure i ux. i e a e ai sca e is


ux i a ime ase sca e is s i

(i) ca cu a e e spee o i emi e e uc ear om ase o e


isp a i e osci oscope a

spee o i ..

(ii) e ri s a o e s e c e isp a i e ai a ime ase


sca es are a us e o ux i a s i respec i e .

Turn over
219
17

10 EITHER
e ia ram e ow s ows a experime o o ser e ow s eam ca e use o
e era e power.

ur i e

wa er ou pu
e era or

ca o e ra
osci oscope

ea

osci oscope is use o e ermi e e e icie c o e s eam e era or.

(a) e ea source is a e ec ric ea er a re uires o power. e ermi e e


e er co sume e ea er i i is ur e o or ours o suppor e
experime .

e er co sume ..

(b) eam pro uce e wa er is a e o osci a e e ur i e a re o u io s per


seco . i e a e e era or ca pro uce a pea o a e o . per
re o u io e er seco a a pea curre o e ermi e
(i) e pea o a e e e era or ca pro uce i seco wi e ur i e
co ec e . ssume ere is o oss i e er a is s a e.

pea o a e ..

Turn over
220
18

(ii) e maximum power e e era or ca pro uce i seco .

maximum power ..

(c) si e ri pro i e s e c e wa e orm pro uce e e era or as


o ser e rom e osci oscope.

ms

(d) e ermi e e maximum e er i e e era or ca pro uce i ours


mi u es.

maximum e er ..

(e) e ermi e e e icie c o e e era or i .

e icie c ..

Turn over
221
19

OR
s u e i es i a es e e ec s o empera ure o e o ume o a as use a
co um o air sea e i a capi ar u e o cross sec io a area o . mm a s or
co um o mercur as s ow i i . . .

mercur

h
air co um
h

air co um mercur

Fig. 10.1 Fig. 10.2

o e e o e air co um h was . cm e a mosp eric pressure is


. Pa e e o e mercur co um is . cm a e e si o mercur
is m.

(a) a cu a e e pressure o e air co um .

pressure ..

(b) a cu a e e wei o e mercur co um .

wei ..

Turn over
222
20

(c) o e s aw s a es a PV co s a w ere P is e pressure o e as


a V is e o ume o e as. ec e rap o P a ai s or e air
co um i e capi ar u e.

(d) e capi ar u e is o ea e . erms o i e ic eor s a e a exp ai e


c a es o e e o e air co um i a a wi e o ser e .

..... .. .. ..
..... .. .. ..
..... .. .. ..
..... .. .. ..
..... .. .. ..
..... .. .. ..
..... .. .. ..
... .. ..

(e) e capi ar u e is a owe o coo ac o o a p ace ori o a as


s ow i i . . . a e e ew pressure o e air co um .

ew pressure ..

- END OF PAPER -

Turn over
223
224
DUNMAN SECONDARY SCHOOL
PRELIMINARY EXAMS 2018
SEC 4 PHYSICS 6091 ANSWER SCHEME

PAPER 1 40M

Q1 Q2 Q3 Q4 Q5 Q6 Q7 Q8 Q9 Q10
B B B D A D B C C D
Q11 Q12 Q13 Q14 Q15 Q16 Q17 Q18 Q19 Q20
D C C D B D C A B B
Q21 Q22 Q23 Q24 Q25 Q26 Q27 Q28 Q29 Q30
D C A D A C D C C C
Q31 Q32 Q33 Q34 Q35 Q36 Q37 Q38 Q39
9 Q40
B A C B D A D B C B

PAPER 2 SECTION A
1(a)

i a
1(b) force weight and acceleration only all 3 m st e correctly
correctlly nderlined
ly nd
n derline
ned
ne
1(c) correc a e e ia ram usi ip o ai me o or para e o ram r m me o
(i) resu a orce s ou a e ou e arrows
resu a orce . (range 11.0 to 11.8 N)
irec io e ree o e . orce ree o e .
e ree orce or
o rom ori o a

1(c) . a swer s ou e e same


same asas resu
su a orce
orc
o rce i c i
(i)

2(a)
2(b)

tension
tensio
on
tension
C.G

weight of plank

orrec a e i o a . owar s spri


orrec rawi o arrows represe i e orces
orrec a e i o e orces e sio mus e o er a e sio

225
2(c) Pri cip e o ome a i pri o e e pi o
oc wise mome a i c oc wise mome
x . . . xK
K . m
is a ce . . . m
2(d) e i e o ac io o e e sio i spri cu s e pi o e ce
perpendicular distance e wee e i e o ac io o e orce a pi o
is zero. e ce as mome orce x perpe icu ar is a ce mome ue
o e e sio i spri a ou wi e ero.

3(a) spee wa e e perio


. . perio
perio . s
3(b) re ue c spee wa e e
. . .
spee re ue c x wa e e
. x . . ms . cm s
3(c) perio is a so . s.
(i) era e spee x . .
cm s . ms
3(c) . cm s c ec or or s
(ii)
3(c)
(iii)

4(a) . x spee
ee o i i prism
(i) spee o i i prism
pris
pr ism
m . x .
. x ms
4(a) si c
(ii) c si .
.
4(a) Incident angle is 45 which is more than the critical angle 38.7
(iii) Total internal reflection occurs e ce e ra i o pass ou o
prism a
re ec i s u e s ci e i ci e a e as o.

226
4(a)
(iv)
B

45°

45°

orrec ra wi a eas a arrow raw .


4(b) o er i e s
(i)
4(b)
(i)

4(b)
(i)

o arrows e uc m

5(a) urre is irec


ec propor io a o e po e ia i ere ce across e circui
i a me a iic
c co
c uc or
i empera ure p sica co i io s are co s a
5(b)
. usii a a ue o a rom rap
.
5(c) a e e .
(i) a e iame er mea s o e ori i a area . x .

x x
x
x .
.

227
5(c)
(ii)
c rrent
0.
0

0.6

0.4

0.2

0
0 1 2 3 4 5

6(a) rass is o a erroma e ic ma ma eria


e ia
er a w i e iro
iro is.
is.
e ce ar ma e ca o in induce
ndu
duce magnetism
ce magagneetiism i rass. r e ce rass
ca o e ma e ise o ecome eco me a ma
come ma e .
6(b) e e po e o e arr ma e is is mo e uic owar s e ri
ere is a change in magnetic
magnet ic fflux
etic linking
lux li
lu link
nking the ring.
e ce ara
a ra a s aaww a e. e.m.
m is i uce i e ri . s e ri is a
closed loop
loo
oopp a current
curre
ent iis
s he
henc
nce induced.
hence

ere
erre is cu
cu i o e ma e ic ux i a e i e rass ri is c a e
i ma e icc ux i uces a em a a i uce curre i e rass ri
ue o ara ara a s aw o E .
6(c) e .s aw e irec io o e i uce curre i e ri a e ce
e induced magnetic flux will oppose the original change in
magnetic flux ue o e ar ma e approac i .
e ce a induced N-Pole will be created in the brass ring on the side
facing the bar magnet a repe e awa rom e approac i ar
ma e .

7(a) a er a i o a e is a o a e a c a es e irec io o i s polarity


(i) perio ica .
7(a) ro core is use as i is a so ma e a ca easi e ma e ise or
(ii) ema e ise . i e s ee ca o e easi ma e ise or ema e ise .

7(b) s p s p
s . x
. ur s
7(c) Pp . x .
Ps . x . .
E icie c . . x
.

228
PAPER 2 SECTION B

8(a) i ma e a sor e e impuri ies o e ass u er e


sea e . e ce i as o e oos e a e repea ers o sure
i wi arrive at the output.

or
is is o e sure a i si a s will arrive at the output end
i e a e ra smissio o i is o e icie .
8(b) ri ica a e is e i e as e angle of incidence in the optically
denser medium w ic ma es an angle of refraction of 90o i e
op ica less dense medium.
8(c) e c a i ac s as a a er o optically less dense medium as s
compare o e ass core. is is o e sure a total internal al
reflection is able to occur i e ass core.
8(di) i series
o a po e ia i ere ce x
8(dii) o a resis a ce i me a s ea . mx m

o a po e ia i ere ce
. x

8(diii) o a po e ia i ere ce

8(div) erma e er oss


. x . x x x
.
s or s

9(a)(i) ime a e or sou s


ime a e or i . x ms

9(a)(ii) s
9(a)(iii)
9(a)
a (iii)
a) e ime i err a wi
w e s or
o er.
er
spee
e sppee o i i wa er
e is s ower
o a e spee o sou i
wa er is as er e ce e imei e i er a wi e s or er.
im

9(b)(i) ime
me a e i x s
s

pee o i x
. x ms
9(b)(ii) ime ase a i
P
Pea o ux iis wice
i e orii i a
e e ui i rium i e a wice e ori i a a e s ape o e rap

10 Either
(a) E P
x x x

(bi) Pea o a e . x re s .
(bii) aximum power x . .

229
(c)

(d) aximum E P . x
.
(e) E icie c ou pu i pu x
x

10 OR
(a) Pressure o air Pressure o Pressure
Pressur
ureeo mosp
mossp ere
. x x . x Pa
Pa s
(b) P
x
. x

r
ass x . x x
. x

ei
ei . x x . x

(c)

s rai i e rap rom ori i a

correc axes a e (only when graph shape is correct)

(d) e ea e e air mo ecu es gain energy and moves faster.


Rate of collision against the walls of the tube as well as the
mercury column increases.
is wi raise the pressure i e u e o i crease pushing the
mercury column upwards
increasing the length of the air column.
(e) Pressure o air Pressure o mosp ere
. x Pa

230
231
Geylang Methodist School (Secondary)
Preliminary Examination 2018

PHYSICS 6091/01

Paper Sec 4 Express

i io a ma eria s
1 hour

Setter : r ip e ou
24 August 2018

E E E

ri e our ame i ex um er a c ass o a e wor ou a i .


o o use s ap es paper c ips i i ers ue or correc io ui .

Do not open this booklet until you are told to do so.


swer all ues io s. a e our a swers o e pro i e .

ee o e exami a io su mi a e ues io paper separa e .

INFORMATION FOR CANDIDATES

Eac correc a swer wi score o e mar .

rou wor s ou e o ei is oo e .

cce era io ue o ra i g is assume o e m s o Ear a . ms o oo


u ess o erwise speci ie .

is ocume co sis s o 16 pri e pa es


Turn over

232
2 P sics P Pre im E

1 Pressure ca e e ermi e usi e o owi ormu a

P FyA
F mua

w ere P pressure pasca Pa A area m m mass


m
F orce ew o s a acce era io
s2
e u i pasca is e ui a e o
A kg B 2
s C 1 D kg
sm kgm kgs2 m s 2m

2 Fig. 2 shows Jenny’s setup of the following experiment.

me a ca wi mar er pe i si e

paper

woo e ramp

Fig. 2

e re ease e ca a i ro e ree ow e ramp wi ou s ippi . a


wou s e o ser e o e paper ma e e mar er pe
A e o s e ar er apar .
B e o s e c oser o e er.
C e o s e s c oser e ur er apar .
D e o s are e ui is a rom eac o er.

3 ro e X a ro e Y are oi e o e er a s re c e spri . ro e X as
wice e mass o ro e Y. e e ro e s are re ease e acce era io o X is
. m s o e ri .
X
Y

a is e i i ia acce era io o ro e Y o e e
A . ms B . ms C . ms D . ms

233
3 P sics P Pre im E

4 ree o ec s are cu rom e same i s ee o me a . e a e e same


o ume u i ere s apes.

isc s uare s ape

ic o e o owi s a eme s is true


A e a e i ere mass a posi io o . .
B e a e i ere mass u same posi io o . .
C e a e e same mass a posi io o . .
D e a e e same mass u i ere posi io o . .

5 e ia ram e ow s ows wo immisci e i ui s P w ic a e ee poure


i o a ma ome er w ic is expose o e a mosp ere a e op o o arms.

a is e ra io o e e si o i ui o i ui P
A B C D

6 e ia ram e ow s ows a u i orm me re ru er wi a wei o u er e


ac io o a er ica orce o . .

0 cm 100 cm

30.0 N

w a mar mus a u crum e p ace o o e ru er i e ui i rium

A . cm B . cm C . cm D . cm

234
4 P sics P Pre im E

7 ic o e o owi o ec s as e least i er ia
A ro o o wei ra e i o e oo .
B ro o o wei ra e i o Ear .
C s a io ar roc o mass o e oo .
D s a io ar roc o mass o Ear .

8 e ia ram s ows a simp e mercur arome er.

e e a mosp eric pressure i creases w ic is a ce i creases


A B C D

9 a w ic wei s is mo e up a s ope rom o app i a orce o


as s ow i e ia ram e ow.

3 3N
5
4

ssumi o e er is os o e surrou i w a is e ai i i e ic e er
ai i ra i a io a po e ia e er a wor o e o e a
Kinetic Energy Gravitational Potential Energy Work Done
A
B
C
D

10 power u a uses a a er . e a e era es o ea e er seco


as i s mo or ur s.

a is e e icie c o is a
A . B . C . D

235
5 P sics P Pre im E

11 ic i es e s a es o ma er i w ic mo ecu es a a i e empera ure a e


e sma es spaci e wee em a mo e a e owes spee
Smallest spacing between molecules Molecules have the lowest speed
A oi oi
B i ui oi
C oi i ui
D i ui i ui

12 ic o ese examp es as e wro pe o ea ra s er associa e wi i


Example Type of heat transfer
A ea rom ame o i pa o uc io
B u i warmi e i erior o a car o ec io
C oo i a cup o co ee wi a s ee spoo o uc io
D ea rom ur i o s o perso esi e ire p ace o ec io

e rap e ow s ows e c a es i empera ure o a so i w e i is ea e


a ea er wi a ra i o . se i o a swer ues io s o .

13 ic o e o owi s a eme s s ow a i ere ce e wee ea a


empera ure.
A e o ea is supp ie e empera ure reac e q .
B ea as a u i i Ke i w i e empera ure u i is i q
C ea i creases w e empera ure i creases.
D ea remai s co s a re ar ess w e er empera ure c a es.

14 ssumi ere is e i i e e er oss o e surrou i s w a is e speci ic


ea capaci o e i ui
A q B q C q D q

236
6 P sics P Pre im E

15 ssumi ere is e i i e e er oss o e surrou i s w a is e speci ic


ae ea o usio o e so i
A B C D

16 e rap s ows e er ica isp aceme o a o ec oa i o wa er as a wa e


passes rou e wa er.

a is e re ue c o e wa e

A B C D

17 p a e mirror . m i e is u o a er ica wa wi i s o om . m
a o e rou . o wi is e es . m a o e rou oo s i o e mirror o
see e ima e o imse .

a e o imse e ow is e es ca e see i e mirror

A . m B . m C . m D . m

18 ra o i e ers a prism ma e o ma eria X a ra e s a o e pa as


s ow i e i ure e ow.

a is e re rac i e i ex o e ma eria X
A . B . C . D .

237
7 P sics P Pre im E

19 ree o ec s P Q a R are iewe rou a p a e mirror as s ow .


o s ac e mo es owar s e mirror as i ica e e arrow.

x x

ic ima e wi isappear irs a w ic ima e wi isappear as


isappears irs isappears as
A P Q
B P R
C Q R
D R Q

20 i ra passes rou ree me ia o re rac i e i ices η η a η


respec i e .

T
T

me ium me ium me ium


η η η

i e a T T T w ic o e o owi is correc

A η >η >η B η >η >η C η >η >η D η >η >η

238
8 P sics P Pre im E

21 ree ra s o i a o a co er i e s as s ow .

ic ia ram s ows e pa o e ra s a er passi rou e e s


A B

C D

22 s u e arra es a i umi a e o ec a e s a a scree suc a e si e o


e ima e is wice a o e o ec . Keepi e is a ce e wee e scree a
e i umi a e o ec ixe e exc a es e posi io o e scree a e
i umi a e o ec .

a wou e o ser e o e scree


A urre ma i ie ima e.
B urre imi is e ima e.
C s arp ima e wice e si e o e o ec .
D s arp ima e a e si e o e o ec .

239
9 P sics P Pre im E

23 ic o e o owi rap s a ou e ec roma e ic wa es ra e i i acuum


is are correc

A o B a o C a o D a

24 e ia ram s ows e mai sec io s o e e ec roma e ic spec rum i or er o


i creasi re ue c . ome o e sec io s are a e e . e sec io as a
re ue c us e ow a o i .

ic app ica io uses e sec io


A eri isa io
B a e i e e e isio
C rea oas er
D aser poi er

25 e se up s ow i e ia ram e ow co sis s o a osci oscope a a spea er


a emi s a sou wa e wi re ue c . microp o e p ace . m
rom e spea er e ec s e sou emi e e spea er. wa e orm is
o ser e o e osci oscope represe i e sou emi e .

osci oscope

microp o e
spea er

e spea er is e mo e o a is a ce . m owar s e microp o e a a ew


wa e orm is o ser e o e osci oscope. ompare o e ear ier wa e orm is
ew wa e orm as a rea er .

A spee B amp i u e C re ue c D wa e e

240
10 P sics P Pre im E

26 Fig. 26.1 is a full-scale ia ram a s ows air par ic es a eir e ui i rium


posi io s represe e o e i es . e a sou wa e passes par ic es are
isp ace i o ew posi io s s ow i Fig. 26.2. Q represe s a par ic e.

Fig. 26.1

Fig. 26.2

ic o e o owi s a eme s is a se
A Poi Q is e ce re o a rare ac io .
B Par ic es ex o Q are mo i i opposi e irec io .
C e amp i u e o e wa e is . cm.
D e wa e e o e wa e is . cm

27 ic o e o owi ia rams correc s ows e u i orm e ec ric ie e wee


wo c ar e para e me a p a es

241
11 P sics P Pre im E

28 wo u c ar e me a sp eres P a Q are p ace o e er wi a ic car oar


i ser e e wee em. o sp eres are suppor e i su a i s a s a Q is
ear e wi a wire.

e ai e c ar e ro is rou ear e sp eres as s ow .


a wou e e is ri u io o c ar es o sp eres P a Qw e e wire is
remo e o owe e c ar e ro
P
A e ai e posi i e
B e ai e eu ra
C eu ra posi i e
D eu ra eu ra

Refer to the following information for Questions 29 and 30.


e rap e ow s ows ow e curre i a amp a a resis or aries wi e
po e ia i ere ce app ie .

e amp a e resis or are co ec e i series as s ow e ow a e amme er


rea i is . .

242
12 P sics P Pre im E

29 e ermi e e e.m. . o e circui .

A . B . C . D .

30 e ermi e e resis a ce o e resis or.

A . : B . : C . : D . :

31 e ia ram s ows a experime a se up o a e ec ric circui o e ermi e e


resis a ce o a u .

ic o e o e o owi s ows e correc circui ia ram

243
13 P sics P Pre im E

32 e circui s ow R1 a R2 are i e ica resis ors.

ic o e o owi c a es o e e ec rica compo e s X a Y wi ecrease


e rea i o e amme er e rea es amou
A mmerse X i a ea er o ice wa er a ecrease e i i e si o Y.
B mmerse X i a ea er o ice wa er a i crease e i i e si o Y.
C mmerse X i a ea er o o wa er a ecrease e i i e si o Y.
D mmerse X i a ea er o o wa er a i crease e i i e si o Y.

33 a pi p u o a acuum c ea er e use is missi .


ic o e o owi s a eme is correc
A e acuum c ea er ca e ur e o u i ee e o a e ec rica au e
acuum c ea er wi e i e.
B e acuum c ea er ca e ur e o u i e e e o a e ec rica au e
acuum c ea er wi e sa e o a e ecause e curre wi ow o ear .
C e acuum c ea er o ce ur e o wi s u ow a o ce.
D e acuum c ea er ca o e ur e o .

34 ree i e ica amps a are co ec e i a circui as s ow e ow.

● ●

a wi appe o e ri ess o e amps i e resis a ce o e r eos a is


i crease

A ri er immer immer
B ri er ri er immer
C immer ri er immer
D immer ri er ri er

244
14 P sics P Pre im E

35 su e carries ou our es s wi a ma e.

ic resu s ow is not correc


arra eme esu

A ma e ma e rac s

B ma e iro ar a rac s

C ma e iro ar repe

D ma e copper ar o e ec

36 rass ro is arra e i a or sou irec io a po i compasses are p ace


a eac o i e s.

ic ia ram s ows e posi io s o e ee es o epo i compasses

245
15 P sics P Pre im E

37 Pa represe s wo para e s rai wires carr i curre s. P a exer


orce o eac o er. ic arrow s ows e orce o

38 ree wires P Q a R are eac carr i a curre o i e irec io as s ow


i e ia ram e ow.

a are e irec io s o orces ac i o e ree wires


Wire P Wire Q Wire R
A e e e
B ri e ri
C e ri e
D ri ri ri

39 e i ure e ow s ows a i ea ra s ormer wi e seco ar coi co ec e o


a : oa .

a is e curre i e primar coi


A . B . C . D .

246
16 P sics P Pre im E

40 e ia rams s ow a simp e a.c. e era or a e rap i us ra i e wa e orm o e


e.m. . ou pu o e a.c. e era or.

ic o e o owi ia rams correc s ows e p a e o e arma ure coi o e


e era or as iewe a o e ax e rom e posi io o e s ip ri s a ime i er a s
e oe t1 t2 a t3 o e rap

END OF PAPER 1

247
Geylang Methodist School (Secondary)
Preliminary Examination 2018
a i ae
ame

ass ex um er

PHYSICS 6091/02
Paper P sics Sec 4 Express

i io a ma eria s ri i Papers 1 hour 45 minutes

Setter : r ip e ou 24 August 2018

READ THESE INSTRUCTIONS FIRST


ri e our ame i ex um er a c ass o a e wor ou a i .
ri e i ar ue or ac pe o o si es o e paper.
o o use s ap es paper c ips i i ers ue or correc io ui .
swer all ues io s.
ri e our a swers o Section A i e spaces pro i e i e ues io Paper.
ri e our a swers o Section B i wri i papers pro i e .
Question 13 has a choice of parts to answer.
a i a es are remi e a a ua i a i e a swers s ou i c u e appropria e u i s.
ou are a ise o s ow a our wor i i a c ear or er ma er.

e e o e exami a io as e a our wor secure o e er.


e um er o mar s is i e i rac e s a e e o eac ues io or par ues io .
cce era io ue o ra i g is assume o e m s u ess o erwise speci ie .

For Examiner’s Use

Section A

Section B

Total

is ocume co sis s o 19 pri e pa es a a pa e.

Turn over

248
2 P sics P Pre im E
Section A

swer all e ues io s i is sec io i e spaces pro i e .

1 i . . s ows a ir ua ima e I orme a co er i e s rom a o ec o


ei . cm.

Fig. 1.1

(a) i . . a o e raw ra s o e ermi e

(i) e posi io o e o ec a e e o ec O.

(ii) e oca e o e e s.

oca e ……………………

249
3 P sics P Pre im E
(b) i . . s ows a i ra ra e i i e co er i e s o re rac i e i ex . .
e ia ram is o raw o sca e.

Fig. 1.2

a cu a e e cri ica a e a exp ai e e a iour o e i a er i is


i ci e o e sur ace P.

………………………………………………………………………………………

………………………………………………………………………………………

(c) a e a use o a compo e o e e ec roma e ic spec rum wi e sma es


wa e e a escri e e e ec o a sor i is e ec roma e ic wa e.

………………………………………………………………………………………

………………………………………………………………………………………

250
4 P sics P Pre im E
2 e spee o a u rasou i air is m s.
(a) omp e e i . . o s ow ow ar e u rasou wa e as ra e e
a seco s a er e u rasou was pro uce .

Fig. 2.1
(b) i . . raw e rap o is a ce ra e e a ai s ime or e
u rasou wa e.

Fig. 2.2
(c) aea escri e w ic pe o wa e is u rasou .
………………………………………………………………………………………

………………………………………………………………………………………

(d) Exp ai w a is wa e e a s a e e maximum wa e e o e


u rasou .
………………………………………………………………………………………

………………………………………………………………………………………

251
5 P sics P Pre im E

3 Experime o e wo oc s o mass a were a ac e a pu e


across a rou rou i e irec io s ow i Fig. 2.1.

oc ri oc Pu i
orce

Fig. 2.1

e experime is e repea e o e same sur ace u wi a i ere pu i


orce. e c a es i e oci ies o e oc s or o experime s were measure
a a u a e i Fig. 2.2.

Velocity / cms-1
Time/s Experiment One Experiment Two

Fig. 2.2

(a) si e a ues i Experime e as s ow i Fig. 2.2 ca cu a e e


(i) acce era io o e oc s.

acce era io ……………………

(ii) e sio i e s ri i e a e ric io e wee e rou a oc


is . .

e sio ……………………
(iii) ma iu eo e pu i orce .

pu i orce ……………………

252
6 P sics P Pre im E
(b) erms o orces exp ai w e e oci o e oc s i Experime e
c a es o er ime u remai s u c a e i Experime wo.

…………………………………………………………………………………………

…………………………………………………………………………………………

…………………………………………………………………………………………

…………………………………………………………………………………………

4 i . . s ows a rec a u ar oc o woo o a a rou ori o a oar .

Fig. 4.1

e oc is pus e a e op as s ow i i . . a i i s o e ri .
(a) e ro ace o e oc raw e i e a wi e er ica a ei sa
e ore e oc opp es o er.

(b) se e pro rac or s ow o i . . o measure ea e rou w ic e


oc i s e ore i opp es o er.
a e ……………………

253
7 P sics P Pre im E
(c) e oc is pu ac o e oar as i i . . . is ime i s ea o e
oc ei pus e e e a e eo e oar is raise .

ae e a e a e oar ma es wi e ori o a a e i sa e
oc opp es o er.
a e ……………………

(d) Exp ai i erms o . . a s a i i ow our a swer o (c) mi i er i


e proce ure is repea e a er e ei o e oc is re uce .

............................................................................................................................

............................................................................................................................

5 (a) e pri cip e o co ser a io o e er s a es a e er ca ei er e


crea e or es ro e . a e does appe o e e er supp ie o a
e ice suc as a mo or or a e e isio

..........................................................................................................................

..........................................................................................................................

(b) e e e isio i i . . is swi c e o o wa c a pro ramme. uri is


ime o e ec rica e er is supp ie .

Fig. 5.1

254
8 P sics P Pre im E
(i) rom e i orma io o i . . i e o a e er pro i e or e iewer
o see a ear e e e isio uri is pro ramme.

e er ……………………
(ii) u es w a appe s o e res o e e er supp ie .

..........................................................................................................................

..........................................................................................................................

(iii) a cu a e ow muc e er is i o e i (b)(ii).

e er ……………………

(iv) a cu a e a comme o e e icie c o e e e isio .

..........................................................................................................................

..........................................................................................................................

6 i . . s ows a e apora i e coo er w ic is a e ice a coo s air rou e


co i uous e apora io o wa er. ir is coo e ei ow pas a coo i pa
co ai i wa er. e coo i pa is we a pump w ic pumps wa er up o e op o
e pa rom w ere i ric es ow .

Fig. 6.1

255
9 P sics P Pre im E
(a) erms o i e ic mo e o ma er exp ai ow co i uous e apora io o
e wa er causes e air owi pas e coo i pa o e coo e .

………………………………………………………………………………………

………………………………………………………………………………………

………………………………………………………………………………………

………………………………………………………………………………………

………………………………………………………………………………………

………………………………………………………………………………………

………………………………………………………………………………………

(b) Exp ai w e er suc e apora i e coo i s s em is more e ec i e i r


or umi a mosp eres.

………………………………………………………………………………………

………………………………………………………………………………………
(c) u es o e mo i ica io a ca e ma e o e esi o e e apora i e
coo er suc a e ra e o coo i i creases.

………………………………………………………………………………………

………………………………………………………………………………………

256
10 P sics P Pre im E
7 wo me a saucepa s co ai e same mass o o wa er a e same i i ia
empera ure. Pa is w i e a pa is ac u o erwise e wo saucepa s are
i e ica . o saucepa s are u co ere a coo u er e same co i io s. e
coo i cur es or e wo saucepa s are s ow i i . . .

Fig. 7.1
(a) Exp ai w pa coo s as er a pa .

.............................................................................................................................

.............................................................................................................................

(b) escri e a exp ai ow i . . is i ere w e e pa s are co ere a


e experime is repea e .

.............................................................................................................................

.............................................................................................................................

.............................................................................................................................

(c) Exp ai w a is mea e speci ic ea capaci o wa er is .

.............................................................................................................................

.............................................................................................................................

257
11 P sics P Pre im E
(d) e speci ic ea capaci o wa er is er i . u es o e isa a a eo
is w e wa er is use or coo i .

.............................................................................................................................

.............................................................................................................................
(e) e wa er i pa coo s or mi u es as s ow i i . . . uri is ime
e wa er oses a a era e o o erma e er per mi u e.
(i) a cu a e e mass o wa er i pa .

mass ……………………

(ii) e mass o wa er i pa is e same as a i pa .


a cu a e e erma e er os rom e wa er i pa uri e
mi u es.

oss o erma e er ……………………

258
12 P sics P Pre im E
8 eac er emo s ra es ma e ic scree i . e a ma e is p ace ear a sma
car oar ox paper c ips o e o er si e o e ox are pic e up as s ow i i .
. . e a sma piece o so iro is p ace i si e e ox as s ow i i . . e
paper c ips a o . a e ic ie i es i eac ia ram are s ow as i i es.

Fig. 8.1 Fig. 8.2


(a) i . . mar a arrow o eac o e ma e ic ie i es o s ow i s
irec io .
(b) Exp ai w p aci e so iro i e ox causes e paper c ips o a o .

.............................................................................................................................

.............................................................................................................................

.............................................................................................................................

259
13 P sics P Pre im E
9 e appara us i i . . is ca e a orce o co uc or a a ce. e ere is a
e ec ric curre I as s ow i ere is a orce o . is orce is measure
pu i wei s i e pa u i is rou ac o i s ori i a posi io .

Fig. 9.1
(a) a e w a wou appe i e curre irec io were rom o .

…………………………………………………………………………………………

(b) s u e uses e a a ce o e ermi e e orce F o or i ere


curre s I. e co c u e a F is irec propor io a o I ase o is resu s
i e e ow.

ow c ear wor i s o es ima e e orce o w e e curre is . .

orce ……………………

END OF SECTION A

260
14 P sics P Pre im E
Section B
swer all e ues io s rom is sec io i wri i papers pro i e .
swer o o e o e wo a er a i e ues io s i Question 13.

11 measuri c i er co ai s wa er w ic as a e si o m.

Fig. 11.1 Fig. 11.2 Fig. 11.3

(a) i . . i dicates four ways the observer’s eye could look when taking the
rea i rom e measuri c i er. Pu a circ e arou e e e posi io a
i es e correc rea i a s a e e o ume o wa er i e c i er.

(b) or er o i e measuri c i er up o e cm mar rops o e


i ui are a e o e i ui a rea i e measuri c i er. a cu a e e
a era e o ume o o e rop.

(a) cm o e wa er i i . . is poure i o a ea er. i . . mar e


e e o e wa er e i e c i er.

(b) ru e ca i ra e i cm is p ace a o si e e measuri c i er as s ow i


i . . . a is e e o e measuri c i er rom ero up o e
cm mar

(c) e o ume o a c i er is ou usi e e ua io


o ume cross sec io a area x e .
e ermi e e cross sec io a area o ec i er co ai i wa er.

(d) e ce or o erwise e ermi e e pressure ac i o e ase e wa er e


i e c i er.

261
15 P sics P Pre im E

12 e circui i i . . is co ec e up.

Fig. 12.1

(a) a e ow oes e curre i e resis a ce wire compare wi e curre i


e : resis or

(b) o me er co ec e across e resis a ce wire s ows e same rea i as


a o me er co ec e across e : resis or. a cu a e e com i e
resis a ce o e wire a e resis or.

(c) e wire a resis or are isco ec e a e reco ec e i para e as


s ow i i . . .

Fig. 12.2
(i) a cu a e e ew com i e resis a ce o e wire a e resis or.

(ii) ompare e curre measure i i . . wi e curre i i . . .

262
16 P sics P Pre im E
(d) a s i ui i s some imes e e op crac s. e wi o a crac ca e
mo i ore measuri e resis a ce o a i wire s re c e across e
crac a irm ixe o ei er si e o e crac as i us ra e i i . . .

Fig. 12.3

(i) e wa mo es a e crac wi e s s i . Exp ai w a appe s o e


resis a ce o e wire.
(ii) op a sui a e circui rom i . . or i . . a a a E i u
o i up w e e crac wi e s

263
17 P sics P Pre im E
13 EITHER

The circuit below consists of a 6 V battery source, two resistors of 150 Ω each, LDR
a ermis or. e ou across e is co ec e o a a w ic wi e swi c e
o w e e ou . .

(a) e a e s ows e ra e o resis a ce a ue or e a ermis or. e


resis a ce o e ermis or aries i ear wi empera ure.

s rume empera ure i i e si esis a ce


o
200 Ω
ermis or o

ri oo a 50 Ω
i 100 Ω

(i) Exp ai w e er e a ermis or are o mic co uc ors.

(ii) a cu a e e e ui a e resis a ce o e w o e circui w e i is uri e


i a e empera ure is o .

(iii) i e rea i o e amme er.

(b) (i) ow wi c ear wor i s a ca cu a io s w e er e a wi e


swi c e o w e e empera ure is o uri e i .

(ii) ow wi c ear wor i s a ca cu a io s a possi e pair o co i io s


or e empera ure a i i e si w ic wi cause e a wi o e
swi c e o .

264
18 P sics P Pre im E
13 OR

e ur i e o a ro e ec ric power s a io is ui e ow e e e o a a e as
s ow e ow.
o nt Al l
a eAl l
ate to ontrol
a o ater r ine it
m
enerator
eed o di ar ed ater 10
m

e ur i e is ro a e e ru i wa er. e ro a es a roup o ma e s
arou a coi i a a.c. e era or. e e era or is esi e suc a e ma e s
wou ro a e a a co s a spee e era i a e.m. o across a coi . e
wa er wou e isc ar e a a co s a spee o m s i o e ow s ream ri er.
e mass o wa er passi rou e a es per seco wou e co ro e
a au oma ic a e. a curre is raw rom e e era or coi ere wou ea
c oc wise mome opposi e ro a io o e ur i e. e ru i wa er wou
ee o pro uce a a i c oc wise mome i i e a es i or er o
o ercome e resis i mome a eep e a es mo i a e re uire
co s a spee o e era e e e.m. .o . o curre is raw rom
e e era or m o mome wou oppose e ro a io a . o
wa er wou e ee e o i e a es per seco .

ur i e
a es

. m
E

e a e e ow s ows e aao ou uu roe ec ric power s a io


urre ome
ass o wa er pee o
e era e raw rom re uire o
i i e a es isc ar e
e.m. . e era or ur e
per seco s wa er ms
ur i e m
.
.

265
19 P sics P Pre im E
(a) ssumi a wa er i s o e a e a a ime a a perpe icu ar is a ce o . m
rom e e ax e as s ow e ow es ima e e orce i mus exer o e a e
w e a curre o is raw rom e e era or.

(b) Exp ai w “If a current is drawn from the generator coil, there would be a
clockwise moment opposing the rotation of the turbine”?

(c) a cu a e e amou o e er per seco a ai a e o e e era or w e


o wa er ows rou e ur i e per seco .

(d) Es ima e e curre a ows i e e era or w e o wa er ows rou


e ur i e per seco .

(e) Exp ai rie w e ac ua curre owi s ou e ess a our es ima e


a ue i (d) w e o wa er ows rou e ur i e per seco .

END OF PAPER 2

266
20 P sics P Pre im E
Blank Page

267
Geylang Methodist School (Secondary)
Preliminary Examination 2018

PHYSICS 5059/01

Paper E Sec 4 Express

i io a ma eria s
1 hour
hou

Setter : r ip e ou
24 August
Augu
Au g st 2018

E E E

ri e our ame i ex um er a c ass


ass
s o a e wor
woor ou o a i .
o o use s ap es paper c ips i i erss ue or correc
cor
orre
rec
c io ui .

Do not open this booklet until you a


are
re ttold
o d to do so.
ol
swer all ues io s. a e ou
our
ur a sw
swer
swers
ersso e pro i e .

ee o e exami
m a io
o su mi
mi a e ues io paper separa e .

INFORMATION
NFORMATION FO
FOR
OR CA
CANDIDATES

Eac correc a swer wi score o e mar .

rou wor s ou e o ei is oo e .

cce era io ue o ra i g is assume o e m s o Ear a . ms o oo


u ess o erwise speci ie .

is ocume co sis s o 16 pri e pa es


Turn over

268
2 P sics P Pre im E

1 Pressure ca e e ermi e usi e o owi ormu a

P FyA
F mua

w ere P pressure pasca Pa A area m m mass


m
F orce ew o s a acce era io
s2
e u i pasca is e ui a e o
A kg B 2
s C 1 D kg
sm kgm kgs2 m s 2m

2 Fig. 2 shows Jenny’s setup of the following experiment.

me a ca wi mar er pe i si e

paper
pa
p aper

woo e ramp

Fig.
g 2

e re ease e ca a i ro e reree ow
o w e ramp wi ou s ippi . a
i s e o ser e o e paper ma e e mar
mar er
e pe
A e o s e ar er apar .
B e o s e c oser o e e er.
r.
C e o s e s c oser
ose
ser e ur er apar .
D e o s are
arre e ui is a rom
rom eac
eac o er.

3 ro e X a ro e Y are oi e
ro o e er a s re c e spri . ro e X as
wice e mass r e Y.
ss o ro e e ro e s are re ease e acce era io o X is
. m s o e riri .
X
Y

a is e i i ia acce era io o ro e Y o e e
A . ms B . ms C . ms D . ms

269
3 P sics P Pre im E

4 ree o ec s are cu rom e same i s ee o me a . e a e e same


o ume u i ere s apes.

isc s uare s ape

ic o e o owi s a eme s is true


A e a e i ere mass a posi io o . .
B e a e i ere mass u same posi io o . .
C e a e e same mass a posi io o . .
D e a e e same mass u i ere posi io o . .

5 e ia ram e ow s ows wo immisci e i ui s P w iic


c a e e
eee ppoure
oure
i o a ma ome er w ic is expose o e a mosp ere a e op o o a arms.
rms.
rms.

a is e ra io o e e si o i u
ui o i u
uii P
A B C D

6 e ia ram e o ow
w s ow
owss a u i orm me re ru er wi a wei o u er e
ac io o a er ica or
orce
ce o . .

0 cm
m 100 cm

30.0 N

w a mar mus a u crum e p ace o o e ru er i e ui i rium

A . cm B . cm C . cm D . cm

270
4 P sics P Pre im E

7 ic o e o owi o ec s as e least i er ia
A ro o o wei ra e i o e oo .
B ro o o wei ra e i o Ear .
C s a io ar roc o mass o e oo .
D s a io ar roc o mass o Ear .

8 e ia ram s ows a simp e mercur arome er.

e e a mosp eric pressure i creases w ic is a ce i creases


crrea
easses
A B C D

9 a w ic wei s is mo e up
p a s op
ope
pe rrom
om o app i a orce o
as s ow i w.
e ia ram e ow.

3 3N
5
4

ssumi o e er isis os
os o e surrou i w a is e ai i i e ic e er
ai i ra i a io a po e ia e er a wor o eo e a
Kinetic
Kineti
ic En
Ener
Energy
ergy
g Gravitational Potential Energy Work Done
A
B
C
D

10 power u a uses a a er . e a e era es o ea e er seco


as i s mo or ur s.

a is e e icie c o is a
A . B . C . D

271
5 P sics P Pre im E

11 ic i es e s a es o ma er i w ic mo ecu es a a i e empera ure a e


e sma es spaci e wee em a mo e a e owes spee
Smallest spacing between molecules Molecules have the lowest speed
A oi oi
B i ui oi
C oi i ui
D i ui i ui

12 ic o ese examp es as e wro pe o ea ra s er associa e wi i


Example Type of heat transfer
A ea rom ame o i pa o ucu io
B u i warmi e i erior o a car o ecec io
C oo i a cup o co ee wi a s ee spoo o ucuc io
D ea rom ur i o s o perso esi e ire p ace o ece ioo

e rap e ow s ows e c a es i empera ure o a so


o i w e i is ea
ea e
a ea er wi a ra i o . se i o a swer ues io s o .

13 ic o e o owi s a eme s s ow a i ere ce e wee ea a


empera ure.
A e o ea is
i supp ie
i e empera ure reac e q .
B ea as a u i i Ke i w i e empera ure u i is i q
C ea i creases w e empera ure i creases.
D ea remai s co s a re ar ess w e er empera ure c a es.

14 ssumi ere is e i i e e er oss o e surrou i s w a is e speci ic


ea capaci o e i ui
A q B q C q D q

272
6 P sics P Pre im E

15 ssumi ere is e i i e e er oss o e surrou i s w a is e speci ic


ae ea o usio o e so i
A B C D

16 e rap s ows e er ica isp aceme o a o ec oa i o wa er as a wa e


passes rou e wa er.

a is e re ue c o e wa e

A B C D

17 p a e mirror . m i e is u o a er ica wa wi i s o om
m . m
a o e rou . o wi is e es . m a o e rou
rou oo s i o e mirror o
see e ima e o imse .

a e o imse
im s eo
ow is e es ca e see i e mirror

A . m B . m C . m D . m

18 ra o i e ers a prism ma e o ma eria X a ra e s a o e pa as


s ow i e i ure e ow.

a is e re rac i e i ex o e ma eria X
A . B . C . D .

273
7 P sics P Pre im E

19 ree o ec s P Q a R are iewe rou a p a e mirror as s ow .


o s ac e mo es owar s e mirror as i ica e e arrow.

x x

ic ima e wi isappear irs a w ic ima e wi is


saap
ppp
pe
isappearea
ear as
isappears irs isappears
is
saap
app
pppe
eaars as
A P Q
B P R
C Q R
D R Q

20 i ra passes rou rree


ee m
mee ia o re rrac
ac i e i ices η η a η
respec i e .

T
T

me ium me ium me ium


η η η

i e a T T T w ic o e o owi is correc

A η >η >η B η >η >η C η >η >η D η >η >η

274
8 P sics P Pre im E

21 ree ra s o i a o a co er i e s as s ow .

ic ia ram s ows e pa o e ra s a er passi rou e e s


A B

C D

22 s u e arra ese a i umi a e o ec a e s a a scree suc a e si e o


e ima e is wice
e a o e o ec . Keepi
w ce
wi e is a ce e wee e scree a
e i umi a e o ecc ixe
ixe e exc a es e posi io o e scree a e
i umi a e o ec .

a wou e o ser e o e scree


A urre ma i ie ima e.
B urre imi is e ima e.
C s arp ima e wice e si e o e o ec .
D s arp ima e a e si e o e o ec .

275
9 P sics P Pre im E

23 ic o e o owi rap s a ou e ec roma e ic wa es ra e i i acuum


is are correc

A o B a o C a o D a

24 e ia ram s ows e mai sec io s o e e ec roma e icc spec


spec ru
rum
um i o
orr er o
i creasi re ue c . ome o e sec io s are a e e . e sec
sec io as a
re ue c us e ow a o i .

ic app ica io uses o


e sec io
A eri isa io
B a e i e e e isio
C rea oas err
D aser poi er er

25 e se up s ow i e iaia ram e ow co sis s o a osci oscope a a spea er


a emi s a ssou
ou wa
wa e wi re ue c . microp o e p ace . m
rom e spea
spea eerr e ec s e sou emi e e spea er. wa e orm is
o ser e o os oscope represe i
e osci e sou emi e .

osci oscope

microp o e
spea er

e spea er is e mo e o a is a ce . m owar s e microp o e a a ew


wa e orm is o ser e o e osci oscope. ompare o e ear ier wa e orm is
ew wa e orm as a rea er .

A spee B amp i u e C re ue c D wa e e

276
10 P sics P Pre im E

26 Fig. 26.1 is a full-scale ia ram a s ows air par ic es a eir e ui i rium


posi io s represe e o e i es . e a sou wa e passes par ic es are
isp ace i o ew posi io s s ow i Fig. 26.2. Q represe s a par ic e.

Fig. 26.1

Fig. 26.2

ic o e o owi s a eme s is a se
A Poi Q is e ce re o a rare ac io .
B Par ic es ex o Q are mo i i opposi e irec io
io .
C e amp i u e o e wa e is . cm.
D e wa e e o e wa e is . cm

27 ic o e o owi ia rams correc s ows e u i orm e ec


ec ric ie e wee
wo c ar e para e me a p a es

277
11 P sics P Pre im E

28 wo u c ar e me a sp eres P a Q are p ace o e er wi a ic car oar


i ser e e wee em. o sp eres are suppor e i su a i s a s a Q is
ear e wi a wire.

e ai e c ar e ro is rou ear e sp eres as s ow .


a wou e e is ri u io o c ar es o sp eres P a Qw e e wire is
remo e o owe e c ar e ro
P
A e ai e posi i e
B e ai e eu ra
C eu ra posi i e
D eu ra eu ra

Refer to the following information for Questions 2


299 an
a
andd 30.
e rap e ow s ows ow e currec rrre i a amp
cu amp a a resis or aries wi e
po e ia i ere ce app ie .

e amp a e resis or are co ec e i series as s ow e ow a e amme er


rea i is . .

278
12 P sics P Pre im E

29 e ermi e e e.m. . o e circui .

A . B . C . D .

30 e ermi e e resis a ce o e resis or.

A . : B . : C . : D . :

31 e ia ram s ows a experime a se up o a e ec ric circui o e ermi e e


resis a ce o a u .

ic o e o e o owi s ows cor


o rec circui
e correc cir
ircu
cui ia ram

279
13 P sics P Pre im E

32 e circui s ow R1 a R2 are i e ica resis ors.

ic o e o owi c a es o e e ec rica compo e s X a Y wi


w ecrease
e rea i o e amme er e rea es amou
A mmerse X i a ea er o ice wa er a ecrease e i si o Y.
i e si
B mmerse X i a ea er o ice wa er a i crease e i si o Y.
i e si
C mmerse X i a ea er o o wa er a ecrease e i i e si o Y.
Y.
D mmerse X i a ea er o o wa er a i crease e i i e si o Y.Y.

33 a pi p u o a acuum c ea er e use is missi


miiss
sii .
ic o e o owi s a eme is correc
A e acuum c ea er ca e ur e o u i ee e o a e ec rica au e
acuum c ea er wi e i e.
B e acuum c ea er ca e ur e o u i e e e o a e ec rica au e
acuum c ea er wi e sa e o a e ecause
ecau
use e curre
curre wi ow o ear .
C e acuum c ea er o ce ur e o wi s u ow
o w a o ce.
D e acuum c ea er ca o e ur
u e o .

34 ree i e ica amp


amps
mps a are
are co ec e i a circui as s ow e ow.

● ●

a wi appe o e ri ess o e amps i e resis a ce o e r eos a is


i crease

A ri er immer immer
B ri er ri er immer
C immer ri er immer
D immer ri er ri er

280
14 P sics P Pre im E

35 su e carries ou our es s wi a ma e.

ic resu s ow is not correc


arra eme esu

A ma e ma e rac s

B ma e iro ar a rac s

C ma e iro ar repe

D ma e copper ar o e ec

36 rass ro is arra e i a or sou irec io a po i ccompasses


ompass
om sses
es a
are
re p ace
a eac o i e s.

ic ia ram s ows e posi io s o e ee es o epo i compasses

281
15 P sics P Pre im E

37 Pa represe s wo para e s rai wires carr i curre s. P a exer


orce o eac o er. ic arrow s ows e orce o

38 ree wires P Q a R are eac carr i a curre o i e irecc io as s ow


i e ia ram e ow.

a are e irec io s o orces ac i o e ree wires


ree wires
Wire P Wire
Wi re Q Wire R
A e e e
B ri e ri
C e ri e
D ri r
ri ri

39 e i ure e ow s o
ows a i ea ra s ormer wi e seco ar coi co ec e o
a : oa .

a is e curre i e primar coi


A . B . C . D .

282
16 P sics P Pre im E

40 e ia rams s ow a simp e a.c. e era or a e rap i us ra i e wa e orm o e


e.m. . ou pu o e a.c. e era or.

ic o e o owi ia rams correc s ows e p a e o e arma a ur


uree coi
co o e
e era or as iewe a o e ax e rom e posi
si io
io o e s ip
ip rrii s a imime
me i er a s
e oe t1 t2 a t3 o e rap

END OF PAPER 1

283
Geylang Methodist School (Secondary)
Preliminary Examination 2018
a i ae AN ER
ame

ass ex um er

PHYSICS 5059/02
Paper P sics Sec 4 Ex
Se Express

i io a ma eria s ri i Papers 1 hour


hour 4
455 minutes

Setter : r ip e ou 24 August
Augu
Au gust 2018

READ THESE INSTRUCTIONS FIRST


ri e our ame i ex um er a c ass o a e wor ou a i .
ri e i ar ue or ac pe o o si es o e paper.
o o use s ap es paper c ips i i ers ue or correc
corre
ec io ui .
swer all ues io s.
ri e our a swers o Section A i e spaces
spa
pace
ces pro
pro i e i e ues io Paper.
ri e our a swers o Section B i wri i papapers
ape
persrs pro
pro i e .
Question 13 has a choice of parts to
o answer.
ans
nswe
wer.r.
a i a es are remi e a a ua a i a i e a swers s ou i c u e appropria e u i s.
ou are
arre a ise o s ow
ow a our
our wor
wor i i a c ear or er ma er.

e e o e exami
exam
ami a ioo as e a our wor secure o e er.
e um er o marr s is i e i rrac e s a e e o eac ues io or par ues io .
cce era io ue o ra
ra i g is assume o e m s u ess o erwise speci ie .

For Examiner’s Use

Section A

Section B

Total

is ocume co sis s o 16 pri e pa es.

Turn over

284
2 P sics P Pre im E
Section A

swer all e ues io s i is sec io i e spaces pro i e .

1 i . . s ows a ir ua ima e I orme a co er i e s rom a o ec o


ei . cm.

Fig. 1.1

(a) i . . a o e raw ra s o e ermi e

(i) e posi io o e o ec a e e o ec O.

(ii) e oca e o e e s.
4.1 cm (r
r 0.3 cm)
oca e ……………………

285
3 P sics P Pre im E
(b) i . . s ows a i ra ra e i i e co er i e s o re rac i e i ex . .
e ia ram is o raw o sca e.

Fig. 1.2

a cu a e e cri ica a e a exp ai e e a iourio


ouurr o e i a eer i is
i ci e o e sur ace P.
………………………………………………………………………………………
………………………………………………………… ………………… ………………
n 1 / sin c
1.5 1 / sin c
c 41.8q
angle of incidence
incidenc ce of
of 5
50
50q
0q wh
which
hic
ich is
s gr
g
greater
rea
e te er than
than
than critical
cri
riti
tical angle of 41.8q so the
ray will goo tthrough
h ough
hr g ttotal
otal
ot a iinternal
ntterrna
n nal re
rreflection
efl
f ecti
ttiion
………………………………………………………………………………………
…………………………………… …………… ………… ………… … ……………………………

(c) a e a use o a compo e o e e ec roma rrom


o a e ic spec rum wi e sma es
wa e e a escri
crii e e e ec
esscr ec o a sor i is e ec roma e ic wa e.
………………………………………………………………………………………
…………………… ……………… ………… ………… … ……………………………………………
cancer
canc
ncerr treatment
nc treattm
me
ent byy de
d
destroying
est
strro
strooyyiiin
ng cancer cells OR sterilising of surgical
instru
instruments
ume
mentss dest
destroys
tro
roy
roys
ysso orr m
modifies
odifies / mutates living tissues and cells
………………
………………………………………………………………………………………
………… ……… ………………………………………………………………

2 e spee o a u rasou i air is m s.


(a) omp e e i . . o s ow ow ar e u rasou wa e as ra e e
a seco s a er e u rasou was pro uce .

Fig. 2.1

286
4 P sics P Pre im E
(b) i . . raw e rap o is a ce ra e e a ai s ime or e
u rasou wa e.

Fig.
F
Fi g. 2.2
(680 1020 1361360
36
60 1 17
1700)
700
00) 5 points
po
p oin
i tts
s plo
plotted
ott
ttted
ed r
ed small square ignore 0 0
(e.c.f.)
reas
reasonable
ason
as o ab e best-fit
ble b s
be stt-fiit line
line through
thr
hroou
ug h points
gh po nts drawn
oiint drawn with rule
dr
(c) aea escri e w ic pe
p e o wa a e is u rasou .
……………………
………………………………………………………………………………………
…………………… …………… … ……………………………………………
Longitudinal
Lo
on
ng
git
i tu
uddin
inal that
al wave th at ttravels
hat ra
avel
ve
vels parallel to the vibration of wave particles.
………………………………………………………………………………………
…………………… ………
………… ……………………………………………………………

(d) Exp ai w a iss waw ee a s a e e maximum wa e e o e


u rasou
ou .
………………………………………………………………………………………
……………… …………………………………………………………………………
Wavelength is the distance between 2 successive points on the wave
that are in phase. Max wavelength Speed y Min frequency 0.017 m
………………………………………………………………………………………

287
5 P sics P Pre im E
3 Experime o e wo oc s o mass a were a ac e a pu e
across a rou rou i e irec io s ow i Fig. 2.1.

oc ri oc Pu i
orce

Fig. 2.1

e experime is e repea e o e same sur ace u wi a i ere pu i


orce. e c a es i e oci ies o e oc s or o experime s were
were m easure
measure
a a u a e i Fig. 2.2.

Velocity / cms-1
Time/s Experiment One Experiment
Experi
ime
ment
nt Two
Two

Fig. 2.2

(a) si e a ues i Experime e as s ow i Fig.


Fig 2.2 ca cu a e e
(i) acce era io o e oc s.

accelerarra
ati
accelerationtion
o ((v
v-u
u)) / t
(2
(20
20 - 4)
4) / 4
4 cm/s
cm
m/s2 ((2) 2)

acce era io ……………………

(ii) e sio i e s rrii i e a e ric io e wee e rou a oc


is
s . .

T – Fric
Friction
cttiion
o Resultant
Resultant Force
T – 3.0 N (5 (5 kg)(0.04 m/s2) (1)
T 3.2 N (1)

e sio ……………………
(iii) ma iu eo e pu i orce .

F – Friction – Tension Resultant Force


F – 3N – 3.2N (10 kg)(0.04 m/s2) (1)
F 6.6 N (1)

288
6 P sics
cs P Pre
Pre im E
pu i orce …………
……
……………………
………
… ……

(b) erms o orces exp ai w e e oci o e occ s i Ex Experimeme e


c a es o er ime u remai s u c a e i Experime Experim me wo.
o.
wo.
In experiment 1 there is a resultant forc ce wher
force erreas
whereasea
e as in
in experiment
ex
xppe
errimen nt 2 there
there
he
he
…………………………………………………………………………………………
……………………………………………………… …… …… ……………… ……
…… …… ……………
………… ………
is zero resultant force. (1) Hencee in exp
experiment
perimmen nt 2 the
the
he pulling
pulllin
ng force
forc
ce is
s
…………………………………………………………………………………………
………………………………………………………………………… ………… …… …………
equal to the frictional forc
ce.
e. H
force. enc
ence
Hencee a ccelera
ra
atiion is zero (1
acceleration 1) in
(1) ndicating
indicating
…………………………………………………………………………………………
……………………………………………………………………… …………………………
that the blocks are mov vin
i g at
moving at constant
con
onst
sta
annt velo
lo
oci
c tyy.
velocity.
…………………………………………………………………………………………
…………………………………………… ………
… …… ………………… … ………………………

289
7 P sics P Pre im E
4 i . . s ows a rec a u ar oc o woo o a a rou ori o a oar .

(1)

Fig.
Fi
ig.
g 4.1

e oc is pus
u e a e op as s o
oww i i . . a i i s o e ri .
(a) e ro ace o e occ raw
raw e i e a wi e er ica a ei sa
e ore e oc opp es o er.
er.

(b) se e pr
proo racc or
o s oow
w o i . . o measure ea e rou w ic e
oc i s eo ore i oopp ess o er.
pp e 24 26 q
a e ……………………

(c) e oc is p
pu
u ac o e oar as i i . . . is ime i s ea o e
oc ei pus
ei pus e e e a e eo e oar is raise .

ae e a e a e oar ma es wi e ori o a a e i sa e
er
oc opp es o er. same answer as (b)
a e ……………………

(d) Exp ai i erms o . . a s a i i ow our a swer o (c) mi i er i


e proce ure is repea e a er e ei o e oc is re uce .
The angle of tilt to topple the block will increase (1) because the C.G of
............................................................................................................................
the block has been lowered so it is more stable (1).
............................................................................................................................

290
8 P sics P Pre im E

5 (a) e pri cip e o co ser a io o e er s a es a e er ca ei er e


crea e or es ro e . a e does appe o e e er supp ie o a
e ice suc as a mo or or a e e isio
It has been changed / converted / transferred into other forms of
..........................................................................................................................
energy.
..........................................................................................................................

(b) e e e isio i i . . is swi c e o o wa c a pro ramme. uri is


ime o e ec rica e er is supp ie .

Fig. 5.1
(i) rom e i or
o
orma
ma iio
o o i . . i e o a e er pro i e or e iewer
o see a ear
ar e e e isio
i io uri
is is pro ramme.
24 k
e er ……………………
(ii) u ess w a appe s o e res o e e er supp ie .
Lost to th the
he surrounding / converted into energy that are not useful.
..........................................................................................................................

..........................................................................................................................

(iii) a cu a e ow muc e er is i o e i (b)(ii).


720 - 24 696 k
e er ……………………

291
9 P sics P Pre im E
(iv) a cu a e a comme o e e icie c o e e e isio .
Useful o/p y total i/p x 100 24 / 720 x 100 3.33 (1)
..........................................................................................................................
Very low in efficiency / very high loss or waste in energy (1)
..........................................................................................................................

6 i . . s ows a e apora i e coo er w ic is a e ice a coo s air rou e


co i uous e apora io o wa er. ir is coo e ei ow pas a coo i pa
co ai i wa er. e coo i pa is we a pump w ic pumps wa er up o e op o
e pa rom w ere i ric es ow .

Fig. 6.1

(a) erms o i e iicc mo e o ma er exp ai ow co i uous e apora io o


e wa er
er causes
caus
ca es e air owi pas e coo i pa o e coo e .
uses
The wwaate
ter mo
water m ole
lecules on the surface of cooling pad absorb thermal
molecules
………………………………………………………………………………………
………… ……………………………………………………………………………………
energy from
fro
fr om
m the warm air (1) increasing their internal energy till their
………………………………………………………………………………………
overcome strong intermolecular forces enabling them to vaporize
………………………………………………………………………………………
leaving behind less energetic cooler water molecules. (1) The air then
………………………………………………………………………………………
exits the cooling pad as cool air. As the new wave of warm air flows
………………………………………………………………………………………
past the cooling pads the same process repeats relying on the cooling
………………………………………………………………………………………
process of evaporation.
………………………………………………………………………………………

292
10 P sics P Pre im E
(b) Exp ai w e er suc e apora i e coo i s s em is more e ec i e i r
or umi a mosp eres.
It is more effective in dry atmospheres because places with high
humidity reduces the rate of evaporation.
………………………………………………………………………………………
[That is why windows and entryways must be closed while running a
ventilation system]
………………………………………………………………………………………
(c) u es o e mo i ica io a ca e ma e o e esi o e e apora i e
coo er suc a e ra e o coo i i creases.
Increasing surface area of the cooling pad increases the rate
………………………………………………………………………………………
…………………………………………………………………………… … ……
…………
of thermal loss from the air to the liquid.
………………………………………………………………………………………
…………………………………………………………………… ………… ……………

7 wo me a saucepa s co ai e same mass o o wa


wa erer a e ssame
ame i i ia
empera ure. Pa is w i e a pa is ac u o erwise
erwi
erwis e e wo saucepa
se sau
auce
c pa s are
i e ica . o saucepa s are u co ere a coo
oo
o u erer e same
same co
co i io
io s. e
coo i cur es or e wo saucepa s are s ow i i . . .

Fig. 7.1

293
11 P sics P Pre im E
(a) Exp ai w pa coo s as er a pa .
Pan B is black which is a better emitter of heat compared to white (1)
.............................................................................................................................

.............................................................................................................................

(b) escri e a exp ai ow i . . is i ere w e e pa s are co ere a


e experime is repea e .
The rate of cooling will be lower OR temperature will take a longer time
.............................................................................................................................
to drop to a same temperature (1). This is because the cover will reduce
...............................................................................................................................
.............................................................................................................................
heat loss by convection (1).
.............................................................................................................................
.............................................................................................................
...................

(c) Exp ai w a is mea e speci ic ea capaci o wa er er iss .


4200 of heat is required to increase / decrease th he te
the em mp peratur
temperature u e of 1
ur 1kgkg
.............................................................................................................................
............................................................................... .....
...............
.................................
. ........
of water by 1 qC.
.............................................................................................................................
............................................................................
..............
......
............................................
(d) e speci ic ea capaci o wa er is er i . u es o e isa a a e o
is w e wa er is use or coo i .
It will require a high amount amo
mount of
mo heat
o hea ea at (o((or require
or re quire a longerr ttime
equ me of heating)
im
.............................................................................................................................
................................................................................ . ..............................................
in order to raise se
s e the
the
h tem temperature
mpepera
perarature off water fo ffor
or c co
cooking.
ookoking.
..................................................
.............................................................................................................................
...
....
. ....................
....
. .....
......
. ...........................................
(e) e wa er i pa coo s or mi u es as s owow i i . . . uri is ime
e wa er oses a a era e o erm e er per mi u e.
o erma
(i) a cu a e e mass o wa aeerr i pa
a .

Q mcm c
c'T
mc'T T
9000
9
90000
0 x 8 min
min
i mx4 4200
200
20
00 //kgqC
kgqC x (94qC – 60qC)
m 0. 0.504
0 5 4 kg (3
50 3 s.f.)
s.f
.f.)
.)

mass ……………………

(ii) e mass
maass
s o wa er i pa is e same as a i pa .
a cu a e e erma e er os rom e wa er i pa uri e
mi u es
es.

Q mc''T
Q 0.504 kg x 4200 /kgqC x (94qC – 40qC)
Q 114000 (3 s.f.)

oss o erma e er ……………………

294
12 P sics P Pre im E

8 eac er emo s ra es ma e ic scree i . e a ma e is p ace ear a sma


car oar ox paper c ips o e o er si e o e ox are pic e up as s ow i i .
. . e a sma piece o so iro is p ace i si e e ox as s ow i i . . e
paper c ips a o . a e ic ie i es i eac ia ram are s ow as i i es.

Fig. 8.1 Fig. 8.2


(a) i . . mar a arrow w o eac
eacc o e ma e ic ie i es o s ow i s
irec io . No
North
orrtth
th to
to South
So
outh
(b) Exp ai w p aci
acci e so iro
ro i e ox causes e paper c ips o a o .
Soft
Sooft iiron
r n is h
ro highly
ighlh y pep
permeable
errmme eaabl
b e to magnetic field (1).
.............................................................................................................................
............
........
. .....
..............
. ...........................................................................................
Magnet
Magnetic etic
et ic ffield
ie
eld
ld ffrom
rro
om tthehe magnet does not reach the paper clips so they
.............................................................................................................................
..............
.....
. ......
. ..
.....................................................................................................
demagnetized
demagnet etiz
et izeedd and fell off (1).
.............................................................................................................................

295
13 P sics P Pre im E
9 e appara us i i . . is ca e a orce o co uc or a a ce. e ere is a
e ec ric curre I as s ow i ere is a orce o . is orce is measure
pu i wei s i e pa u i is rou ac o i s ori i a posi io .

Fig. 9.1
(a) a e w a wou appe i e curre irec io were
we rom o .
XY would mov
move up / anticlockwise
ve up anttic
icllo
oc
ckkwise
s / motionn rreversed
ev
e errsed / pan moves down
ve
…………………………………………………………………………………………
…………………………………… ………
……………… ………… ………… … …………………………

(b) s u e uses e a a ce e o ee erm


ermi
rmii e e orce F o or i ere
curre s I. e co c u e a F is ir
irec
rec propor io a o I ase o is resu s
i e e ow.

ow c e
ear
ar wor
wor i s o es ima e e orce o w e e curre is . .

Assuming (2.0 0.047) is on the best-fit line


m 0.047 y 2.0 0.0235
Using y mx
y 0.0235 x 1.6
0.036 0.038 N
orce ……………………

END OF SECTION A

296
14 P sics P Pre im E
Section B
swer all e ues io s rom is sec io .
swer o o e o e wo a er a i e ues io s i Question 13.

11 measuri c i er co ai s wa er w ic as a e si o m.

Fig. 11.1 F
Fi
Fig.
g.. 11.2 Fig. 11.3

(a) i . 1.1 indicates four way


ways
ys th
the
e obse
observer’s
serv
rver
er’s
’s eeye
y could look when taking the
ye
rea i rom e measuri c i er.
er. Pu a circ
c rc e arou
ci e e e posi io a
i es e correc rea i a s a e e o ume um o wa er i e c i er.
90 cm3
o ume

(b) or er o i e m
measuri
easu
ea suri
ri c i er up o e cm mar rops o e
i ui are a e o e i ui a rea i e measuri c i er. a cu a e e
a era e o ume
ume o o e rop.
10 cm3 / 80 0.125 cm3
o ume

(a) cm o e wa er i i . . is poure i o a ea er. i . . mar e


e e o e wa er e i e c i er.
level shown at 70 (ignore meniscus) 1mm

(b) ru e ca i ra e i cm is p ace a o si e e measuri c i er as s ow i


i . . . a is e e o e measuri c i er rom ero up o e
cm mar
16 cm
e

297
15 P sics P Pre im E
(c) e o ume o a c i er is ou usi e e ua io

o ume cross sec io a area x e .

e ermi e e cross sec io a area o ec i er co ai i wa er.


100 cm3 area x 16 cm (1)
area 100 cm3 y 16 cm
6.25 cm2 (1)
cross sec io a area

(d) e ce or o erwise e ermi e e pressure ac i o e ase e wa er e


i e c i er.
P FoF
Force
orce / Area
1000 kg/m3 x 70 cm3 x 10 0 N/kg
N/kkg y 6
6.25
.25 cm2 (1)
(1 g/cm3 x 70 cm3)y1000 x 10
10 N/kg
N//kg
N kg y 0.00
0.000625
0006
00
0006625 m2 (1)
1
1120
120
12 0 N/kg
N kg or
N/ or 1
1120
12
20 Pa
Pa (1)
pressure
pr
p re
es
sssu
su
surre
e

12
2 e circui i i . . is co ec e up.

Fig. 12.1

(a) a e ow ow oes o s e curre i e resis a ce wire compare wi e curre i


e : resis resi
re siss or
They are the the
h same in amount / magnitude.
.............................................................................................................................

.............................................................................................................................

(b) o me er co ec e across e resis a ce wire s ows e same rea i as


a o me er co ec e across e : resis or. a cu a e e com i e
resis a ce o e wire a e resis or.

298
16 P sics P Pre im E

Same voltmeter reading implies resistance wire is also 2 : (1)


? combined resistance in series 2 + 2 4 : (1)

com i e resis a ce ……………………

(c) e wire a resis or are isco ec e a e reco ec e i para e as


s ow i i . . .

Fig. 12.2

(i) a cu a e e ew com i e resis


res
esis
is a ce
eo e wire
wir
ire
ea e resis or.

New
Ne
ew co
c
combined
m ined
mb ed rresistance
ed es
e sis
istta
annc
ce in
in parallel
pa
arrral
allle
a lel
el (2x2)/(2+2)
( x2)/(2+2)
(2 1: (1)

com
co i e resis a ce ……………………

(ii) ompare
e e curre
e meas
me
measure
asur
ure i i . . wi e curre i i . . .

Since
S nc
Si nce combined
c mb
co binneedd rresistance
es
e sistance decreased by 4 times new current will
.........................................................................................................................
...........
. .......
. ...........................................................................................................
incr rea
increaseease se (1)(1)
1) by
by 4 times (1).
............
.........................................................................................................................
. ..............................................................................................................

299
17 P sics P Pre im E
(d) a s i ui i s some imes e e op crac s. e wi o a crac ca e
mo i ore measuri e resis a ce o a i wire s re c e across e
crac a irm ixe o ei er si e o e crac as i us ra e i i . . .

Fig.
g 12.3

(i) e wa
w mo es a e crac
cra
rac c wi wi e s s i . Exp ai w a appe s o e
resis a ce o e wire.
Wire
Wire
Wi i stretches/length
str
teettches e //llen
e gth h in
iincreases/cross-sectional
nccrreea asesess//c
crroosss-sectional area decreases (1)
.......................................................................................................................
........................................... ........
. .......................................................................
caausing the
causing the resistance
th rre
esi
sist
s ance ce to
ce to increase
increase (1).
in
.......................................................................................................................
..............
...
. ...................
. ......
. ....
.............................................................................
(ii) op a sui a e circ
ci
circui
rcui
ui rom i . . or i . . a a a E i u
o i up
up w e e crac wi e s.

Suit
ta
ab
ble
le circuit
Suitable cir
ircu
c it chosen from Fig. 12.1 (1) Correct LED symbol
connec
connected
ecte
ec ted parallel to the wire (1)

300
18 P sics P Pre im E
13 EITHER

e circui e ow co sis s o a a er source wo resistors of 150 Ω each, LDR


a ermis or. e ou across e is co ec e o a a w ic wi e swi c e
o w e e ou . .

(a) e a e s ows e ra e o resis a ce a ue or e a ermis


ermis or.
erm
er e
resis a ce o e ermis or aries i ear wi eempera
emmp
peerra
auure.
rre
e.

s rume empera ure i i e ssii esiss a ce


o
2000Ω
ermis or o

ri oo a 50 Ω
i 100 Ω

(i) Exp ai w e er e a e
er
ermis
miis or are
are o mic
m co uc ors.
No the
they
he
h ey ar
a
aree non-ohmic
non
n-oh
ohhmi
mic b
be
because
eca
caus
use thei
ttheir
thhei
e r re
rresistance
es
siistance is not constant / current
flowwin
flowingng th
hro
r ug
ugh th
through tthem
e is no
em ot d
not diirre
ect
ctly
directly ly p roportional
ro po
p o
proportional to the p.d across them.
(ii) a cu a e e e ui a e resis res
esis
is a cce e o e w o e circui w e i is uri e
i a e empera
emp
em erra ure
ure is o .

1//R1 + 1/R2 1/R


1/R 1/Rnet
1/
1/(150
1/(1
1550
0 + 150) + 1/(200
1/((2
1/ 20
00
0+1
100)
00)
00 1/Rnet
R net 150
150 Ω
15

(iii) i e rea
rea i o e amme er.

I V/R
6 / 150
0.04 A

(b) (i) ow wi c ear wor i s a ca cu a io s w e er e a wi e


swi c e o w e e empera ure is o uri e i .
R1/(R2 + R1) u 6 100/300 u 6 2 V
No fan will not switch on.

301
19 P sics
siccs P Pre
si P im E
(ii) ow wi c ear wor i s a ca cu a io s a possi e pair
pai
airr o c
co
o i io s
or e empera ure a i cau
i e si w ic wi cause use e a w wii o e
swi c e o .
Temperature 30 oC
Light intensity night (1)
Vout R1/(R2 + R1) u 6
100/100 u 6 (1)
6 V (1)

3 OR
13

302
20 P sics P Pre im E
e ur i e o a ro e ec ric power s a io is ui e ow e e e o a a e
as s ow e ow.
o nt Al l
a eAl l
ate to ontrol
a o ater r ine it
m enerator
eed o di ar ed ater 10
m

e ur i e is ro a e e ru i wa er. e ro a es a ro roup
up o ma e s
arou a coi i a a.c. e era or. e e era or is esi esi e suc su
uc a e
ma e s wou ro a e a a co s a spee e era i a e.m.e.m
m. o
across a coi . e wa er wou e isc ar e a a co o s a speespeee o ms
i o e ow s ream ri er. e mass o wa er passi
pass
sssi rou
rro
ou
ou e a es per
seco wou e co ro e a a
auu oma
o a ic
om a e.
a curre is raw rom e e era or coi coii ere
re wou
ere wou
ou e a c oc wise
mome opposi e ro a io o e ur i e. e rru u i w waa er wouu ee o
pro uce a a i c oc wise mome i i e a es i or or er
e o o ercome
e resis i mome a eep e a es mo o i a e ree uire co s a
spee o e era e e e.m. .o . o curre iis s raw rom e
e era or m o mome wou oppose e e ro a io a . o
wa er wou e ee e o i e a es es per seco
seco .

u i e
ur
a es

. m
E

e a e e ow s ows e aao ou uu roe ec ric power s a io


urre ome
ass o wa er pee o
e era e raw rom re uire o
i i e a es isc ar e
e.m. . e era or ur e
per seco s wa er ms
ur i e m
.

303
21 P sics P Pre im E
(a) ssumi a wa er i s o e a e a a ime a a perpe icu ar is a ce o .
m rom e e ax e as s ow e ow es ima e e orce i mus exer o e
a e w e a curre o is raw rom e e era or.
Moment Fxd
17160 F x 1.5 m (1)
?F 11440 N (1)

(b) Exp ai w “If a current is drawn from the generator ccoil, oil,l, tthere
oi here
he re w
would
ould be a
clockwise moment opposing the rotation of the turbine turbine”?e”?
According to Lenz’s Law, induced current mus must
us
u st op
o
oppose
ppo pose tthe he
e cchange
hange
ha
causing it (1). Hence a resisting moment orr ffo force
orrc
ce is
is produced
produ duuced by the
induced current (1).
(c) a cu a e e amou o e er per seco a a aii a e o e e era era or w e
o wa er ows rou e ur i e per seco co .
co
Loss in Ep mgh
500 x 1010 x 3300
00
00
15 50
500
00 000
000 ((1)
1)
Gain in Ek m
mv v2
x 50
5 00 x 102
500
25
2 5 000
000 0 (unused
(unus
unnus
used
e e nerg
ne rgy
energy) y)) (1)
Energy ya v ilab
va
availableble
e 155000 000
500 000 – 25 000
00 00
00
0 1 475 4757 000 (1)
(d) Es ima e e curre a ow owss i e e er e
era
a or w e o wa er ows
rou e ur i e per seco co .
P V
Ext V
1 47
4 75 00
475 00 x 1 50 0
000 00
0
0000x (1)
? 29
29.5
9.5
.5 A (1)
(e) Exp ai rrie
e w e ac ua curre owi s ou e ess a our
es imaae ue i
a ue w e o wa er ows rou e ur i e per
seco .
Some work
wo
orrk nen
needs
eds to be done to overcome friction in the axle of the
turbine. T he energy used in this way is converted to heat in the axle.
The

END OF PAPER 2

304
305
ame ass

e re um er ex um er

KR AN I SEC ON D AR Y S CHOO L
Preliminary Examination
Secondary 4 Express

PHYSICS 6091/01
Paper u ip e oice

ues a 20 August 2018 1 hour


KRAN I SECONDARY KRAN I SECONDARY KRAN I SECONDARY KRAN I SECONDARY KRAN I SECONDARY KRAN I SECONDARY KRAN I SECONDARY KRAN I SECONDARY
KRAN I SECONDARY KRAN I SECONDARY KRAN I SECONDARY KRAN I SECONDARY KRAN I SECONDARY KRAN I SECONDARY KRAN I SECONDARY KRAN I SECONDARY
KRAN I SECONDARY KRAN I SECONDARY KRAN I SECONDARY KRAN I SECONDARY KRAN I SECONDARY KRAN I SECONDARY KRAN I SECONDARY KRAN I SECONDARY
KRAN I SECONDARY KRAN I SECONDARY KRAN I SECONDARY KRAN I SECONDARY KRAN I SECONDARY KRAN I SECONDARY KRAN I SECONDARY KRAN I SECONDARY

INSTRUCTIONS TO CANDIDATES

ere are forty ues io s i is paper. swer all ues io s.


or eac ues io ere are our possi e a swers A B C a D.
oose e o e ou co si er o e correc a recor our c oice i soft pencil o e
pro i e .

Eac correc a swer wi score o e mar . mar wi o e e uc e or a wro a swer.


rou wor i s ou e o ei is oo e .
e use o a appro e scie i ic ca cu a or is expec e w ere appropria e.

INFORMATION FOR CANDIDATES

a e e acce era io ue o ra i o e ms .

Set by : Koh Tai Xiang

This question paper consists of 16 printed pages.


Turn over

306
1 microme er screw au e is use o measure e iame er o a s ee a . s u e a es a
i i ia ero error rea i a e a rea i o e iame er o e s ee a as s ow e ow.

ero error rea i rea i o iame er o s ee a

a is e ac ua iame er o e s ee a

A 5.48 mm B 5.94 mm C 5.98 mm D 6.04 mm

2 e rap s e ow s ow ow e isp aceme s o a o ec c a es wi ime .

rap A rap B rap C

sm sm sm

s s

ic rap s s ow s e distance travelled by e o ec i creasi a a ecreasi ra e

A rap o
B rap a o
C rap a o
D a o ea o e

3 car ri er imme ia e s eppe o e ra es w e e saw a ca as i across e roa .

e car ece era e a a co s a ra e o ms . e car i a came o a s op a er i ra e e


or m.

a is e spee o e car w e e ri er s eppe o e ra es

A 3.0 m/s B 21 m/s C 22 m/s D 30 m/s

307
4 parac u is o mass is a i rou air w e e ope s is parac u e. er e parac u e
ope s e i i ia ece era io o e parac u is is ms.

e ra i a io a ie s re is .

a is e i i ia air resis a ce ac i o e parac u is a er e parac u e ope s

A 700 N B 1400 N C 2100 N D 2400 N

5 Which of the following statement(s) about an object moving in a straight line through air is correct?

I When it moves at a steady speed, the air resistance acting on it is zero.


II When it moves at a steady speed, the resultant force acting on it is zero.
III When it moves, there is a resultant force acting on it.

A II o
B Ia III o
C II a III o
D I II a III

6 a experime o e ermi e e e si o su s a ce e mass a o ume o i ere


samp es o are measure .

ic o e o owi rap s s ows e correc re a io s ip e wee e mass a o ume o

A B
mass mass

o ume o ume

C D
mass mass

o ume o ume

308
7 o e u o wa er as a mass o . e e same o e is i e up wi a o er u ow
i ui e o a mass is .

e mass o e emp o e is a e si o wa er is . cm w a is e e si o

A 7.8 g/cm3 B 8.2 g/cm3 C 13.0 g/cm3 D 20.5 g/cm3

8 ree orces are app ie o a rec a u ar car oar o si e . cm . cm as s ow . e


car oar is pi o e a e ce re .

. cm
.

. cm

.
a is e si e o orce suc a e car oar oes o ro a e a ou pi o

A 3.75 N B 5.00 N C 6.25 N D 10.0 N

9 e ia ram e ow s ows a u i orm woo e p a wi a e o cm. e mass o e


woo e p a is .

cm cm cm

e pa is pi o e a e cm mar w ic o e o owi mass s ou e use o a a ce


epa

A mass o p ace a cm mar


B mass o p ace a e cm mar
C mass o p ace a e cm mar
D mass o p ace a e cm mar

309
10 The diagram shows a tall cylinder containing some water. A syringe is used to prevent the water
from spurting out from the outlet at the bottom of the cylinder. The height of the water above the
outlet is 2.5 m. The density of water is 1000 kg/m3 and the cross-sectional area of the piston of
the syringe is 2.0 x 10-3 m2.

wa er

. m area . x m

s ri e

ou e
What is the minimum force F that must be applied to the piston of the syringe to prevent it from
moving outwards?

A 1.25 N B 8.0 N C 50 N D 12500 kN

11 am across a a e is i i e i o wo sec io s a roc . ec io P o e am is o er a


sec io u e wo sec io s are o erwise i e ica . e wa er i e a e e am is e
same ep e er w ere. e ia ram s ows a iew rom a o e o e a e a e am.

sec io P o am

wa er i
a e
roc

sec io o am

e wa er exer s a o a orce o eac sec io o e am a a a era e pressure o eac sec io


o e am.

ic s a eme is correc

A e a era e wa er pressure o P e ua s e a era e wa er pressure o .


B e a era e wa er pressure o P is ess a e a era e wa er pressure o .
C e o a orce o P e ua s e o a orce o .
D e o a orce o P is ess a e o a orce o .

310
12 e ia ram s ows a simp e mercur arome er a o si e a mercur ma ome er.

e ma ome er co ai s some rappe as.

cm

acuum

rappe
as

cm
mercur

arome er ma ome er

a is e pressure o e rappe as

A 10 cmHg B 50 cmHg C 66 cmHg D 86 cmHg

13 a is roppe o o a ar sur ace a ou ces. oes o ou ce a e wa ac o w ere


i s ar e .

a roppe
rom ere
a ou ces
o ere

ar sur ace

ic s a eme accou s or is

A E er was es ro e as e a i e rou .
B E er was es ro e as e a ra e e rou e air.
C e erma e er o e a a i s surrou i s a e i crease .
D e c emica po e ia e er a e as ic po e ia e er o e a a e i crease .

311
14 wo armers use a e ec rica powere e e a or o i a es o a . e a es o a a e e
same mass.

aeo a

e e a or

s su se approac es e ecrease e spee o e e e a or so a ess a es are i e up i


a i e ime.

ow oes is a ec e wor o ei i i eac aea e use u ou pu power o e e e a or

wor o ei i i use u ou pu power o e


eac a e e e a or

A ecreases i creases

B ecreases ecreases

C oc a e i creases

D oc a e ecreases

15 ox wi mass o . is pus e rou a is a ce o m a o a ori o a sur ace a


u i orm orce o . e ric io a orce opposi e mo io is . .

.
.

ori o a sur ace

ow muc o e wor o e is co er e i o erma e er a i e ic e er

erma e er i e ic e er

312
16 e a as is rapi compresse o a sma er o ume i s empera ure i creases.

a appe s o e as mo ecu es

A e mo e c oser o e er a eir a era e spee ecreases.


B e mo e c oser o e er a eir a era e spee i creases.
C e ecrease i si e a eir a era e spee remai s u c a e .
D e ecrease i si e a eir a era e spee i creases.

17 er sma po e rai s are suspe e i a ea er o wa er. ri i s i es rom e si e as


s ow i e ia ram.

ma ri os o i are see rou a microscope. e o s mo e i rapi c a i


ra om irec io s.

e e

microscope

po e rai s
i wa er

a are e ri os

A po e rai s ei i o er po e rai s
B po e rai s ei i wa er mo ecu es
C wa er mo ecu es ei i o er wa er mo ecu es
D wa er mo ecu es ei i po e rai s

18 ic o e o owi o ec s ai ea ra ia io o

A a ice cu e a i air
B a car wi ac me a sur aces a i air u er e su
C a me a a wi w i e sur ace a i wa er a
D as i me a sa e i e a i space aci e su

313
19 su e wis es o c ec e upper a e ower ixe poi s o a e sius sca e ermome er.

e as our rea ers P a .

x ea er P co ai s a mix ure o ice a sa .


x ea er co ai s a mix ure o ice a wa er.
x ea er co ai s oi i sa so u io .
x ea er co ai s oi i wa er.

ic wo ea ers s ou s e use o c ec e ixe poi s

A P and R B P and S C Q and R D Q and S

20 e e o mercur i e ore o a ermome er is . cm a a . cm a .

a is e e o e mercur i e ore w e e empera ure is

A 3.3 cm B 4.5 cm C 8.3 cm D 9.5 cm

21 e same ua i o erma e er is supp ie o wo so i o ec s a . e i crease i


empera ure o o ec is sma er a e i crease i empera ure o o ec .

ic s a eme exp ai s is

A as a i er me i poi a .
B as a i er e si a .
C as a i er ea capaci a .
D is a e er erma co uc or a .

22 a appe s o e spee re ue c a wa e e o a wa er wa e as e ep o e wa er
i creases

spee re ue c wa e e

A i creases i creases i creases

B i creases remai s co s a i creases

C ecreases i creases remai s co s a

D ecreases remai s co s a ecreases

314
23 e ia ram e ow s ows a wa e represe e o a isp aceme is a ce rap . e spee o
e wa e is m s.
isp aceme cm

is a ce m

. . . . . . . . . .

ic o e o owi i orma io is correc a ou i s amp i u e a re ue c

amp i u e re ue c

A . cm

B . cm

C . cm

D . cm

24 e ia ram e ow s ows e p a e iew o a o ec p ace i ro o a p a e mirror.

ic o e o e re ec e ra s o i appears o come rom e ima e o

A B C
D

10

315
25 ra o i e ers a ass prism perpe icu ar o e sur ace P a ra e s a o e pa as
s ow e ow.
P

ass
prism

a is e spee o i i ass

A 1.93 x 108 m/s B 2.00 x 108 m/s C 2.05 x 108 m/s D 2.14 x 108 m/s

26 pe e is roppe i o a s i wa er so a circu ar wa e ro s are see o ra e ou war s wi


a spee v.

e wa e e is λ w a is e ime a e or e is ur a ce a P o reac

A λ / (2v) B λ/v C 3λ / (2v) D 2λ/v

27 i rare ra ia io is emi e a . x .

a is i s wa e e a perio o osci a io

wa e e m perio s

A . x . x

B . x . x

C . x . x

D . x . x

11

316
28 e ia ram e ow represe s e posi io o e air mo ecu es i a sou wa e. e wa e e
o is wa e is . cm.

ic is a ce represe s . cm

air mo ecu e
A

29 are uma s o a e o ear u rasou

A e amp i u e is oo i .
B e re ue c is oo i .
C e spee is oo i .
D e wa e e is oo o .

30 a P are i i su a e a s suspe e o s ri s. e e are rou ear eac


o er e e a e as o ows

repe s
a rac s a
repe s P.

e e ec ric ie o P is i e i e ia ram e ow.

ic o e o owi s a eme s mus e rue

A mus e posi i e c ar e .
B ma e posi i e c ar e or eu ra .
C ma e eu ra or e a i e c ar e .
D mus e e a i e c ar e .
12

317
31 spo i a e e is co ec e oa supp . e spo i s i es a orma
ri ess or ours.

a is e si e o e c ar e a passes rou e spo i i is ime

A 6.25 C B 375 C C 5 184 C D 22 500 C

32 e o owi ia ram s ows par o a comp e e circui .

Ω Ω

a is e curre rou e Ω resis or

A 1.11 A B 1.39 A C 2.22 A D 2.78 A

33 e ia ram s ows par o a circui use o swi c s ree amps o a o au oma ica .

po e ia
i ere ce

ic row s ows e e ec o e resis a ce o e i epe e resis or a o e


po e ia i ere ce p. . across i as i e s ri er

resis a ce o p. . across

A ecreases ecreases

B ecreases i creases

C i creases ecreases

D i creases i creases

13

318
34 e ec rica app ia ce is use or ours. e curre i e app ia ce is . .

o e u i o e ec rici cos s . w a is e cos o usi is e ec rica app ia ce

A $10.56 B $34.32 C $68.64 D $286.00

35 e ia ram e ow s ows a ea er marked “ ” co ec e i para e o a o er ea er


mar e “ ”. o ea ers are co ec e oa . supp .

P a P are e powers issipa e i ea ers a respec i e w ic o e o owi is


correc

P P

A . .

C .

36 piece o s ee ca e ma e ise s ro i i wi a ma e.

e e ma e is mo e i e irec io s ow w ic po es are pro uce a a a

po e a po e a

A sou sou

B sou or

C or sou

D or or

14

319
37 A current I is flowing in a coil of wire in the direction shown.

coi o
wire
P

ic o e o owi i es e correc irec io o e ma e ic ie a poi P

A ou o epa eo e paper
B i o epa eo e paper
C o e ri
D o e e

38 e ec ro eam passes rou a pair o e ec roma es a as s ow e ow.

curre

e ec ro eam

w ic irec io wi e e ec ro eam e e ec e

A owar s
B owar s
C i o e paper
D ou o e paper

15

320
39 simp e a.c. e era or pro uces a o a e a aries wi ime as s ow .

o a e

ime s

ome a us me s are ma e o e a.c. e era or o pro uce e o owi rap .

o a e

ime s

a are e a us me s ma e

um er o ur s o e coi spee o ro a io

A remai s co s a ou e

B ou e ou e

C remai s co s a a e

D ou e a e

40 oor e is esi e o opera e w e co ec e o a supp . e co ec e o a


ra s ormer e curre i e oor e is . a i opera es orma .

e ra s ormer is co ec e o e mai s supp a i as a e icie c o .

a curre is raw rom e mai s supp

A 0.075 A B 0.083 A C 27 A D 30 A

- End of Paper –

16

321
ame ass

Section B 30 marks
swer all e ues io s i is sec io .
swer o o e o e wo a er a i e ues io s i Q11.

9 i . . s ows a i s u ecoc a is use or p a i a mi o . a experime


usi e ec ro ic appara us a s u ecoc is re ease rom res a e is a ce a e is
measure a i ere imes .

Fig. 9.1

i . . s ows e resu s o ai e w e e s u ecoc is roppe rom a ei o a ew


me res.

s m

. .
. .
. .
. .
. .
. .
. .
. .
. .
. .

Fig. 9.2

(a) i . . raw a a e e orces ac i o e a i s u ecoc .

322
(b) Exp ai ow e aa i i . . or e i s u ecoc su es a e spee is
i creasi a . s.

.........................................................................................................................................

.........................................................................................................................................

...................................................................................................................................

(c) si e aai i . . e ermi e e ermi a e oci o e i s u ecoc .

ermi a e oci ................................

(d) Exp ai i erms o orces

(i) w e s u ecoc acce era es a irs

................................................................................................................................

................................................................................................................................

................................................................................................................................

................................................................................................................................

................................................................................................................................

...........................................................................................................................

(ii) w e s u ecoc reac es a s ea spee .

................................................................................................................................

................................................................................................................................

................................................................................................................................

................................................................................................................................

................................................................................................................................

..........................................................................................................................

323
(e) Exp ai w a wi appe o e ermi a spee o a s u ecoc w ic as a mass
a e i si e e co e o e s u ecoc .

.........................................................................................................................................

.........................................................................................................................................

.........................................................................................................................................

.........................................................................................................................................

...................................................................................................................................

10 (a) i . . s ows a simp e a er a i curre a.c. e era or w ic ca e use o


e era e e ec rici .

S N

Fig. 10.1

(i) ae e ame o e wo ri s s ow i i . . .

...........................................................................................................................

(ii) e rec a u ar coi is ro a e c oc wise as s ow i i . . . raw a arrow


i i . . o i ica e e irec io o e i uce curre i wire .

324
(b) armer co ec s a ouse o e mai s supp o e ec rici . e ouse is a a o
is a ce rom e eares mai s supp o e ec rici .
i . . s ows e mai s supp co ec e o e ouse.

o is a ce ca es

mai s
supp

Fig. 10.2

(i) e armer uses amps i e ouse u e o o i up a eir


orma ri ess. Exp ai w e amps are im.

................................................................................................................................

...........................................................................................................................

(ii) e armer a e ra s ormers as s ow i i . . .

mai s
supp
ra s ormer ra s ormer
A B

Fig. 10.3

e amps i e is a ouse i up a orma ri ess. Exp ai w e


amps are ow ri er.

................................................................................................................................

................................................................................................................................

................................................................................................................................

................................................................................................................................

...........................................................................................................................

325
(c) i . . s ows a ra s ormer wi coi s a w ic are wou o a iro core.

Fig. 10.4

oi as ur s a coi as ur s. e e.m. . i uce across coi


is . e ra s ormer opera es wi e icie c .

(i) a cu a e e o a eo e power supp pro i e o coi

o a e ...............................

(ii) Exp ai w a a er a i curre supp s ou e co ec e o coi i


or er or e ra s ormer o wor proper .

................................................................................................................................

................................................................................................................................

................................................................................................................................

................................................................................................................................

...............................................................................................................................

...............................................................................................................................

..........................................................................................................................

326
Either

11 s u e per orms a experime wi a semicircu ar ass oc a a ra o w i e i .


i . . s ows e pa a e is ra o i as i e ers e ass a P u i i i s e
s rai e ea .

semicircu ar
ass oc

w ie i

Fig. 11.1

e s u e i s a ere is o c a e i irec io as e ra e ers e ass a P a


a o i passes ou o e ass a . e ass oc as a re rac i e i ex o . .

(a) Exp ai w a is mea a re rac i e i ex o . .

....................................................................................................................................

...............................................................................................................................

(b) a cu a e e cri ica a eo e ass oc .

cri ica a e ...............................

(c) Exp ai w e i ra oes o c a e irec io a P.

....................................................................................................................................

...............................................................................................................................

327
(d) ea e θ is o
exp ai w a wi appe o e i ra a .

....................................................................................................................................

....................................................................................................................................

....................................................................................................................................

...............................................................................................................................

(e) esu e irec s e ra o i i o e ass a o i ere pa s re uci


e angle θ slowly.

escri e ec a es o e pa o i a .

....................................................................................................................................

....................................................................................................................................

....................................................................................................................................

...............................................................................................................................

(f) isi e i is par o e ec roma e ic spec rum.

isi e i as a re ue c o . x i acuum ca cu a e i s wa e e .

wa e e ...............................

328
OR

11 (a) i . . s ows a e ec rica circui . e e.m. . o e a er is . . a are


i e ica resis ors. e swi c is ope e amme er rea i is . a e
o me er rea i is . .

x
amp S

V
A

Fig. 11.2

(i) a e w a is mea e a er as a e.m.f. of 6.0 V.

................................................................................................................................

...........................................................................................................................

(ii) a cu a e e resis a ce o e amp.

resis a ce o amp ...............................

(iii) a cu a e e resis a ce o resis or .

resis a ce o ...............................

329
(iv) e swi c S is e c ose . ompare e ri ess o e amp ow wi e
ri ess o e amp w e e swi c S is ope . Exp ai our a swer.

................................................................................................................................

................................................................................................................................

................................................................................................................................

...........................................................................................................................

(b) i . . s ows a simp i ie ia ram o a e ec rica app ia ce co ec e wi i e


eu ra a Ear E wires.

e ec rica
app ia ce

Fig. 11.3

(i) a use is o e a e as a sa e e ice i e circui s ow i i . . raw


a “X” in Fig. 11. w ere e use s ou e p ace .

(ii) Exp ai our a swer or (b)(i).

................................................................................................................................

................................................................................................................................

................................................................................................................................

...........................................................................................................................

End of Paper

330
331
1 2 3 4 5 6 7 8 9 10

11 12 13 14 15 16 17
7 18
18 1
19 20

21 22 23 24 25 26 27
27 28 29 30

31 32 33 34 35 36 37 38 39 40

17

332
Physics Prelim P2 Answer Scheme

Section B
air resis a ce
a correc arrows a e

wei a air resis a ce

wei

rom . s o . s e is a ce ra e e ra
ra e e is
is . – . . m.
rom . s o . s a e e iis
e is a ce ra e e rra s . – . . m.

i ce e is a ce ra e e or e same perio o iime


me
m e . s i creases
crrea
ease
ses
se
e spee o e s u e coc i creases a . s.

c ermi a e oci c a e i is a ce ime


. – . . .
. ms

i e s u ecoc a s ue o i s weiwei
e .
s wei ac i ow war s is rearea er a e air resis a ce ac i upwar s.
ere is a ow war resu
u a orce.
orc
rce.
e.
us e s u ecoc ac acce era
cce e ra es
es owow war
wa s si ce a m w ere m is co s a .

ii s e spespee
ee o e s u ec oc i creases e air resis a ce ac i upwar s
ecoc
creases
i crease
sess u i i iss e ua o e wei ow war s o e s u ecoc .
ere is o reresu
esu a o orce
orc
ce ac i o e s u ecoc a us ere is o
acce eera
ra io
o or e s u ecoc si ce a m w ere m is co s a .
e e s u ecoc
e cce e reac es i s s ea spee .

e e s u ec
ecoc wi rea er mass as a rea er wei .
ee
e s rea er air resis a ce ac i a ai s i .
a es a o er ime or e air resis a ce o e e ua o e ew o a wei .
us a rea er ermi a spee is reac e .

333
a i ip ri s.

ii irec io o i uce curre is rom o .

i e o is a ce ca es as re a i e i resis a ce
ere is power e er oss or o a e rop i e ca e si ce P .

ii ra s ormer s eps up e o a e so e curre i e ra smissio ca e is ower.


is re uces e power e er oss i ca e si ce si ce P .
ra s ormer is e use o s ep ow e o a e o w ic wi cause e
amps o i up a orma ri ess.

c i s p s p
p
p .

ii . . is a curre a c a es ma i u e a irec io
o .
e ce e primar coi wi pro uce a c a i ma ma e ic ic ie
ie .
e iro core wi co ce ra e a i e ma e ic
ic ie
ie o coico
oii .
e ce ere wi e a c a e i ma e ic ie ccu u i ccoi oii a
o ere
ere wi ei uce
em curre i coi .

Ei er
a e rac i e i ex is e ra io o spee o i i acuum o e sspee
pee o i
pe i
me ium ass is . .

si c
. si c
. or

c e ra is pe
e sur ace perpe icu arr
icu i . e ce i wi o c a e irec io .

o a i er a re
re ec
ec io wi occur.
occ
ccur
ur.
is ecause
ecausse e ra iss ra
r e i rom a op ica e ser me ium o a ess e se
me ium a e a e o i ci e ce is rea er a cri ica a e.

e e i cri
cri ica
ica a e e ra wi ra e a o e ori o a e eo e ass
oc .
e i e ra wi re rac ou o e ass oc e i awa rom e orma .

x wa e e
. x . x x wa e e
a ee . x m

334
a i e wor o e e source i ri i a u i c ar e rou a comp
comp e e circui is .

ii esis a ce o amp
. .
. :

iii p. . across . – .
.

. .
. :

i e e swi c is c o
ose
e e o a rresis
is a cce o
esis
es a amp
ecreases e o es ass ’ .
ecreases

e p. . across e amp
amp ecreases
ecre
ec reas
a es si
si ce amp ’ ’ x . .
us e amp is imm immer
er as
mmer as i asa ower power P w ere is e
resis a ce o amp p a is
is co s a .

i “X
X” should
“X” sh be on tthe
be he p ositive terminal side of the cell.
positive

ii e e ccurre
urre
ur re excee s e use ra i e use wi me a ope e
circui
ci cui .
circ
wi
wi isco s o ec e circui rom e i
iisc o a ea e i e wire a pre e
o er eae i o e circui .

E o Paper

335
336
Pasir Ris Secondary School
N gi N

SECONDARY 4 EXPRESS

PRELIMINARY EXAMINATION 2018

PHYSICS 6091/01
Paper u ip e oice 12 September 2018

Wednesday 0800 0900 1 hour

i io a a eria s u ip e oice swer ee

READ THESE INSTRUCTIONS FIRST

ri e our ame c ass a re is er um er o a e wor ou a i .


ri e i so pe ci .
o o use s ap es paper c ips i i ers ue or correc io ui or ape.

ere are forty ues io s i is paper. swer all ues io s. or eac ues io ere are our possi e
a swers A B C a D.
oose e one ou co si er correc a recor our c oice i soft pencil o e separa e swer ee .

e use o a appro e scie i ic ca cu a or is expec e w ere appropria e.

Read the instructions on the Answer Sheet very carefully.


Eac correc a swer wi score o e mar . mar wi o e e uc e or a wro a swer.
rou wor i s ou e o ei is oo e .

is ocume co sis s o 14 pri e pa es i c u i is co er pa e.

e er Turn over

337
PRSS_2018_S4E_PRELIM_PHYSICS_6091_P1

. e ia ram s ows a er ier sca e.

cm

a is e er ier rea i

A . cm B . cm C . cm D . cm

. pe u um c oc ma es use o e osci a io o a pe u um o eep ime. e pe u um


c oc is ou o e ru i s ow w a ca e o e o correc e c oc

A ecrease e amp i u e o osci a io


B i crease e amp i u e o osci a io
C ecrease e e o e pe u um
D i crease e e o e pe u um

. car ra e s rom i apore o e i i a si a a sia. e map s ows e rou e.

e i
i a s

i apore

e rou e rom i apore o e i i a s is m. e car mo es a a a era e


spee o m w a is e ime a e or e our e

A . our
B . ours
C ess a . ours as e our e is o a s rai i e
D more a . ours as e our e is o a s rai i e

. e ia ram s ows a s rip o paper ape pu e u er a i ra i arm a ro e mo i a


co s a spee . e arm is i ra i re u ar ma i o s per seco .

cm

a is e spee o e ro e

A . cm s B . cm s C cm s D cm s

338
PRSS_2018_S4E_PRELIM_PHYSICS_6091_P1

. car acce era es u i orm rom ms o m s. uri is acce era io e car ra e s


m. a is e acce era io o e car

A . ms B . ms C . ms D . ms

. e rap s ows ow e e oci o a ump o p as ici e aries a er ei row er ica


upwar s i o e air rom e rou .
e oci ms

ime s

a is e ime ee e or e p as ici e o reac i s i es poi rom e rou

A s B s C s D s

. io ru s a a i spee o ca c i s pre . ere is ric io e wee e io a e air


a e wee e io a e rou . er w ic co i io s o ric io wi e io reac i s
rea es maximum spee

ric io wi air ric io wi rou


A i i
B i ow
C ow i
D ow ow

. pu i orce o . causes a o car o acce era e o a ori o a sur ace. e ric io a


orce e wee e sur ace a e o car is . . ic o e o owi s a eme s es
escri es e su se ue mo io o e car w e e pu i orce is ecrease o .

A wi co i ue o acce era e. B wi ece era e.


C wi mo e wi a co s a spee . D wi s op mo i .

339
PRSS_2018_S4E_PRELIM_PHYSICS_6091_P1

. c i umps o o a rampo i e a ou ces upwar s.

e seco ump e ou ces i er. a wi mos i e remai co s a o o


umps

A is acce era io i e air


B is maximum ra i a io a po e ia e er
C is maximum i e ic e er
D is spee o co ac wi e rampo i e

. e a e s ows e e si o arious su s a ces.

su s a ce e si cm
copper .
iro .
erose e .
mercur .
wa er .

ic s a eme is rue

A o iro as a sma er o ume a o copper.


B o mercur as a rea er o ume a o wa er.
C e mass o cm o mercur is rea er a cm o a e o er su s a ces.
D e mass o cm o wa er is sma er a cm o a e o er su s a ces.

. pe u um is a ac e o e roo o a ruc . e ruc is i i ia mo i a a co s a


spee a o a s rai roa .

irec io o mo eme o ruc

pe u um o

a is e mo io o e pe u um o w e e ruc s ar s o s ow ow

A wi remai i i s ori i a posi io .


B wi swi o e e .
C wi swi o e ri .
D wi swi o e ri a e .

340
PRSS_2018_S4E_PRELIM_PHYSICS_6091_P1

. ic o e o owi ua i ies epe so e s re o e ra i a io a ie

A e si B mass C o ume D wei

. e ia rams s ow ow ree oc s X Y a Z are a a ce o a u i orm eam.

X Y Y Z

X Z

ic o e o owi s ows e ree oc s arra e i or er o i creasi mass

A X Y Z B X Z Y C Y X Z D Z Y X

. piece o car as i s ce re o ra i a X.

A B

C D

ic ia ram s ows ow i a s w e suspe e a rea

. car i i ia mo i wi a e oci v as i e ic e er K. a is e i e ic e er o e
car w e e e oci is v

A K B K C K D K

. o pus es a o car a co s a spee a o a e e roa . a is e i es e er


c a e

A c emica po e ia o ea B c emica po e ia o i e ic
C i e ic o ra i a io a po e ia D i e ic o ea

341
PRSS_2018_S4E_PRELIM_PHYSICS_6091_P1

. e ia ram s ows a rau ic ra e s s em use i e ic es. rau ic i ui is use o


i e s s em a a o ow u e i s e mas er pis o o e s a e pis o . o pis o s are
c i rica a e iame er o e s a e pis o is wice a o e mas er pis o .

u e

e ri er presses ow o e pe a suc a a orce o is app ie o e mas er


pis o w a is e orce app ie e s a e pis o o e ra e pa

A B C D

. co um o air is rappe some mercur i a capi ar u e. e capi ar u e is e i


i ere posi io s as s ow i e ia ram.

P2
P1
mercur

P3

P1 P2 a P 3 are pressures o e e c ose air i e capi ar u e. ic re a io s ip is


correc

A P1 P2 P3 B P1 P2 P3 C P1 P3 P2 D P3 P2 P1

. ic o e o owi correc s a es e proper ies o so i s i ui s a ases

so i s i ui s ases

A oes o ow easi ow easi ow easi

B ar o compress easi compresse easi compresse

C ixe s ape ixe s ape o ixe s ape

D ixe o ume ixe o ume ixe o ume

342
PRSS_2018_S4E_PRELIM_PHYSICS_6091_P1

. umi a e smo e par ic es suspe e i air are iewe wi a microscope. e are see
o mo e ra om . ic o e o owi es exp ai s e mo io o e smo e par ic es

A air mo ecu es are ar apar rom eac o er


B air mo ecu es co i e wi eac o er
C air mo ecu es mo e ra om
D smo e par ic es mo e ra om

. acuum as eeps wa er o or o perio s o ime. ic o e o owi s a eme s


are correc

e p as ic cap pre e s ea oss ra ia io .


e si ere sur aces re uce ea oss ra ia io .
e acuum i e as pre e s ea oss co uc io a co ec io .

A a o B a o
C a o D a

. u mar e mercur ermome er is a ac e o a ru er as s ow i e ia ram.

cm

ermome er

ru er

mercur

e e ermome er is p ace i pure me i ice e mercur e e a s o . cm. e


e ermome er is p ace i s eam a o e oi i wa er e mercur e e rises o . cm.
a is e empera ure s ow e mercur e e i e ia ram

A q B q C q D q

343
PRSS_2018_S4E_PRELIM_PHYSICS_6091_P1

. e rap s ows ow e ermome ric proper o our su s a ces c a es wi


empera ure.

ermome ric proper

D
C

B
A

ice poi s eam poi empera ure

ic su s a ce ca e use o co s ruc a empera ure sca e

. e se up s ow i e ia ram is use o measure e speci ic a e ea o aporisa io


o wa er.

ea er
wa er

compressio
spri a a ce

ree rea i s are a e e compressio spri a a ce.

m1 mass o e se up a e s ar o e experime
m2 mass o e se up mi u es a er e wa er s ar s o oi
m3 mass o e se up mi u es a er e wa er s ar s o oi

e ea er as a power o w ic expressio cou e use o ca cu a e e speci ic


ae ea o aporisa io o wa er

A B C D
m1 m2 m2 m3 m1 m2 m1 m3

344
PRSS_2018_S4E_PRELIM_PHYSICS_6091_P1

. e ia ram represe s circu ar wa e ro s comi rom S. e wa e ro s are a ou o


s ri e a so i arrier rom w ic e wi e re ec e so as o appear o come rom S 1 .

S1 S

arrier

ic ia ram correc s ows e re ec e wa e ro s

S1 S

S1 S

S1 S

S1 S

345
PRSS_2018_S4E_PRELIM_PHYSICS_6091_P1

. our o es are oa i i a a e. e ia ram s ows e o es as see a pi o i a


o eri e icop er. e i es represe e cres s o e wa es i e a e.

xx x
x

ic wo o es are o e same wa e ro

A a B a C a D a

. e ia ram s ows ow isp aceme aries wi ime as a wa e passes a ixe poi .

a is e re ue c o is wa e

A . B . C . D .

. su e oo s i o a periscope.

B C D

ere wi e see e ima e o e o ec a O

346
PRSS_2018_S4E_PRELIM_PHYSICS_6091_P1

. ic o e ra s s ows e correc irec io o e emer e ra w e e i ci e ra


s ri es e ass prism

i ci e ra C

ass prism

. e ia ram s ows a e ri i i si e a i er e e ar co ec e o a acuum pump.

o a er

i er e e ar

o acuum
pump

a appe s o e pi c a ou ess o e sou w e e acuum pump is swi c e


o

pi c ou ess
A ecreases ecreases
B ecreases i creases
C remai s e same ecreases
D remai s e same i creases

. s u e aces a wa w ic is m awa a ires a s ar i pis o . is rie sa i


m e i im ears wo sou s. e spee o sou is m s w a is e ime
i er a e wee e wo sou s ear is rie

A . s B . s C . s D . s

347
PRSS_2018_S4E_PRELIM_PHYSICS_6091_P1

. e ia ram s ows e e ec roma e ic spec rum wi e ue a re e so e isi e


spec rum mar e .
ue re

A B isi e C D
i

ic sec io o e spec rum as wa es wi e o es wa e e

. ic o e o owi s a eme s a ou a e ec ric ie is correc

A is a ie a co ai s e ec ric c ar es.
B is a ie a surrou s e ec ric c ar es.
C is a re io w ere a me a experie ces a orce.
D is a re io w ere a e ec ric c ar e experie ces a orce.

. circui is use o i e curre I passi rou a ixe resis or or arious o a es V.

R
aria e irec
o a eV R

ic rap s ows ow e curre I aries wi o a eV

I I I I

V V V V

A B C D

348
PRSS_2018_S4E_PRELIM_PHYSICS_6091_P1

. e ia ram s ows a : resis or co ec e o a aria e resis or.

X x
S

Y x
:

a appe s o e po e ia i ere ce across e : resis or as e s i er S is mo e


rom X o Y

A ecomes ero. B ecreases.


C i creases. D remai s a .

. mai s e ec rica circui uses i su a e copper ca e. e ca e o er ea s. ic o e


o owi c a es wi pre e e ca e rom o er ea i

A use a ic er copper ca e as i as ess resis a ce


B use a ic er i su a io o re uce ea oss o e surrou i s
C use a i er copper ca e as i as ess resis a ce
D use a i er i su a io o re uce ea oss o e surrou i s

. e ia ram s ows a curre carr i co uc or p ace i e wee wo ma e s.

X Y

curre i co uc or

ic ma e ic po es a X a Y wi cause e co uc or o mo e i o e pa e o e
paper

X Y
A or or
B or sou
C sou or
D sou sou

349
PRSS_2018_S4E_PRELIM_PHYSICS_6091_P1

. a is e u c io o e so iro c i er p ace e wee e cur e po es o e ma e


i a .c. mo or

A o co ro e spee o ro a io o e coi
B o e a e e coi o ur i o e irec io o
C o i crease e orces ac i o e coi
D o i crease e ma i u e o e i uce e.m. . i e coi

. ar ma e is roppe rou a so e oi co ec e o a a a ome er.

so e oi

posi io posi io posi io

w ic posi io s wi e a a ome er s ow a e ec io

A o B a o C a o D a

. e ia ram s ows a ca o e ra osci oscope c.r.o. race or a o a e pro uce a


a.c. e era or. e ime ase o e c.r.o. is Ps i .

e coi i e a.c. e era or is ro a e a ree imes i s curre spee w a is e perio


o e ew o a e

A Ps B Ps C Ps D Ps

End of Paper

350
Pasir Ris Secondary School
N gi N

SECONDARY 4 EXPRESS

PRELIMINARY EXAMINATION 2018

PHYSICS 6091/02
Paper eor 11 September 2018

Tuesday 0800 - 0945 1 hour 45 minutes

a i a es a swer o e ues io Paper.


o a i io a ma eria s are re uire .

READ THESE INSTRUCTIONS FIRST


ri e our ame c ass a re is er um er o a e wor ou a i .
ri e i ar ue or ac pe . ou ma use a so pe ci or a ia rams or rap s.
o o use s ap es paper c ips i i ers ue or correc io ui or ape.

Section A 50 marks
swer all ues io s.

Section B 30 marks
swer all ues io s. ues io as a c oice o par s o a swer.

a i a es are remi e a all ua i a i e a swers s ou i c u e appropria e u i s.


a i a es are a ise o s ow a eir wor i i a c ear a or er ma er as more mar s are awar e
or sou use o P sics a or correc a swers.
e use o a appro e scie i ic ca cu a or is expec e w ere appropria e.

ee o e exami a io as e a our wor secure o e er.


e um er o mar s is i e i rac e s a ee o eac ues io or par ues io .

or Exami er s se

ec io

ec io

oa

is ocume co sis s o 17 pri e pa es i c u i is co er pa e.

e er Turn over

351
PRSS_2018_S4E_PRELIM_PHYSICS_6091_P2

Section A (50 marks)


swer a e ues io s i e spaces pro i e .

. sui case s i es ow a co e er e rom a aerop a e as s ow i i . . . e


sui case s ar s rom res a A w ic is a e op o e e a s i es ow e e u i i
reac es B w ic is . m e ow A. e sui case e ece era es as i mo es a o e
ori o a oor.

aerop a e

sui case
A

. m co e or e

oor B C

Fig. 1.1

e mass o e sui case is . e sui case is mo i a a ori o a spee o . ms


a B. a e ra i a io a ie s re .

a escri e e e er c a es a a e p ace as e sui case mo es rom A o B a


rom B o C i i . . .

a cu a e e e icie c o e sui case w e i mo es rom A o B.

352
PRSS_2018_S4E_PRELIM_PHYSICS_6091_P2

c e sui case comes o a res a C ree seco s a er passi B. e ermi e e


a era e re ar i orce exer e e oor o e sui case.

. i . . s ows a s i er a i er ica rom a aircra .

Fig. 2.1

a i . . raw a a e e orces experie ce es i er as e a s.

i . . s ows e e oci ime rap or es i er uri e irs so is ump.

e oci ms

ime s
Fig. 2.2

e s i er a s rom res a ime s a i i ia acce era es a ms . e


reac es a s ea e oci a er s. ime s e ope s is parac u e.

353
PRSS_2018_S4E_PRELIM_PHYSICS_6091_P2

i Exp ai e mo io o es i er rom o s.

ii i . . s ec e acce era io ime rap or es i er rom ime s o


ime s.

acce era io ms

ime s
Fig. 2.3

iii Exp ai w es i er ece era es a er e ope s is parac u e a ime s.

. i . . s ows e appara us a cou e use o ea up wa er i a po . e ex erior


wa o e po is co ere wi a a er o po s re e oam.

i ci e
so ar ra ia io

po

ermome er

po s re e oam wa er

Fig. 3.1

354
PRSS_2018_S4E_PRELIM_PHYSICS_6091_P2

s u e pu s e po i irec su i a measure e ime i oo or e empera ure


o e wa er o i crease. e e er ra s erre o e wa er is a e ime a e
or e empera ure o e wa er o i crease rom . q o . q is mi u es. e
empera ure o e air ou si e e po is . q .

a a cu a e e power supp ie e u o e wa er i e po .

i er a ea exc a e wi e surrou i s su es a o er reaso w e


su e s resu s ca o e use as a es ima e o e power o e u .

ii a cu a e e mass o e wa er i si e e po . e speci ic ea capaci o wa er


is q .

c escri e ow e po s re e oam i su a es e po .

Exp ai w e wa er reac es a s ea empera ure a er some ime.

355
PRSS_2018_S4E_PRELIM_PHYSICS_6091_P2

. i . . o raw o sca e s ows par o e pa o a ra o i PQR ra e i rou


a op ica i re. e op ica i re co sis s o a i re o e ser ra spare ma eria coa e
wi a a er o ess e se ra spare ma eria . e e si e o e i re is ori o a w i e
e ri si e o e i re is e .

op ica ess op ica e ser


e se ma eria ma eria

R
T
Q q
q

P
Fig. 4.1

a omp e e e pa o e ra o i u i i reac es ee o e op ica i re.

e re rac i e i ex o e op ica ess e se ma eria is assume o e . .

i e ermi e e mi imum a e T ee e or o a i er a re ec io o a e p ace.

ii Exp ai w i is ecessar or e e ser ma eria o a e a re rac i e i ex muc


i er a a o e ess e se ma eria .

c er a e cos i curre s a e o e a a a e o usi op ica i res ra er a


copper wires or ra smissio o i orma io .

356
PRSS_2018_S4E_PRELIM_PHYSICS_6091_P2

. i . . s ows a o ec AB ear a i co er i e s. e pa o o e ra rom poi A


passi rou e e s is raw .

e s

Fig. 5.1

a i . . raw a sui a e ra rom poi B passi rou e e s o oca e e


posi io o e ima e o o ec AB. a e e ima e A’B’.

rawi a o er ra rom poi B passi rou e e s mar e oca e o


e e s. a e i f.

c e co er i e s ma e use o orm a ir ua ima e o o ec AB. a e w ere


e o ec is p ace re a i e o e e s or a ir ua ima e o e orme .

. i . . s ows a ar e me a sp ere X suppor e o a i su a i sa . p ere X is


co ec e o ear rou swi c S. posi i e c ar e sp ere Y is rou er c ose
o sp ere X a is e i p ace wo s ri s A a B.

Y
B
+
S
X

i su a i sa

Fig. 6.1

wi c S is e c ose .

a i i . . raw e c ar e is ri u io o sp ere X.

357
PRSS_2018_S4E_PRELIM_PHYSICS_6091_P2

ii Exp ai our a swer o a i .

ri B is ow cu wi swi c S remai i c ose .

i escri e a exp ai e su se ue mo io o Y u i i comes o a res .

ii i . . raw e i a posi io o res o Y.

Fig. 6.2

. i . . s ows a e ec ric circui . E is a . ce a T is a ermis or w ose aria io o


resis a ce wi empera ure is s ow i i . . .

x
x
S T
x
E
. x

Fig. 7.1

358
PRSS_2018_S4E_PRELIM_PHYSICS_6091_P2

resis a ce

empera ure
Fig. 7.2

a e e ermis or is a q a swi c S is c ose e rea i o e amme er is


m . e ermi e e resis a ce o resis or R.

e empera ure o e ermis or i creases rom q o q .

i a cu a e e rea i o e amme er a q .

ii Exp ai e c a e i e po e ia i ere ce across T i a w e e


empera ure o e ermis or i creases.

359
PRSS_2018_S4E_PRELIM_PHYSICS_6091_P2

. i . . s ows ow power ca es are use o ra smi e ec rica e er rom a power


s a io o e co sumers i a i a e. e power s a io pro uces e ec rici a a
a er a i o a e o . e ou pu o a e o ra s ormer is w ie e
o a e supp ie o e co sumers is . ssume a ra s ormers a K are i ea .

ra s ormer power ca es ra s ormer K

co sumers

power s a io

Fig. 8.1

a Exp ai w ra s ormer is o a e o ra s orm e o a e o a irec curre .

a cu a e e um er o ur s i e primar coi o ra s ormer i ere are


ur s i e seco ar coi .

c Exp ai w e o a e o power s a io was s eppe up rom o e ore


ei ra smi e o er e power ca es.

360
PRSS_2018_S4E_PRELIM_PHYSICS_6091_P2

Section B (30 marks)


swer a e ues io s i e spaces pro i e .
ues io as a c oice o par s o a swer.

. s u e i es i a e ow curre aries wi po e ia i ere ce or wo i ere amps X


a Y. i . . s ows e rea i s o ai e .

po e ia i ere ce curre i amp X curre i amp Y

. . .

. . .

. . .

. . .

. . .

. . .

. . .

. . .

. . .

Fig. 9.1

a e space e ow raw e ia ram o a circui a esu e cou use o o ai


e resu s i i . . . e curre i amp X mus e o ai e a e same ime as e
curre i amp Y.

a e e a ues o po e ia i ere ce or w ic both amp X a amp Y e a e i e


o mic co uc ors. Exp ai our a swer.

361
PRSS_2018_S4E_PRELIM_PHYSICS_6091_P2

c si i . . exp ai w ic amp as a i er resis a ce.

si our a swer rom c exp ai w ic amp is ri er a a po e ia i ere ce.

e e s u e repea s is i es i a io wi a s ro wi owi o er e amps.


i . . s ows e rea i s o ai e a a po e ia i ere ce o . .

po e ia i ere ce curre i amp X curre i amp Y

. . .

Fig. 9.2

Exp ai w e a ues o curre i i . . are i ere rom a i i . . or e


same po e ia i ere ce across e amps.

362
PRSS_2018_S4E_PRELIM_PHYSICS_6091_P2

. i . . s ows a ou spea er ma e asu e . coi o wire is ixe o o e e o e


paper co e. e e swi c is c ose e paper co e mo es o e ri .

c am
ru er a
.c.
supp

coi o wire

ma e

paper co e ru er
ru er a

c am

Fig. 10.1

a Exp ai w e paper co e mo es o e ri w e e swi c is c ose .

e s u e i es i a es ow c a i e si e o e curre i e coi o wire a ec s


e ori o a is a ce mo e e paper co e. e resu s o e su e s
i es i a io are s ow i i . . .

ori o a
is a ce
mo e
paper
co e cm

curre

Fig. 10.2

363
PRSS_2018_S4E_PRELIM_PHYSICS_6091_P2

aea exp ai e resu s o esu e si es i a io w e e curre i creases


rom . o . .

c ou is pro uce w e ere is a a er a i curre i e coi .

i Exp ai ow sou is pro uce w e ere is a a er a i curre i e coi .

ii escri e ow e par ic es o e co e pro i es a examp e o o i u i a wa e


mo io .

iii Exp ai e i ere ce i e sou ear w e e si e o e a er a i curre


is i crease .

364
PRSS_2018_S4E_PRELIM_PHYSICS_6091_P2

EITHER

. i . . s ows a ma ome er a ac e o a re o measure e pressure i ere ce


e wee e air i si e e re a e a mosp ere ou si e. e a mosp eric pressure
ou si e e re is Pa.

ma ome er

re h me re ru e

air

rou
a e
wa er

Fig. 11.1

a i ar o e ma ome er a poi P w ere e pressure is cm o wa er i er


a a mosp eric pressure.

ii a cu a e e pressure o e air i si e e re. e e si o wa er is


m a e ra i a io a ie s re .

iii ae w a c a e i a wi occur o e ei h i a i ui e ser a wa er


is use i e ma ome er.

escri e e mo io o air mo ecu es i si e e re a exp ai ow is resu s i a


pressure exer e air o e wa s o e re.

365
PRSS_2018_S4E_PRELIM_PHYSICS_6091_P2

c e re oes o er a s o e as s ow i i . . .

re

air

rou
so e
Fig. 11.2

aea exp ai

i ow e pressure exer e o e rou c a es.

ii ow e pressure i si e e re c a es.

OR

. e wei o a roc w ic is more a ca o e measure irec usi a spri


a a ce wi a maximum rea i o . i . . s ows a se up a cou e use o
e ermi e e wei o e roc .

spri a a ce
roc
pa

. cm . cm

rou

Fig. 11.3

wei ess p a is pi o e wi e roc p ace . cm rom e pi o . e p a is


suspe e a spri a a ce a a poi . cm rom e roc . e pa a a ces
ori o a a e rea i o e spri a a ce is . .

366
PRSS_2018_S4E_PRELIM_PHYSICS_6091_P2

a i ae e principle of moments.

ii Exp ai w is se up ca e a e e wei o e roc o e e ermi e .

iii a cu a e e wei o e roc .

e roc a s o e pa a i s e rou wi a spee o . m s. e er ica


is a ce o e p a rom e rou is . m.

i a cu a e e ime a e or e roc o i e rou . a e wo assump io s a


ou a e ma e uri our ca cu a io s.

assump io

assump io

ii a cu a e e wor o eo e roc e orce o ra i .

End of Paper

367
368
PRSS_2018_S4E_PRELIM_PHYSICS_6091_P1
PRSS_2018_S
S4E_PRE
S4E
S4 ELIM_PHYSICS_6091
ELI
EL

Answers to PRSS Prelim 2018 4E Pure Physics Paper 1

1 11 21 31

2 12 22 32

3 13 23 33
3 3

4 14 24 34

5 15 25 3
35

6 16 26 36
6

7 17 2
27 37

8 18 28 38

9 19 29 39

10
0 20
0 30 40

369
PRSS_2018_S4E_PRELIM_PHYSICS_6091_P2

Answers to PRSS 4E Physics Prelim 2018

Section A

. a rom A o B ra i a io a po e ia e er o e sui case is co er e o i e ic e er


a erma a sou e er .
rom B o C e i e ic e er o e sui case is co er e o erma a sou
e er .

E icie c use u e er ou pu i pu e er x
KE a B PE a A x
x x . x x x

c cce era io o sui case a u


.
. ms

ma
re ar i orce x . E
e ar i orce . E

e oci
oci
ci ms
ms

or o e re ar i orce oss i KE
x x x . x x .
.

iim
ime s

. a

air resis a ce

wei

i s e a s is e oci i creases a is causes e air resis a ce o


i crease .
e ow war resu a orce ecreases e ce is acce era io is
ecreases .

370
PRSS_2018_S4E_PRELIM_PHYSICS_6091_P2

ii

s rai i e or cur e wi e a i e ra ie
rom o .
No marks if the starting value of acceleration is
not 10 m/s2

s rai ori o a i e s owi a


acce era io is ero rom o s.

iii er e s i er ope s e parac u e e resis i e orce or airr resis


res is a ce ac i o
esis
im ecomes rea er a is wei .
esu a orce o im ecomes e a i e resu a orc orce
rce
rc e ac s i a irec
iire
recc io o
re opposi
pposi e
upwar s o is mo io a e ce e ece era es.

. a Power P E
x
.

i ome o e e er rom e u is use o ea


a e po ass we as e wa er .

e i e si o so ar ra ia io o e appara us c a e uri e mi u es.

e ermome er is ouc i e a
ase o e po a is o exac measuri e
emper
ra ure o e wa er
empera

ou s oc su ra s rom
m rreac
eacc i
ea e po .

e is a ce o rom
e po rom e u is ar e a some o e u s e er is os
rea
eac
e ore reacc i er.
e wa er.
o e

ii mc'T
mc 'T
mx x . .
m .
. or

c e air rappe i po s re e is a poor co uc or o erma e er .


o ec io is res ric e as air is rappe .
e ce ea ra s er co uc io a co ec io is re uce .

e ra e a w ic e wa er ai s ea rom e u is e ua o e ra e o ea oss o
e surrou i s.

ere is o e ea ra s er e wee e wa er a e surrou i s.


o e

371
PRSS_2018_S4E_PRELIM_PHYSICS_6091_P2

. a

orrec raws a ra o i w ere e a e o re ec io


is e ua s o e a e o i ci e ce a R.

i e rac i e i ex si i si r
si q si q
.

si c
. E
c . q E

i imum a eT . q

ii is wi e sure a sma cri ica a e.


e easi
o a i er a re ec io wi a e p ace more easi
a si
as morre
mo
moree ra
ra s ca u er o o a
er
i er a re ec io .

c o
p ica i res ca carr more i orma io coppe
per wires.
a copper

ra smissio o i orma io is as er.

ere wi e ower e er oss si a oss


o s uri
os ra ssmi
m ssio .
smissio

orma io ca e ra smi e o e
err a o ce.
er is a ce. o e

. a

a
B’
a

f A’

raws a ra rom B passi rou e op ica ce re o e e s.


raws a a e e ima e A’B’ correc .
raws a ra para e o pri cipa axis a passi rou e e s o reac
poi B’.
ica es a a e e oca e f correc .

c e o ec mus e p ace ess a e oca e rom e e s.

372
PRSS_2018_S4E_PRELIM_PHYSICS_6091_P2

. a i
raws e a i e c ar es o e e si e o X
a o posi i e c ar es o e ri si e.

ii e posi i e c ar es i Y a rac s e ec ro s rom ear as u i e c ar es


a rac .
is i uces a e a i e c ar e o e e si e o X.

i Y mo es owar s X as u i e c ar es a rac .
e Y ouc es X Y is isc ar e eu ra ise e ear .
Y e mo es awa rom X ue o ra i a io a orce a osci a es
e ore comi o a res .

ii

ws Y ei
raws suspe e er ica s rii A.
A.

. a esis a ce o e w o e circui
. x

esis a ce o ermis or a

esis a ce o resis or R
E

esis a ce o ermis
ermis or
rmis or a

P across
ss ermis
rmiss or
erm
e or
x x
.

P across resis orr . .


.

esis a ce o resis or R . x

i esis a ce o T a

urre o amme er a
.
m E

373
PRSS_2018_S4E_PRELIM_PHYSICS_6091_P2

ii e empera ure i creases resis a ce o ermis or ecreases .


is ecreases e ra io o resis a ce o T o e o a resis a ce o ew oe
circui .

is ecreases e ra io o resis a ce T o e resis a ce o R.


e po e ia i ere ce across T ecreases.

. a irec curre oes o pro uce a co i uous c a i ma e ic ie i e


primar coi o ra s ormer .
is wi o resu i e ec roma e ic i uc io or o em wi e i uce i e
seco ar coi .

p s p s
p
p ccep

c eppi up e o a e wi re uce e ra smissio curre .


s power oss P is wi re uce power oss as ea i e ca es.

Section B

. a

amps i para
paara e wi o me
me er co ec e i para e wi e amps .
amme er o eac ac ra ra c o measure e curre rou amps.
a e rresis
aria is or
esis
es o or po po e iome er is use o ar e po e ia i ere ce across
e amps.
amp ps.
Deduct ½ mark
mar battery
a k if b attery / power source missing
g

a ues o po e ia i ere ce is rom . o . .


e ra io o po e ia i ere ce o curre is co s a curre is propor io a po e ia
i ere ce s owi a resis a ce is co s a .

c amp Y as a ower curre a amp X or a a ue o po e ia i ere ce .


amp Y as a i er resis a ce a amp X .
No marks for a simple answer e.g. ‘the resistance of Y is higher’.

s power ou pu P X as a i er power ou pu as i s resis a ce is


ower . e ce X is ri er .
No marks for a simple answer e.g. ‘X is brighter’.

374
PRSS_2018_S4E_PRELIM_PHYSICS_6091_P2

e i remo es ea rom e amps a owers eir empera ure.


s empera ure owers e resis a ce o e amps ecreases a e ce curre
is i er .

. a e swi c is c ose curre ows rou e coi a pro uces a ma e ic


ie i e coi
e ri si e o e coi is ma e ise wi a sou po ari . e ma e a rac s
e coi as u i e po es a rac causi e co e o mo e o e ri .

e ori o a is a ce mo e e co e i creases o i ear wi


curre .
e s re o ma e ic ie i e coi i creases wi curre or e coi ecomes a
s ro er e ec roma e a resu s i a rea er a rac i e orce e wee e coi
a e ma e .

c i a er a i curre causes e coi o e co i uous a rac e a repe e


e ma e .
is causes e co e o i ra e a a sou wa e iss pro
pro uc
u
ucee .

ii e sou pro uce ra e i a irec io para e o e i ra


ra io
io o pa ic es o
e par
e co e.

iii e sou ear is ou er.


s curre i creases e amp i u e o i ra io o e co e i crea
ase
ses.
s.
creases.

EITHER
. a i Poi P mar e a cm mar o e ma ome e
err u e.

ii Pressure o air mosp


sp e
eri
ricc pr
eric p essu
sure
pressurere
e U
x . x
Pa
Pa . x Pa

iii h ecreases.

e air mo e cu es
ecu e are
a e mo
ar o i rara omo co i uous a i spee s .
e e air
ir m
moo ecu
u es
e i ew wa
a s o e re a orce is exer e a is si ce
pressure is orce per
per u i area
are
ea a pressure is exer e o e wa s.

c i ere
ere is ess
e s sur
es sur ace
a e area i co ac wi
ac e rou . e ce pressure
creases
i creas
ases
es .

ii e air i si e e re is compresse or o ume o air i si e e re


ecreases . e um er o air mo ecu es per u i o ume i si e e re
i creases a is i creases e re ue c o co isio e wee e air
mo ecu es a e re . Pressure i si e e re i creases .

OR
. a i or a o ec i e ui i rium e sum o c oc wise mome a ou a pi o is e ua
o e sum o a ic oc wise mome a ou e same pi o .

ii e c oc wise mome ue o spri a a ce is e ua o e a ic oc wise


mome ue o roc a ou pi o .

375
PRSS_2018_S4E_PRELIM_PHYSICS_6091_P2
PRSS_2018_S4E_PRELIM_PHYSICS_6091_P

s mome orce x perpe icu ar is a ce a ar er perp


perpe
pe icu
rp icu ar is a ce
o spri a a ce rom pi o e a es a rea i ess a sm er
sma
sm er rea i
o e recor e o e spri a a ce.

iii oc wise mome a ou pi o mome


ic oc wise mome u pi
a ou pi o
. x . x .
.
. or
or s

e oci ms
is a ce ra e e rea u err e rap
p
. . x . x
. s or s

ime s

ssump io e roc a s ro rom


m rees
res e i i ia e oci o e roc is m s.
ssump io e roc is experie
exxpe
peri
r e ci
ri ree a acce era es co s a ere is o
air re
ai resi
s s a ce a
resis acci o e roc .

c or o eo e rocc x
. x . E
. orr s E

376
377
ass e is er o.

a i a e ame

PEIRCE SECONDARY SCHOOL


PRELIMINARY EXAMINATION 2018
SECONDARY 4 EXPRESS

PHYSICS 6091 / 01
Paper u ip e oice 18 September 2018
1 hour
i io a a eria s
u ip e oice swer ee

INSTRUCTIONS TO CANDIDATES

ri e i so pe ci .
o o use paper c ips i i ers ue or correc io ui .
ri e our ame c ass a re is er um er o e spaces pro i e a o e a o e
u ip e oice swer ee .

ere are forty ues io s o is paper. swer all ues io s. or eac ues io ere are
our possi e a swers A B C a D.
oose e one ou co si er correc a recor our c oice i soft pencil o e separa e
u ip e oice swer ee .

Read the instructions on the Multiple Choice Answer Sheet very carefully.

Eac correc a swer wi score o e mar . mar wi o e e uc e or a wro a swer.


rou wor i s ou e o ei is paper.

is paper co sis s o 17 pri e pa es a 1 a pa e.


e er rs su K

Turn over

378
1 ic o e ua i is e ui a e o o e me re

A . x mm
B . x m
C . x m
D . x m

2 ic is a sca ar ua i

A wei
B e ec ric ie
C mome o a orce
D e ec romo i e orce

3 e ia ram s ows e e oci ime rap or e mo io o a o .


e oci m s

ime s

a is i s isp aceme i e irs seco s

A . m B m C m D m

4 car o mass is mo i a a co s a spee o m s.


a is e a era e ra i orce ee e or i o come o a s op i m

A B C D

Turn over

379
5 ma is s a i o a wei i sca e i si e a i . e wei i sca e rea s
w e e i is s a io ar .

ic op io escri es correc e rea i o e wei i sca e w e e i


acce era es upwar s a w e e i acce era es ow war s

i acce era es upwar s i acce era es ow war s


A more a ess a
B ess a more a
C ess a
D more a

6 ma as o pus a ox wei i up a ramp rom e rou o e raise


p a orm.
ramp
ox

e ermi e e mi imum orce re uire .

A
B
C
D

Turn over

380
7 wo me a oc s a are a i rom e spri a a ces a e same oca io
i e a ora or as s ow e ow.

ic s a eme is rue a ou a

A e a e same mass u i ere wei .


B e a e same wei u i ere e si .
C e a e i ere o ume a i ere wei .
D e a e i ere e si a i ere mass.

8 ic orce A B C or D wou e e largest orce re uire o e exer e i or er o


pus e a up e s ep
C

B D

A a

s ep

9 o ec o mass is row er ica upwar s. e i e ic e er o e o ec


w e i is a a ei o . m a o e e rou is . .
ssumi e i i e air resis a ce w a is maximum ei reac e e o ec
e ra i a io a ie s re is .

A . m B . m C . m D . m

Turn over

381
10 ea ai is ixe irm o a wa . is pu e a s ri a o
o e er ica .
e ai oes o mo e.

ree orces ac o e ai .
i s wei W
e e sio T i e s ri
e reac io orce R e wee e ai a e wa

ic ia ram raw o sca e represe s e ree orces i si e a irec io

Turn over

382
11 e arra eme s ow i e ia ram a perso o mass is s a i o a
pis o o area cm .

pis o o area cm

wa er

a cu a e e ei h o e co um o wa er.
e ra i a io a ie s re is a e e si o wa er is m.

A . m B . m C . m D . m

12 e i e po e rai s suspe e i wa er are iewe u er a microscope e are


see o e ma i sma ra om mo eme s.
ic se e ce exp ai s is o ser a io

A ere are co ec io curre s i e wa er.


B e po e rai s are ei i wa er mo ecu es.
C e po e rai s are mo i a co i i wi o e a o er.
D e po e rai s are i i or a isms so e mo e arou .

13 e pressure o a as i a c i er is e same a a poi s i ec i er.


ic s a eme exp ai s is

A e mo ecu es o e as are a o e same si e.


B e mo ecu es o e as a rac o e a o er.
C e mo ecu es o e as mo e a i ere spee s.
D ere are ma mo ecu es a mo i a ra om.

Turn over

383
14 e ia ram s ows a as o co oure i ui . arrow u e passes rou e
s opper. e e as is e i pure me i ice e i ui co um measure
mm. e e as is e i oi i wa er e i ui co um measure mm.
e e as is e i oi a a co s a empera ure e e o e i ui co um
ecame mm.

a is e empera ure o e oi

A o
B o
C o
D o

15 umi ium oi s are commo use o wrap swee po a oes w ic are o e coo e i
a ar ecue ire as s ow i e ia ram.

umi ium oi wee po a o

si e u si e

ow s ou e swee po a oes e wrappe i e a umi ium oi as o e s i si e


a o e u si e

A es i si e s ou eo e ou si e ecause i is a e er emi er o ra ia io .
B es i si e s ou eo e ou si e ecause i is a e er co uc or o ea .
C e u si e s ou eo e ou si e ecause i is a e er a sor er o ra ia io .
D e u si e s ou eo e ou si e ecause i is a e er co uc or o ea .

Turn over

384
16 ir is ow i o e er rou e ass u e as s ow i e ia ram.
er some ime i is o ser e a e i m o wa er ree es i o ice.

ow air

ass u e

a i m o wa er e er

woo e oar

ic op io best escri es e processes a resu s rom owi e air

ra e o e apora io o e er empera ure o e er ea ra s er


A i creases rises rom wa er o e er
B i creases a s rom wa er o e er
C ecreases a s rom e er o wa er
D ecreases rises rom e er o wa er

17 e ra io o e masses o wo me a oc s G a H is . e are o ea e
u i orm usi i e ica ea ers. e empera ure ime rap s o e oc s are
s ow e ow.

empera ure o

G H

ime s

a is e ra io o e speci ic ea capaci ies o G a H

A B C D

Turn over

385
18 i ra or is p ace a e ce re o a i c i e ripp e a as s ow .

ripp e a

ow oes e wa e e o e wa er wa e c a e w e i is mo i owar s e
wo e s P a Q o e ripp e a

owar s P owar s Q
A ecrease ecrease
B ecrease i crease
C i crease ecrease
D oc a e oc a e

19 e ia ram s ows e posi io o air par ic es a a par icu ar i s a ce w e a sou


wa e is passi .

e wa e e is e is a ce e wee

A a
B a
C a
D a

20 e ia ram s ows i ere re io s o e e ec roma e ic spec rum wi some o e


re io s i e i ie .

ic re io co ai s wa es a ca e use o e ec cou er ei o es

Turn over

386
21 ree s u e s s a m apar i ro o a p a e mirror a is m o .

m
irror

m m
u e X u e Y u e Z

u e Xsa si i e wi o ee eo e mirror as s ow a o e.
ow ma s u e s ca see e ima es o e o er wo

A B C D

22 e ia ram s ows e pa o a ra o i as i s ri es e wa er o air ou ar .

a is e spee o i i wa er

A . x m s
B . x m s
C . x m s
D . x m s

23 uri a u ers orm a o o i i se s ou a e ec ric c ar e o rom a


u erc ou o e Ear . e e er pro uce e i i is a ou
e ermi e e po e ia i ere ce e wee e u erc ou a e Ear .

A B C D

Turn over

387
24 iso a e co uc i sp ere P as a c ar e is ri u io s ow e ow.

simi ar sp ere co ec e o Ear a o wire is rou c ose o P.


ic ia ram s ows e i a is ri u io o c ar e o e wo sp eres

A B P
P

C P D P

25 ic rap s ows e I / V c arac eris ic or a semico uc or io e

A B C D

26 e ia ram e ow s ows a circui .

:
PP Q
Q
: :

a is e e ec i e resis a ce e wee ermi a s P a

A B C D

Turn over

388
27 e ec ric ea er as wo ea i coi s wi i e ica resis a ces. e ca e
co ec e i ree i ere wa s as s ow i circui s a e ow.

ic o e o owi correc ra s e circui s rom e owes o e i es


ase o e power o e circui

owes power me ium power i es power


A
B
C
D

28 e ec ric iro mar e is co ec e oa mai s supp .


a is e power issipa e e e ec ric iro

A B C D

29 i epe e resis or a a ermis or are co ec e i series wi a


a er .

ic co i io s cause e po e ia i ere ce across e o e e sma es

A ri a co
B ri a o
C ar a co
D ar a o

Turn over

389
30 e cos o a u i o e ec rici is ce s.

app ia ce power ra i ime use


amp ours
ea er . ours
coo er mi u es

a is e o a cos w e a ese app ia ces are use i e imes s ow a o e

A . ce s B ce s C ce s D ce s

31 ic s a eme es escri es a examp e o i uce ma e ism

A ar ma e a rac s a piece o so iro .


B ar ma e oses i s ma e ism i i is repea e roppe .
C ar ma e swi i ree comes o res poi i i e or ou irec io .
D wo or po es repe eac o er u a or po e a rac s a ou po e.

32 e ma e ic ie i es o wo ar ma e s are s ow e ow. e irec io o one o


e ie i es is a so s ow .

ic ia ram represe s e correc arra eme o e ma es

Turn over

390
33 e ia ram e ow s ows a circui wi a wire co ec e o a a er a e swi c
. e compasses a are p ace a o e e wire a e compass is p ace
e ow e wire.

e swi c is c ose w ic ia ram correc s ows e orie a io s o e


compass ee es

A B

C D

34 e ia ram s ows a mo e circui rea er. e curre owi i e circui is


excessi e e e ec roma e a rac s o rea e circui .

ic s a eme a ou e circui rea er is correc

A is es ma e o copper.
B e circui rea er is sui a e or circui carr i a a er a i curre .
C e imi i curre i e circui i creases i e curre i e circui is re erse .
D e imi i curre i e circui wi o c a e p aci a so iro core i si e
e coi .

Turn over

391
35 wo para e er ica wires eac carr a upwar curre .
ic ia ram s ows e ma e ic ie pa er arou e wires a e irec io o
e orce F o eac wire

36 e ia ram s ows a pi o e coi e e wee e wo po es o a ma e.


e pi o e coi carries a s ea curre i e irec io s ow .

e e coi is re ease i ro a es a e s ops a a a e o i s i i ia posi io .


e iewe as s ow i w ic irec io oes e coi ro a e a w a is e a ue
o

irec io
A a ic oc wise
B a ic oc wise
C c oc wise
D c oc wise

Turn over

392
37 s u e pus es e po e o a ar ma e i o e e o a o so e oi a
o ser es a e ec io o e ri o e se si i e amme er.

ic process wi pro uce a e ec io i e same irec io

A pu i e po e ou o e
B pu i e po e ou o e P
C pus i e po e i o e P
D pus i e po e i o e P

38 ma e mo es up a ow a o e a coi o wire.
e o om o e ma e mo es up a ow e wee P a .

ere is e o om o e ma e w e ere is o i uce e ec romo i e orce i e


coi

A a o
B a o
C a Pa
D a Pa

Turn over

393
39 e um er o ur s e wee eac pair o ou pu ermi a s o a ra s ormer is s ow
i e ia ram.

e wee w ic wo ermi a s wi e ou pu e

A Pa
B a
C a
D Pa

40 e ia ram s ows a o ra smissio i e supp i e er a o wo ouses


a wi ou usi ra s ormers. o ouses e ec ric ea ers are swi c e o .

e occupier o ouse swi c es o e ea er i is ouse.


a appe s i ouse

e o a e supp ie o ouse e power supp ie o ouse


A ecreases ecreases
B ecreases sa s e same
C i creases i creases
D i creases sa s e same

End of Paper

Turn over

394
ass e is er o.

a i a e ame

PEIRCE SECONDARY SCHOOL


PRELIMINARY EXAMINATION 2018
SECONDARY 4 EXPRESS
PHYSICS 6091 / 02
Paper eor 11 September 2018
1 hour 45 minutes
a i a es a swer o e ues io Paper.
o a i io a ma eria s are re uire .

INSTRUCTIONS TO CANDIDATES
ri e our ame c ass a re is er um er o a e wor ou a i .
ri e i ar ue or ac pe i e spaces pro i e o e ues io Paper.
ou ma use a pe ci or a ia rams a rap s.
o o use paper c ips i i er ue or correc io ui .

Section A.
swer all ues io s.

Section B
swer all ues io s. ues io as a c oice o par s o a swer.

a i a es are remi e a all ua i a i e a swers s ou i c u e appropria e u i s.


e use o a appro e scie i ic ca cu a or is expec e w ere appropria e.
a i a es are a ise o s ow a eir wor i i a c ear a or er ma er as more
mar s are awar e or sou use o P sics a or correc a swers.

e e o e exami a io as e a our wor secure o e er.


e um er o mar s is i e i rac e s a e e o eac ues io or par ues io .

For Examiner’s Use


PARENT’S
Section A
SIGNATURE
Section B

Total

is paper co sis s o 21 pri e pa es a 1 a pa e.


e er rs su K

Turn over

395
Section A (50 marks)
swer all e ues io s i is sec io .

1 pe ro ri e car acce era es rom res o i s cruisi spee a o a s rai e e roa .

(a) ae e mai e er c a es i e car a i i s surrou i s w e

1. e car is acce era i

.........................

2. e car is cruisi a a co s a spee .

...................................................

(b) e car ow mo es up a s ope wi co s a spee .


Exp ai w e er e ra e o pe ro co sump io wi i crease s a e same or
ecrease.

........

2 sma e p a e w ic ca carr six peop e is s ow i i . . .

Fig. 2.1

e mass o e u oa e p a e is . is i i ia a res . e e p a e is
a i o e wo e e i es ca exer a o a rus orce o a e ric io
e wee e w ee s a e rou is . o orces remai co s a uri
a eo .

(a) a cu a e e acce era io o e p a e as i s ar s o mo e.

acce era io ..............................

Turn over

396
(b) Exp ai w a appe s o is acce era io as e p a e spee s up.

...................................................................................................................................

...................................................................................................................................

...................................................................................................................................

...................................................................................................................................

..............................................................................................................................

(c) e a era e acce era io uri a e o is . m s .

(i) a cu a e e ime a e p a e wi a e o reac a a e o spee o


m s.

ime a e .............................

(ii) a is e mi imum e o e ru wa a is re uire or epa e o


a eo

mi imum e ...............................

(d) u es w e w ee s o e p a e are o e i o e o o e p a e a er
a eo .

...................................................................................................................................

..............................................................................................................................

Turn over

397
3 arc er pu s e s ri o is ow a i is s re c e a ori o a is a ce o
cm as s ow i i . . . s e re eases e s ri a a era e orce o ac s
o e arrow e ore i oses co ac wi e s ri .

Fig. 3.1

(a) a cu a e e a era e wor o eo e arrow.

wor o e

(b) a is e spee o e arrow as i ea es e ow i e arrow as a mass o

spee ... .

(c) a e two wa s i w ic e spee o re ease ma e i crease .

... ..

... ..

...................................................................................................................................

... .....

Turn over

398
4 ra ar s s em suc as e o e s ow i i . . is o e use i airpor s or air ra ic
co ro . e s s em co sis s o microwa e ra smi ers a recei ers arra e i a
sp erica s ruc ure. emi i microwa e si a s a recei i re ec e si a s e
ra ar s s em pro i es ower co ro ers wi i orma io o e mo eme o aircra s
approac i e airpor .

Fig. 4.1

aerop a e is approac i e airpor . e ime e a o recei i a microwa e si a


re ec e rom e aerop a e is . x s.

(a) a is e is a ce o e aerop a e rom e air ra ic co ro s s em

is a ce ................................

(b) e wa e e o e microwa e si a is cm. a cu a e i s re ue c .

re ue c ...................................

Turn over

399
(c) Exp ai w e microwa e ra smi ers a recei ers are arra e i a sp erica
s ruc ure.

...................................................................................................................................

...................................................................................................................................

..............................................................................................................................

(d) ea i er p a es are i isi e o ra ar s s ems ecause e pre e e


microwa e si a s rom ei re ec e ac o e recei er o e s s em.
u es o e wa i w ic s ea i er p a es ca ac ie e is.

...................................................................................................................................

...................................................................................................................................

..............................................................................................................................

Turn over

400
5 i . . s ows e empera ure c a es o a so i su s a ce as erma e er is
supp ie o e su s a ce a a co s a ra e.

Fig. 5.1

(a) ae e process a is appe i e wee P a Q.

..................

(b) Exp ai i mo ecu ar erms w a appe s o e e er supp ie e wee

(i) Pa Q

..........................

.............................

.............................

.....................

(ii) Qa R.

..........................

.....................

(c) a e ow e speci ic ea capaci o e i ui i er rom e speci ic ea


capaci o e so i a exp ai ow ou e uce is rom e rap s ow
i i . . .

.......................

.......................

.......................

.......................

..................

Turn over

401
6 i . . s ows e o ec a i s ima e wi ra X mo i owar s a co er i e s.

op a o co er i e s

ob ect ray X

Pri cipa
axis
image

. cm

. cm
Fig. 6.1

(a) (i) raw o i . . o e ra o oca e e oca poi o e e s.


ar e oca poi wi e e er F.

(ii) e ermi e e oca e o e e si i . . .

oca e ....................................

(iii) omp e e e pa or ra X.

(b) e op a o e co er i e s is remo e saea exp ai w e er e


ima e is s i orme .

...................................................................................................................................

...................................................................................................................................

...................................................................................................................................

..............................................................................................................................

Turn over

402
7 ci i m s ow i i . . is a i p as ic i m pica use or sea i oo i ems
i co ai ers o eep em res o er a o er perio o ime. e i m c i s o o ma
smoo sur aces a us ca remai i o er e ope i o a co ai er wi ou
a esi es or o er e ices. is ma e o a ma eria w ic ecomes c ar e easi .

Fig. 7.1

(a) Exp ai i erms o c ar es ow a eu ra c i i m is i ere rom a c ar e


ci i m.

...................................................................................................................................

...................................................................................................................................

...................................................................................................................................

..............................................................................................................................

(b) s u e ries o pee a c i i m rom i s ro . e oes is wi muc i icu as


e i s a eci i m e s o s ic o e ro .
Exp ai i e ai w is appe s.

...................................................................................................................................

...................................................................................................................................

...................................................................................................................................

...................................................................................................................................

...................................................................................................................................

..............................................................................................................................

Turn over

403
8 (a) i . . s ows a iro ri suspe e a rea . ar ma e is e c ose o
e ri . e iro ri is a rac e o e ma e.

Fig. 8.1

Exp ai w e iro ri is a rac e o e ar ma e.

....

(b) i . . s ows a a umi ium ri suspe e a rea c ose o a ar ma e.

Fig. 8.2

Exp ai w e a umi ium ri is o a rac e o e ma e.

...

Turn over

404
(c) e e po e o e ar ma e i i . . is mo e uic owar s e
a umi ium ri ere is a i uce curre i e ri a e ri mo es awa
rom e ar ma e .

(i) Exp ai w a curre is i uce i e a umi ium ri .

...

...

...

..

(ii) Exp ai w e a umi ium ri mo es awa rom e ma e.

..

..

...

..

Turn over

405
9 a er a i i pu o a e is app ie across e p a es o a ca o e ra osci oscope
a pro uces e race s ow i i . . .

Fig. 9.1

e pea o a eo e a er a i i pu o a e is . a i s re ue c is
e ermi e e

(a) ai se i

ai se i ............................. cm

(b) ime ase se i

ime ase se i ............................ ms cm

(c) ec o i . . e ew race w ic wi e o ai e or e same i pu o a e


i e ai is c a e o per i isio a e ime ase se i o . ms
per i isio .

Turn over

406
Section B (30 marks)
swer all e ues io s rom is sec io .
swer only one o e wo a er a i e ues io s i Question 12.

10 i . . s ows a so ar w ic is use o e ermi e e spee o a e ic e.


e so ar se s pu ses o u rasou owar s o comi e ic es a recei es e
re ec e pu ses.

Fig. 10.1

ime s rip s owi w e e pu ses are emi e a recei e is pri e as s ow i


i . . . e spee o u rasou i air a room empera ure is m s. e car is
ra e i a a co s a spee o e e .

Fig. 10.2 o raw o sca e

(a) ime s e so ar emi s a pu se P . ime . s e so ar recei es e


re ec e pu se . e ime s rip or ese wo recor e pu ses are s ow i
i . . .

(i) ae e ime a e emi e pu se P wi mee e o comi car.

ime ..............................

Turn over

407
(ii) a cu a e e is a ce o e car rom e so ar.

is a ce ...............................

(b) ime . s e so ar emi s a seco pu se P a recei es e seco


re ec e pu se a ime . s. e car is ow a a is a ce rom e so ar.
e ime s rip or ese wo recor e pu ses are s ow i i . . .

Fig. 10.3 o raw o sca e

(i) a cu a e e is a ce o e car rom e so ar.

is a ce .............................

(ii) a cu a e ow ar e car as mo e o e e uri is perio o ime.

is a ce mo e ................................

Turn over

408
(c) e ermi e e spee o e car.

spee o car ...............................

(d) (i) Exp ai w e re ec e pu se is sma er i amp i u e a e emi e


pu se.

...........................................................................................................................

......................................................................................................................

(ii) escri e usi i eas a ou e i ra io o mo ecu es i e air w a is


mea a lower amplitude.

...........................................................................................................................

......................................................................................................................

Turn over

409
11 i . . s ows a ri i rec a u ar car w ic as a rec a u ar o e cu ou i e
ce re.
i . . s ows e se up use o measure e acce era io o e car as i a s
ree o e rou . orc i w ic is irec e owar s e i epe e resis or
is ur e o . compu er is use o measure e po e ia i ere ce across .

op

. cm

. cm

o om
. cm
car a s
orc i ree
.

Fig. 11.1 Fig. 11.2

i . . s ows e rap o po e ia i ere ce p. . across a ai s ime.

p. .
ime ime
i er a i er a

.
ime s
. . . .

Fig. 11.3

Turn over

410
(a) aew a appe s o e resis a ce o e as e car a s.

...................................................................................................................................

...................................................................................................................................

...................................................................................................................................

..............................................................................................................................

(b) Exp ai w e p. . across e aria e resis or rops o . .

...............................................................................................................................................

...............................................................................................................................................

...............................................................................................................................................

...............................................................................................................................................

..........................................................................................................................................

(c) Exp ai w ime i er a is o er a ime i er a as s ow i i . .


w e e ri i car a s.

...................................................................................................................................

...................................................................................................................................

...................................................................................................................................

..............................................................................................................................

(d) a cu a e e a era e acce era io o e car i cm s .

acce era io ............................. cm s

Turn over

411
12 EITHER
or er o c ec w e er a pipe sec io is ea i o e s o e pipe are sea e .
s ri e is co ec e o o e e w i e a wa er ma ome er is co ec e o e o er
e as s ow i i . . .
pis o pipe sec io
s ri e

cm o air

cm o air
wa er ma ome er
Fig. 12.1

e s ri e i i ia co ai s cm o air w i e e pipe as cm o air.


e air i i ia as a pressure o . x Pa.
e pis o o e s ri e is e pus e i a e s ea i i posi io .
is causes e o ume o air i e s ri e o ecrease.

(a) Exp ai i erms o mo ecu es w e pressure o e air i si e e pipe


i creases w e e pis o o e s ri e is pus e i .

................

................

................................................................................................................................

.............

(b) ssumi a e pipe does not ea a e empera ure o e air i si e e


pipe remai s co s a w e e pis o is pus e u i a e i p ace
ca cu a e

(i) e ew air pressure i e pipe i Pa

pressure .. Pa

Turn over

412
(ii) e i ere ce i wa er e e s i o arms o e ma ome er.
ssume e s re o e Ear s ra i a io a ie is a e
e si o wa er is m .

i ere ce i wa er e e s ..

(c) e pipe is ea i er s ow w a wi e e o ser a io a er e pis o is


pus e u i a e i p ace.

................

................................................................................................................................

. ...............

................................................................................................................................

................

.. ............

(d) a e o e wa ow e ma ome er cou e mo i ie o i e a ar er i ere ce


i i ui e e or e same pressure i e pipe.

......................

.............

Turn over

413
12 OR

armer co ec s a ouse o e mai s supp o e ec rici .


e ouse is a a o is a ce rom e eares mai s supp o e ec rici .
i . . s ows e mai s supp co ec e o e ouse.

Fig. 12.2
(a) e armer uses amps i e ouse u e o o i up a eir orma
ri ess. Exp ai w e amps are im.
...................................................................................................................................

...................................................................................................................................

...................................................................................................................................

..............................................................................................................................

(b) e armer a e ra s ormers as s ow i i . . .

Fig. 12.3
e amps i e ouse ow i up a eir orma ri ess.
Exp ai w e amps are ow ri er.
...................................................................................................................................

...................................................................................................................................

...................................................................................................................................

..............................................................................................................................

Turn over

414
(c) Exp ai i e ai ow a ra s ormer pro uces a ou pu o a e.

...................................................................................................................................

.......................

.......................

...........................

.......................

..................

(d) e mai s supp pro i es o power o ra s ormer i i . . .


(i) a cu a e e curre supp ie o e ra s ormer.

curre ............................

(ii) a cu a e e e er supp ie o e ra s ormer i mi u es.


i e our a swer i ou es.

e er ............................

END OF PAPER 2

Turn over

415
416
2018 PRELIMINARY MCQ 4E Physics 6091/1

Question No. Answer Question No. Answer

D B

D B

B A

B D

A B

B A

B A

A A

A A

A B

A A

B D

D C

A B

C A

B A

A D

B D

B C

C C

417
1

2018 Preliminary Examination


4E Physics (6091/2)
Marking Scheme

Section A (50 marks)


Questi Answers Marks
on No.
a . emica e er i e ue is primari co er e o i e ic e er o
e car.

. emica e er i e ue is mai co er e o wor o e a ai s


ric io a air resis a ce.

e ra e o co sump io o pe ro wi i crease as e ccarar as a e ai i


a
e er ue o i s i creasi ra i a io a po e ia e e
err .

a e

rom e ma a
. ms

s e p a e spee s up e air resis a ce i experie


e perie ces i creases.
ex crea
eases.
ea
i ce e orwar orce is co s a e e orce e ac i o epa e
ecreases a is ea s o a ecrease
ecrea
e se i i s acce
acce
c era
e io .

c i si a u .
s

ii i imum e o ru
u wa area
ar u er rap
e x x E
. m
m

e w ee
ee s are
are o e i o e o o e p a e o re uce air resis a ce
w e i i .

a or o e x
x . e uc mar i u i is wro

m
x . x E or e er
. m s

418
2

c Pu e s ri a o er ori o a is a ce ac .
se a i er arrow.

a pee o microwa e c x m s

is a ce o e aerop a e x x x . x E
m

si x . E or a ue o c x
. x e uc mar i u i is
is wro
wrro
w

c e sp erica s ruc ure a ows microwa es si a s o e se a recei ei e


rece
ce
rom a irec io s e ce pro i i e ower co ro ers wi accura
accu
ac c ra
cu ae
i orma io o a i comi aircra s.

e o o e i er p a es a sor s e microwa e
essa e ce
pre e s em rom ei re ec e ac .
or
e o o e i er p a e is s ape i a wa
wa a re ec s e
re ec
microwa e si a s awa rom e source.

a e i

i E er is ai e e so i o o ercome e i er ermo arr o s


m ecu a
e wee e mo ecu es so a ereerre is a c a e o s a e rom so i o
i ui .
is resu s i a i crease i e i err a popo e ia
ia e er o e i ui
mo ecu es.

ii E er is ai e u o i crease
e i ui crea
crease
s e i er a i e ic e er o e
mo ecu es. o e emp
empera
per
era
a ur
uree o e i ui i creases.

c e sp
speci
pec
ecii icc ea capaci
capa
ca paci
ci o e i ui is i er a e speci ic ea
capaci
ci o e so s i .

rra
a ie o e i e ra e o c a ei empera ure o e su s a ce

P mc so c P m

e ra ie o e i e P so i is rea er a e i e i ui .
e ra e o ea supp ie P o e su s a ce o mass m is co s a .
ere ore e speci ic ea capaci c o e so i is sma er a a o e
i ui .

419
3

a
op a o co er i e s

o ec
e ra X

ima
iim
ma e

. cm

. cm
i correc ra rawi
correc mar i o

ii oca e x . cm

iii correc su se ue ra rawi


raw
aw
wi or
or ra

e ima e is s i orme as
a e ra s rro
romm e o ec
ec cca
a si e re rac e rou e o om par o
e e s.

a eu ra c i i m as a e ua um er o posi i e a e a i e c ar es
w ie
a c ar e c i i m as a excess o ei er posi i e or e a i e c ar es.

e eci i m is pee e rom e ro ric io e wee e sur aces


wi cause e ec ro s o e ra s erre e wee e pee e c i ima e
o e remai i o e ro .
e si e o e c i i m wi ose e ec ro s a ecome posi i e
c ar e w i e e o er si e wi ai e ec ro s a ecome e a i e
c ar e .
i ce u i e c ar es a rac e a rac i e orces e wee ese wo si es
wi cause e c i i m o s ic o e ro .

a ro is a ma e ic ma eria so i ecomes ma e i e i uc io wi a
i uce ou po e aci e or po e o e ar ma e .
i e po es a rac . o e iro ri is a rac e o e ar ma e .

umi ium is a o ma e ic ma eria so i wi o e a rac e o e ar


ma e .

c i s e ar ma e mo es uic owar s e a umi ium ri ere is a


c a e i ma e ic ux i a e wi e ri cu i o ma e ic ux
e ri .
o e ec roma e ic i uc io occurs a a i uce curre is pro uce i
e ri .

420
4

ii ccor i o e s aw aw o o ser a io o E er e i uce


curre owi i e me a ri wi pro uce i s ow ma e ic ie
ie wi i s
or po e aci e or po e o e ar ma e approac i i .
i e po es repe so e ree suspe e ri wi swi awa a rom
rom e ar
ma e .

a Pea o a e . so ai se i . cm

. s ms
so i . y . s ms
e ce ime ase ms cm

correc ei

correc perio

421
5

Section B (30 marks)


Question Answers Marks
No.
a i e emi e u rasou pu se wi mee e e ic e a ime . s.

ii si is a ce s co s a spee x ime a e
msx . s
m

i is a ce s x .
m

ii e i ere ce i is a ce s s m E

c e ime i er a e wee e emi e pu ses rea


ac i
reac e car
. . . s
uri is ime i er a e car a a ces m.
m.
o e spee o e car . . m s

s e sou wa e passes rou e air a re ec s ac


ac par o e
i
wa e e er is issipa e i o e surrou i air a r a a sor
ai sor e e
re ec i sur ace. o e re ec e pu se is sma
sm
ma er i amp i u e a
e ori i a pu se.

e air mo ecu es i ra e wi a er
sma e maximum
maxi
ma ximu
mum isp aceme rom
ii
eir ori i a posi io .

a e irom e orcorc i iss oc


o e e car e resis a ce o
e i cr
creases.
rea
eases.
e e i iss o oc
oc e e car a reac es e e
resis
resi
s s a ce o e ecreases.
e

e i is
is oc e e car e ri ess arou e is
ow
ow e ce e resis a ce o e i creases.

e resis a ce o i creases e po e ia i ere ce across e


a so i creases.

i ce e aria e resis or is i series wi e w e p. . o


i creases e p. . across e aria e resis or wi ecrease.
em

422
6

c e ime i er a is cause e o om o e car w ic reac es


e e e irs

ime i er a is cause e op o e car w ic i reac es e


e e a er.
ar ca e i e i s u e somew a ma es e correc i e wee
e ime i er a s a e par o e car

e e op o e car reac es e e e e spee o e car


is as er ue o acce era io cause ra i e ce e s orr er ime i
i er a .

i ia spee . . .
. cm s

i a spee . . .
. cm s

ime i er a . . . .
. s

era e acce era io u


. . .
c
cm s E a owe

or
s u a
a . .
a cm s

Ei er e e pis
s o iss pus
pus e i e um er o mo ecu es per u i o ume
a i si e e pipe
pip
ipee i creases.
crea
cr e ses.
ea
s a resu
res
esuu e rer ue c o co isio s e wee e air par ic es a e
i erer wa o e pipe i creases. e ce e pressure i creases.

i P P
P . x x
. x Pa

U . x . x E
ii
x x
. m

423
7

c e wa er e e i e e arm o e ma ome er wi i crease w i e


e wa er e e i e ri arm wi ecrease
u i o are a e same e e .

se a i ui wi a ower e si a wa er i e ma ome er.

r e e curre I rom e mai s supp ows rou e o ca e


a o resis a ce e ec rica e er wi e was e as ea ue o e
ea i e ec i e ca e. P .
o e power e er ou pu a e ouse is ower a e o a e
supp ie o e ouse is a so ower .
e ce e amps are im.

e s ep up ra s ormer i creases e o a e u rre uces


e uc
c es e
curre I i e ca es.
o ess e er wi e was e as ea i e ra
ra smi
smi i ca
sm ca es.
s
o ou pu power a s ep ow ra s ormer
orme
me
er wi
wi e i er er a a a
e o a e supp ie o e ouse is a mos .

c e a a er a i curre ows rou e primar co


coi
oi i pu i
se s up a a er a i ma e ic ie
w ic i s wi e seco ar coi ou pu ia e so iro core.
ue o e c a e i ma e icc ux i a e wi wi e seco ar coi
e ec roma e ic i uc io o occurs
occu
oc urs
r a e seco
seco ar coi a a
a er
er a i e.m. . is pro uce
uce across
acro
ac ross
sss e e s o e seco ar coi .

i P I
I P .

ii E P x x

Note:
x or i a a swerr or eac mis a e e uc mar up o a maximum o
mar s per paper.
or exac a ue ee o wri e a swer o .
x o u i wri e or i a a swer or eac mis a e e uc mar up o a
maximum o mar s per paper.
x e o o s ow or ca cu a io s e o i e mar or swer.
x o i e mar .

Setter: Mrs Hsu Lay Keok

THE END

424
425
E E

EE E Pare s i a ure

P E E

E E P E

PHYSICS 6091/01
Paper u ip e oice 12 Sep 2018
1 hour
i io a a eria s u ip e oice swer ee

READ THESE INSTRUCTIONS FIRST

ri e i so pe ci .
o o use s ap es paper c ips ue or correc io ui .
ri e our ame a i ex um er o e swer ee i e spaces pro i e .

ere are forty ues io s o is paper. swer all ues io s. or eac ues io ere are our
possi e a swers A B C a D.
oose e one ou co si er correc a recor our c oice i soft pencil o e separa e swer
ee .

Read the instructions on the Answer Sheet very carefully.

Eac correc a swer wi score o e mar . mar wi o e e uc e or a wro a swer.


rou wor i s ou e o ei is oo e .
e use o a appro e scie i ic ca cu a or is expec e w ere appropria e.

is ocume co sis s o 18 pri e pa es.


e ers rs Pa s a Turn over

426
2

1 ic u i is e ui a e o e u i or power

A ms
B m s
C m s
D m s

2 orces o . a . ac a a poi .

ic o e o e o owi cou not e e ma iu eo eir resu a

A .
B .
C .
D

3 e rap e ow s ows ow e e oci o a mass c a es wi ime.

e oci

ime

ic o e o owi s a eme s a ou e mo io o e mass is not rue

A e spee o e mass ecreases a a i creasi ra e a e i creases a a


ecreasi ra e.
B e acce era io o e mass is e a i e rou ou i s mo io .
C e i a spee o e mass is ar er a i s i i ia spee .
D e o a isp aceme o e mass is posi i e.

427
3

4 car is ece era i u i orm a i s e oci c a es rom m s o m s i . s.

a ur er is a ce wi i ee o ra e e ore i s ops comp e e

A m
B m
C m
D m

5 a is row across a a rou .


pa o a

ic s a eme escri es e mo io o e a w e e e ec s o air resis a ce are


e i i e

A e a a s wi e same spee a w ic i is row .


B e spee o e a is ero a e i es poi o e mo io .
C e acce era io o e a is ero a e i es poi o e mo io .
D e acce era io o e a is ar es a e s ar a ecreases s ow o ero.

6 cra e o mass . res s o a rou ori o a sur ace. i s ri a ac e o e


cra e passes o er a smoo pu e a suppor s a oa o mass . a i s o er e .

cra e
. smoo pu e
rou sur ace

e e cra e is re ease a ric io a orce o . ac s o i . e ra i a io a ie


s re is .

a is e acce era io o e cra e

A . ms
B . ms
C . ms
D ms

428
4

7 a er res s o a rou rou a ea s a ai s a rou wa .

rou wa

rou rou

s wei W ac s rou e ce re o ra i . orces a so ac o e a er a a


. ese orces are X a Y respec i e .

ic ec or ria e represe s e orces o e a er

A B

X X

W Y
W
Y

C D

X X

Y
W W
Y

429
5

8 ree a s a e e si ies o . cm . cm a . cm respec i e . ur


e are pu i o our ea ers co ai i i ere i ui s.

ree o e ea ers o oi o e si . cm wa er o e si . cm a mercur


o e si . cm . ese are

A a respec i e .
B a respec i e .
C a respec i e .
D a respec i e .

9 s ape ri i e er arm is pi o e a poi P. ree orces ac o e e er arm as


s ow i e ia ram.

a is e ma iu eo e resu a mome o ese orces a ou poi P

A m
B m
C m
D m

430
6

10 e s a i i o a us is es e i i i o a ramp. e ia ram s ows a us a is us


a ou o opp e o er.

ere is e ce re o ra i o e us

11 e ia ram s ows a u e co ai i oi a wa er.

oi
cm wa er
cm

cm

a is e ra io o e e si o oi o e e si o wa er

A
B
C
D

431
7

12 o ec o mass m s i es up a ric io ess s ope rom poi wi a i i ia e oci v as


s ow i e i ure e ow. e o ec comes o a s op a a ei ha o e .

v
h

v
seco o ec o mass m ow s i es up e same s ope rom wi a e oci o .
a is e ei i wi rise o

h
A
h
B
h
C
D h

13 s u e o ser es e row ia mo io o smo e par ic es i air wi a microscope. e


sees mo i poi s o i .

ere o ese poi s o i come rom

A e i ra i air par ic es o .
B e i ra i smo e par ic es o .
C e co i uous mo i air par ic es o .
D e co i uous mo i smo e par ic es o .

14 esse co ai s a as. ome as is pumpe ou o e esse .

e o ume o e esse is ep co s a w ic s a eme is not correc

A e pressure o e as ecreases.
B e o a wei o e esse ecreases.
C e a era e i ermo ecu ar is a ce ecreases.
D e a era e i e ic e er o e mo ecu es ecreases.

432
8

15 w a processes oes a ea er o o wa er ose e er

A o ec io a ra ia io o .
B E apora io a ra ia io o .
C o uc io co ec io a ra ia io o .
D o uc io co ec io e apora io a ra ia io .

16 ic o e o owi processes is a i us ra io o ea ra s er ra ia io o

A rom e su o e ear a mosp ere.


B rom e o ire p ace o e res o e room.
C rom a o ame o e a s p ace a o e e o ame.
D rom e i ame o a amp o e a s p ace esi e e amp.

17 ic o e o owi is not o e o e s eps ee e i e e ermi a io o e ice poi


w e ca i ra i a i ui i ass ermome er wi e e sius sca e
i mmerse e u a e ower par o e ermome er s em i o a u e
co ai i pure me i ice.
ii easure e empera ure o e irs ew rops o me e ice w ic s ou e
.
iii e e mercur e e i e ermome er s em remai s s ea mar a e e
as ice poi o e s em.

A i o
B ii o
C i a ii
D i a iii

18 ree i ui s wi eir respec i e oi i a ree i poi s are s ow i e a e e ow.

mercur a co o pe a e
ree i poi
oi i poi .

ic i ui s cou e i e i a ermome er a ca measure e wee a

A a co o
B pe a e
C mercur
D a co o or pe a e

433
9

19 o o wa er a is a e o some ice a . e speci ic a e ea o usio


o ice is a e speci ic ea capaci o wa er is . .

a is e mi imum mass o ice a was me e

A
B
C .
D .

20 ra s erse wa e ra e s a o a s ri wi a co s a spee . e ia ram e ow s ows


e s ape o e s ri a a cer ai i s a .

ic o e o owi ia rams s ows e s ape o e s ri a a uar er o a perio


a er

A B

C D

434
10

21 ipper mo i up a ow ma es wa es i a ripp e a .

a wi appe w e e ipper re ue c is ecrease

A e wa es wi e ur er apar .
B e wa es wi mo e more s ow across e a .
C e wa es wi mo e more uic across e a .
D e wa e pea s wi e ower a e rou s i er.

22 a er wa es ra e across a ripp e a . e ori o a is a ce e wee a cres a e


ei ouri rou is mm a e er ica is a ce e wee a cres a a rou is
. mm. cres ra e s . cm i . s.

ic o e o e o owi is correc

re ue c o wa er wa e amp i u e o wa er wa e mm
A . .
B . .
C . .
D . .

23 ra o i is i ci e a a a eo o a mirror . o er mirror is arra e


a a a eo o .

er re ec io rom e ra is i ci e o .

a is ea e o i ci e ce o e ra a e mirror

A
B
C
D

435
11

24 i . a s ows a i eam e eri i o a semi circu ar ass oc a a a e b a


ei re rac e a a a e a. ea eba i s correspo i a e a is s ow i
i . .

b/° a/°

Fig. (a) Fig. (b)

a is e cri ica a eo e ass oc

A
B
C
D

25 e ow are our s a eme s a ou e uses o e ec roma e ic ra ia io .

x ra s are use i ra ar mo i ori o spee o mo or e ic es.


x icrowa es are use i sa e i e commu ica io .
x amma ra s are use i me ica rea me .
x a io wa es are use i i ru er a arms.

ow ma o ese s a eme s are correc

A
B
C
D

26 so ic ape measure is use o measure e e o a room. measures a ime


i er a o . s e wee ra smi i a sou pu se a recei i e ec o. e spee
o sou i air is m s.

ow ar is e re ec i wa rom e ape measure

A . m
B . m
C . m
D . m

436
12

27 e a e s ows ow e spee o sou aries wi su s a ces o i ere e si ies.

su s a ce spee o sou i su s a ce e si o su s a ce
ms m
air as .
ox e as .
a umi ium me a
iro me a
ea me a

a co c usio s a ou e spee o sou ca e raw rom is i orma io

A e spee is rea er i me a s a i ases.


B e spee is rea es i e mos e se me a .
C e spee is rea er i ess e se su s a ces.
D e spee i creases as e e si o e su s a ce i creases.

28 i u c ar e co uc i a is mo e owar s e posi i e p a e.
o rea

ic ia ram correc s ows e c ar es o e a us a er i as ouc e e


posi i e p a es

A B C D

437
13

29 ome o ser a io s were ma e w e our c ar e ro s P a are p ace ear o


eac o er o e a a ime.

x P repe s
x P a rac s
x a rac s

is e a i e c ar e w a are e c ar es o P a

P
A
B
C
D

30 e po e ia i ere ce across a i u is . uri a ime o s e amou o


e ec rica e er co er e o o er orms o e er is .

a is e curre owi i e i u uri is ime

A .
B .
C .
D

31 a is e e i i io o e e ec romo i e orce e.m. . o a power supp

A e po e ia i ere ce across i s ermi a s w e o curre is owi


B e e er co er e rom o er orms o e ec rica per u i c ar e w ic passes
rou i
C e e er co er e rom e ec rica o o er orms per u i c ar e w ic passes
rou i
D e po e ia i ere ce across i s ermi a s i u i po e ia i ere ce is ee e o
mo e u i c ar e rou i

438
14

32 ome e ec ric i u i ame s are ma e o car o . is ow a e resis a ce o


car o i ame s ecreases as eir empera ure i creases.

ic rap s ows ow e curre I rou suc a u aries wi e po e ia


i ere ce V across i

A B
I I

s s

C D
I I

s s

33 e ia ram s ows a e wor o i e ica resis ors a co ec e o a ce .


e swi c is c ose e power issipa e i eac resis or is P.

e swi c is ope e w a are e powers issipa e i resis ors a

power issipa e i power issipa e i


A ower a P remai s as P
B ower a P ower a P
C remai s as P rea er a P
D ower a P rea er a P

439
15

34 a er o e.m. . E is co ec e oa i epe e resis or a wo resis ors


eac o resis a ce R as s ow .

e resis a ce o e w e i is expose o ri i is R. e curre s i e wo


resis ors are I a I respec i e .

I
R

E
I
R

ow o e curre s c a ew e e i i e si o e is re uce o ero

I I
A ecrease ecrease
B i crease ecrease
C ecrease i crease
D i crease i crease

35 use is co ec e o e eu ra wire o a a .

ic o e o owi s a eme s is are correc

e a wi o opera e i e use ow.


e a wi e sa e o ouc w e e use ow.
e a wi s i e co ec e o e i o a e source w e e use ow.

A a o
B a o
C a o
D a

440
16

36 ar ma e is o e p ace i a o u i orm ma e ic ie as s ow i e i ure e ow.

ic o e o owi escri es e su se ue mo io o e ma e

ro a io mo eme
A a i c oc wise o e e
B a i c oc wise o e ri
C c oc wise o e e
D c oc wise o e ri

37 s rai co uc or res s i e space e wee wo arms o a so iro core.

swi c

so iro core

co uc or

er e swi c as ee c ose c
or a w i e i w ic irec io is em
ma e ic orce ac i
o e co uc or

A up
B ow
C e
D ri

441
17

38 our para e co uc ors a carr i e ua curre s pass er ica rou


e our cor ers o a s uare. co uc ors a e curre is owi i o e pa e
a i co uc ors a curre is owi ou o e pa e.

ic o e o owi incorrectly escri es e resu a orce o co uc or E wi


curre owi i o e pa e a e ce re o e s uare

A e resu a orce ue o wires a poi s owar s i e perpe icu ar o


.
B e resu a orce ue o wires a poi s owar s i e perpe icu ar o
.
C e resu a orce ue o wires a poi s owar s i e perpe icu ar o
.
D e resu a orce ue o wires a poi s owar s i e perpe icu ar
o .

39 e ia ram s ows a experime a se up s owi e ec roma e ic i uc io .

wire

w ic irec io mus e wire e mo e oi uce a curre ow i e irec io as


s ow i e ia ram

A er ica upwar
B er ica ow war
C ori o a owar s e or po e
D ori o a owar s e sou po e

442
18

40 ra s ormer co sis s o o e coi wi ur s a a seco coi wi oa o


ur s w ic ca e appe a arious p aces.

ur s

ur s
ur s
ur s
ur s

ic pair o ermi a s s ou e co ec e oa amp or i o e i orma

A
B
C
D

END OF PAPER

443
E E

EE E Parent’s Signature:

P E E

E E P E

PHYSICS 6091/02
Paper eor 11 Sep 2018
1 hour 45 minutes
o i io a a eria s are re uire .

READ THESE INSTRUCTIONS FIRST

ri e our ame a i ex um er o a e wor ou a i .


ri e i ar ue or ac pe .
ou ma use a pe ci or a ia rams or rap s.
o o use s ap es paper c ips ue or correc io ui .

Section A:
swer all ues io s.

Section B:
swer all ues io s. ues io as a c oice o par s o a swer.

a i a es are remi e a all ua i a i e a swers s ou i c u e appropria e u i s.


e use o a appro e scie i ic ca cu a or is expec e w ere appropria e.
a i a es are a ise o s ow a eir wor i i a c ear a or er ma er as more mar s
are awar e or sou use o P sics a or correc a swers.
e um er o mar s is i e i rac e s a e e o eac ues io or par ues io .

For Examiner s Use


Section A /50
Q9 /10
Q10 /10
E/O Q11 /10
TOTAL /80

is ocume co sis s o 24 pri e pa es.


e ers rs Pa s a Turn over

444
2

SECTION A
swer all e ues io s i is sec io i e spaces pro i e .
e o a mar or is sec io is .

A1 i . . s ows a oi rum as i oa s i e ui i rium i seawa er.

Fig. 1.1

e rum is . m o a as a cross sec io a area o . m . e e o e


rum su mer e u er wa er is . m. e a mosp eric pressure a o e e sur ace o
e wa er is . u Pa. e e si o seawa er is . u m− a e ra i a io a
ie s re is − .

(a) (i) i . . raw wo arrows o s ow e irec io s o e orces ue o e


pressures o e op o e oi rum a o e ase o e oi rum.

(ii) a cu a e e resu a o e wo orces i (i).

resu a orce ........................................

(iii) e ce e ermi e e a era e e si o e rum a i s co e .

e si ........................................

445
3

(b) e rum is pus e i o e seawa er cm a e re ease . osci a es


er ica . i . . s ows e aria io o i s er ica isp aceme s wi ime t.

Fig. 1.2

(i) escri e ow ou wou use e rap o i . . o e ermi e e maximum


er ica spee o e oi rum.

……………………………………………………………………………………….

……………………………………………………………………………………

(ii) Exp ai w e amp i u e o e osci a io s is o co s a .

……………………………………………………………………………………….

……………………………………………………………………………………

446
4

A2 coi o a ou ur s o i su a e wire is i wou o a o ma e ic u e o


ma e a so e oi o mea ra ius mm as s ow i i . . . e o a e o e wire
i e coi is m. e wire i se as ra ius . mm a is ma e o a ma eria o
resis i i . u  Ω m. e coi is co ec e o a supp o e.m. . .

Fig. 2.1

(a) a cu a e

(i) e o a resis a ce o e coi

resis a ce ........................................

(ii) e curre i e coi .

curre ........................................

(b) i . . raw e pa er o e ma e ic ie wi i a arou e so e oi .


se arrows o s ow e irec io o e ie i si e e so e oi .

447
5

(c) s ape piece o s i wire E pi o e a E is i ser e i o e so e oi


as s ow i i . . .

Fig. 2.2

as e mm a E eac a e e mm.

e s i wire is irs a a ce w e ere is o curre i i . curre is e passe


rou a i or er o re a a ce e s i wire a orce o . u  is app ie
a a is a ce o mm rom e pi o as s ow rom e si e iew i i . . .

Fig. 2.3

(i) ae e irec io o curre ow i .

……………………………………………………………………………………

(ii) a cu a e e ma e ic orce o .

ma e ic orce ........................................

448
6

A3 (a) i e o e i ere ce e wee e wo me o s o ea ra s er co uc io a


co ec io .

……………………………………………………………………………………………….

……………………………………………………………………………………………

(b) Exp ai e o owi i erms o ea ra s er.

(i) e ouc e a emp me a cup as s ow i i . . ee s co w ereas


i ee s o w e i co ai s o co ee.

Fig. 3.1

……………………………………………………………………………………….

……………………………………………………………………………………….

……………………………………………………………………………………….

……………………………………………………………………………………….

……………………………………………………………………………………

(ii) ou e wa e ass as s ow i i . . use o ser e o ea oes o


ee o w e i is ei e e a .

Fig. 3.2

……………………………………………………………………………………….

……………………………………………………………………………………….

……………………………………………………………………………………

449
7

A4 e e wa s o i ou ow muc ice is ee e o ex rac e same amou o e er


rom a room as a air co i io er.

air co i io er u i o is swi c e o or ours o coo a room o .

(a) (i) a cu a e e amou o erma e er ex rac e rom e room e air


co i io er i e ours.

erma e er ........................................

(ii) a e wo assump io s a e e as ma e i is ca cu a io s.

1. …………………………………………………………………………………….

……………………………………………………………………………………….

2. …………………………………………………………………………………….

……………………………………………………………………………………

(b) a cu a e e mass o ice a ee e o ex rac e same amou o e er as


e air co i io er o reac e room empera ure o .
e speci ic ea capaci o wa er is a e speci ic a e ea
o ice is .

mass o ice ........................................

450
8

A5 i . . s ows a i correc e ec roma e ic spec rum raw a su e . e


compo e s o e spec rum a e wa e e s are i e wro or er. e a ues o
e wa e e s o o ma c e correc compo e s o e spec rum.

s or o
wa e e wa e e
ra io amma
microwa es u ra io e i ra re ra s isi e
wa es ra s
m − m − m − m − m − m − m

Fig. 5.1

(a) i . . comp e e e a e o e e ec roma e ic spec rum i e correc or er


o e arious compo e s a eir correspo i wa e e s.

s or o
wa e e wa e e

Fig. 5.2

(b) ae e spee o a e ec roma e ic wa es i a acuum.

spee ........................................

(c) a e o e o er proper o er a spee a a e ec roma e ic wa es a e i


commo .

……………………………………………………………………………………………

(d) ame o e e ec o a sorp io o e ec roma e ic wa es uma s.

……………………………………………………………………………………………….

……………………………………………………………………………………………

451
9

A6 i . . s ows a co er i e s o oca e cm ei use as a simp e


ma i i ass. ir ua ima e is orme cm rom e e s.

e sca e use i i . . is cm cm.

(a) i . . comp e e e ra ia ram o e ermi e e posi io o e o ec .

ima e

e e

cm
Fig. 6.1

(b) a e wo wa s i w ic e ima e orme a co er i e s use i a camera


i ers rom a orme a p a e mirror.

……………………………………………………………………………………………….

……………………………………………………………………………………………

452
10

(c) i . . s ows a ima e see e i a mirror a cm rom e mirror. e sca e


use is cm cm.

rawi wo ra s o i s ow ow e poi s P a are ei orme as


see a o ser er E. ow c ear e posi io o e o ec a e e a .

cm
E

Fig. 6.2

453
11

A7 e a i e c ar e ro is rou ear wo eu ra iso a e a s a . a is


e ear e mome ari as s ow i i . . .

e a i e c ar e ro
________
________

Fig. 7.1

(a) escri e a exp ai w a appe s o e wo a s w e e e a i e c ar e


ro is irs rou ear o em. ou are o assume a e a s a ro o o
come i o co ac .

……………………………………………………………………………………………….

……………………………………………………………………………………………….

……………………………………………………………………………………………….

……………………………………………………………………………………………….

……………………………………………………………………………………………….

……………………………………………………………………………………………

(b) (i) escri e w a appe s o e c ar es o a w e i is ear e


mome ari .

……………………………………………………………………………………….

……………………………………………………………………………………….

……………………………………………………………………………………

454
12

(ii) i . . raw e posi io s o e wo a s a i ica e eir c ar es i


a w e a is ear e mome ari .

e a i e c ar e ro
________
________

Fig. 7.2

(c) ae e c ar es o e wo a s w e e ro is remo e .

……………………………………………………………………………………………….

……………………………………………………………………………………………

455
13

A8 u c ar e car a er as a e.m. . o . is a er ca e i er a co s a
curre o . or a perio o . ours.

(a) a cu a e e o a amou o c ar e passi rou e a er i a ime o


. ours.

c ar e ........................................

(b) e u c ar e car a er is co ec e o a . Ω resis or a s ar er mo or


our si e i s a wo ea i s as s ow i i . . . e s ar er mo or is use
o s ar e e i e.

. Ω

Fig. 8.1

(i) e resis a ce o eac ea i is . Ω a e resis a ce o eac


si e i is Ω. a cu a e e curre i e a er w e swi c es a
are c ose a swi c is ope .

curre ........................................

456
14

(ii) e si e i s a ea i s are swi c e o . i c ose e curre i


e s ar er mo or is . Exp ai w a e i s ecome ess ri w e
is c ose .

……………………………………………………………………………………….

……………………………………………………………………………………….

……………………………………………………………………………………….

……………………………………………………………………………………

(c) e si e i s are acci e a e o w e e car is par e a pm. e ermi e


ua i a i e w e er e ri er is a e o s ar e e i e a am e o owi
a .

……………………………………………………………………………………………….

……………………………………………………………………………………………

END OF SECTION A

457
15

SECTION B
swer all e ues io s i is sec io .
swer o o e o e wo a er a i e ues io s i Question 11.

B9 ccor i o some scie is s a er powere cars o ere ma a a a es o er


pe ro ri e cars. ec ar ea e ea aci a eries are e mos commo pe o
a eries use i cars. i . . s ows some proper ies o pe ro o a par icu ar ea aci
a er a o a pica car.

Petrol
e si m−
c emica e er a ai a e −

Typical lead-acid battery


e er a ai a e w e u c ar e
mass
e.m. .

Car
o ume o pe ro a . u 
m
E icie c o ra s er o c emica e er
o pe ro o i e ic e er o car
ra orce a m s−

Fig. 9.1

(a) a cu a e e c emica e er a ai a e rom a u a o pe ro .

e er ........................................

(b) a cu a e e o a is a ce ra e e e car o a u a o pe ro w e
ra e i a a co s a spee o m s− o a e e roa .

is a ce ........................................

458
16

(c) (i) a cu a e e cos o c ar i e a er u i e cos o e ec rici is .


or o e .

cos ........................................

(ii) u c ar e a er e i ers a co s a curre o . . a cu a e e


ime i ours e ore e a er ee s o e c ar e a ai .

ime ........................................

(d) (i) a cu a e e o a mass o ea aci a eries ee e o pro i e e same


e er as a u a o pe ro .

mass ........................................

(ii) u es ow our a swer o (i) ma a ec e per orma ce o a a er


powere car.

……………………………………………………………………………………….

……………………………………………………………………………………

459
17

B10 e aria io wi ime t o e er ica spee v o a i a a i rou air is s ow


i i . . .

v m s−

t s
Fig. 10.1

e mass o e a is . e ra i a io a ie s re is − .

(a) i . . raw a i e o s ow e aria io wi ime t o e er ica spee v o


e a a i rom res i a acuum.

(b) se i . . o e ermi e e acce era io o e a a i rou air a ime


t . s. ow our co s ruc io o i . . .

acce era io ........................................

460
18

(c) or e air resis a ce ac i o is a ca cu a e

(i) e maximum resis i e orce

orce ........................................

(ii) e resis i e orce a ime t . s.

orce ........................................

(d) e a i s a so rou a ece era es u i orm a m s− .

e ermi e

(i) e ime a e or e a o come o a comp e e s op

ime ........................................

(ii) e is a ce mo e e a i e so rou .

is a ce ........................................

461
19

EITHER

B11 i . . s ows a coi o wire wou o a piece o so iro . ma e is ro a e i e


ap i e so iro as s ow . e e ma e ro a es e amp co ec e o e coi
ows.

so iro
core
coi

ma e
amp

axis o
ro a io

Fig. 11.1

(a) Exp ai w e amp ows w e e ma e ro a es.

……………………………………………………………………………………………….

……………………………………………………………………………………………….

……………………………………………………………………………………………….

……………………………………………………………………………………………

(b) escri e wo a era io s a cou e ma e o e par s o e appara us or e


amp o ow more ri .

……………………………………………………………………………………………….

……………………………………………………………………………………………….

……………………………………………………………………………………………

(c) ae e purpose o e so iro core.

……………………………………………………………………………………………….

……………………………………………………………………………………………

462
20

(d) e s ape so iro is rep ace a piece ma e rom woo exp ai i e amp
wi s i ow w e e ma e ro a es.

……………………………………………………………………………………………….

……………………………………………………………………………………………….

……………………………………………………………………………………………….

……………………………………………………………………………………………

(e) e coi o wire is e wou o o e par s o e so iro core us a o e a


e ow e ma e as s ow i i . . . e ma e is a ai ma e o ro a e as
s ow wi e po e mo i ou o e p a e o e paper a e po e mo i
i o e p a e o e paper a e i s a s ow i i . . .

so iro
core

A
irec io o
ro a io

ma e

Fig. 11.2

(i) escri e ow e ma e ic po es i uce i par o e core c a e uri


o e ro a io o e ma e .

……………………………………………………………………………………….

……………………………………………………………………………………….

……………………………………………………………………………………….

……………………………………………………………………………………….

……………………………………………………………………………………….

……………………………………………………………………………………

463
21

(ii) i . . s ec e aria io o e e.m. . pro uce e wee ermi a s


a uri o e ro a io o e ma e . ssume a is e a
rou ou e ro a io .
e.m. .

um er o ro a io s

Fig. 11.3

464
22

OR

B11 i . . s ows a ru er e a o e e o oa e c wi a s or e o e ru er
pro ec i ou a ei i ra e .

Fig. 11.4

(a) Exp ai ow e i ra i ru er pro uces sou i e surrou i air.

……………………………………………………………………………………………….

……………………………………………………………………………………………….

……………………………………………………………………………………………….

……………………………………………………………………………………………….

……………………………………………………………………………………………….

……………………………………………………………………………………………….

……………………………………………………………………………………………….

……………………………………………………………………………………………

465
23

(b) i . . s ows e i i ia posi io o some par ic es e ore i ra io e i sa a


i ere imes o e sou wa e.

2 3 4 5 6 7 8 9 10

initial position of particles before vibration begins

1 2 3 4 5 6 7 8 9
(a) time 0.00 s

2 3 4 5 6 7 8 9

(b) time 0.01 s

3 4 5 6 7 8 9 10
(c) time 0.02 s

3 4 5 6 7 8 9 10 11
(d) time 0.03 s

Line X

(e) time 0.04 s

scale: 4 cm
Fig. 11.5

(i) a cu a e e re ue c o e wa e mo io .

re ue c ........................................

466
24

(ii) i . . mar e amp i u e o e wa e mo io .

(iii) si e i e sca e e ermi e e wa e e o e wa e mo io .

wa e e ........................................

(iv) e ce ca cu a e e spee o e wa e.

spee ........................................

(v) i . . raw o i e e posi io s o par ic es a a


ime . s.

(vi) i . . s e c a isp aceme ime rap o par ic e rom ime s o


. s a i e isp aceme o e ri as posi i e. ow c ear a ues
o o axes.

isp aceme cm

ime s

Fig. 11.6

END OF PAPER

467
468
Solutions to 2018 4E Physics Prelim

Paper 1

Multiple Choice Questions 40 marks

1 11 21 31
2 12 22 32
3 13 23 33
4 14 24 34
5 15 25 35
6 16 26 36
7 17 27 37
8 18 28 38
9 19 29 39
10 20 30 40

Paper 2

Section A: Structured Questions 50 marks

Qn Solution Mark
A1(a)(i)

or er ica
arrows raw
o e rum

A1(a)(ii) i ere ce i p
pressure
re
ess
s ur
ure
e
hU g
. u . u u
Pa

rresu
esu
es u a orce
or
u .

A1(a)(iii) wei o rum a i s co e

a era e e si
m
V

469
. u .
m

A1(b)(i) e ermi e e ra ie a e s eepes par o e rap w ere


s cm.

A1(b)(ii) ere is wor o e a ai s e resis i e orce ue o wa er.

A2(a)(i) o a resis a ce
L
U
A
. u 
u
Su . u 

| . Ÿ

A2(a)(ii)
(a)(ii) V
curre
R

.
| .

A2(b)
(b)

or ie
pa er

or irec io
o ie

A2(c)(i)
(c)(i) e cu
curre
urr
rree ows
ow
ws rom
ro o .

A2(c)(ii)
(c)(ii) a i mom
mome
ome e s a ou e pi o
o a a ic oc wise mome s o a c oc wise mome s
Fu . u  u
F| . u 

A3(a) o uc io i o es i ra io s o par ic es wi ou a ow o
me ium w i e co ec io i o es mo eme s o mo ecu es ue
o i ere ce i e si

ccep o uc io occurs mai i so i s w i e co ec io occurs


o i i ui s a ases

470
A3(b)(i) e a is a oo co uc or o ea . ea rom e a is co uc e
awa easi o ouc i a emp me a cup e ce ee i co
w ereas ea rom e o co ee is co uc e rom e o co ee
o e a easi w e e cup co ai s o co ee e ce ee i
o.

A3(b)(ii)
(b)(ii) e ou e wa e ass as air i e wee e wa s. ir is a a
co uc or o ea a so ea rom e o ea is o easi eas
asii
ra s erre o e a e ce oes o ee o .

A4(a)(i)
(a)(i) E er Power x ime x x
s

A4(a)(ii)
(a)(ii) . ec s i e room o o re ease or a sor a erma
erm
rma e er
o or rom e room.
. e air co i io er is e icie .

A4(b)
(b) Q ml c ș
mc
m m or wor i
m m
m
m swer
or a swe

A5(a)
(a)
s or o or a
wa e e wa e e correc or er o
compo e s
infra-
gamma rays X-ray
X-
X-rays
r s ultrav
ultraviolet
aviol
io et visibl
vis
visible
ib e microwaves ra io wa es
red

10-14 m 10-100 m 10-88 m 10-66 m 10-55 m 10-22 m m


or a
correc
correspo i
wa e e

A5(b)
(b) . ms

A5(c)
(c) e are a ra s erse wa es carr o e ec ric c ar e o e e a o e
aws o re ec io a re rac io . proper

A5(d) auses io isa io i i i ce s es ruc io or mo i ica io o i i a o e


issues i ra re ea i e ec

471
A6(a)

o ec

mar or
eac ra o i
posi io o
o ec
o cm rrom
e s

A6(b)
(b) o er i e s am
amera
mer
era
a imi
imi is e rea i er e mar or
a corr
correc
P a e mirror same
sam
amee si e ir
ir ua upri correspo i
c arac eris ics
ic

A6(c)
(c) correc wo
P ra s o i o
e e
correc wo
ra s rom o ec
posi io o
o ec wi
correc ra s
cm cm raw rom
E em.
cm rom
mirror

472
A7(a) e e ro is rou ear o e a s mo es owar s e ro escrip io
a mo es owar s . e e e a i e c ar e ro is o posi io o
rou ear o e c ar es o are i uce wi e ai e e a s a
c ar es o repe e o e ar e earer o ea i e i e c ar es i
posi i e c ar es o earer o e ro . is is ue o i e c ar es a
repe . i e e a i e c ar es ear o e c ar es o are exp a a io
a so i uce suc a e e a i e c ar es i are repe awa wi c ear
rom ea i posi i e c ar es ear o ma i mo es co cep s
owar s as u i e c ar es a rac .

A7(b)(i) e e a i e c ar es o ow ow o ear a e posi i e iescri i


c ar es remai a rac e o e e a i e c ar es o e ro . o posi i e
a e ai e
c ar es o

A7(b)(ii)
(b)(ii) corr
correc
posi io s o
e a i e c ar e ro a a
correc
corr
co ec c ar es
rrec
Y s ow

A7(c)
(c) is posi i e c ar e a is eu ra . correc or
o
o a

A8(a)
(a) Q It
Q . u . u
Q

A8(b)(i)
(b)(i) o a resis a ce across
acr
cros
o s e
ea
a i sa si e i s

§ ·
¨ .  ¸
© ¹
. Ÿ

o a resis
is a ce
ce i circui
.  .
. Ÿ

o a curre i e a er

.
.

A8(b)(ii) e o a curre i e a er i creases rom . o more a


a ere is a ar er po e ia i ere ce across e .
resis or.
e ce e po e ia i ere ce across e ea i s a si e i s
ecreases a e power e e ope ecreases as we . e ce
e i s ecome ess ri .

473
A8(c) o a e er use
u
.  .
| .

o a e er supp ie u c ar e a er
. u u .
| .

e a er wi e ep e e a e ri er wi o e a e o s ar
e e i e.

Section
tion B: Structured Questions 30 marks

Qn Solution Mark
B9(a)
(a) mass o ue UV
u . u 

c emica e er u u
. u

B9(b)
(b)
wor o e Fddriving u . u

Fdriving u d . u

i ce car is ra e i a a co
o sa spee
spe
pee
e Fdrivin
driving
driving
ng Fdrag
d

ud . u
d| . u m

B9(c)(i)
(c)(i) u
cos u .
u
| .

B9(c)(ii)
(c)(ii) E P ut
u . u ut
t s
t| . ours

B9(d)(i) . u
mass re uire u
u

B9(d)(ii) wi a e a er ar e i er ia a is i icu o acce era e. a


reaso a e
a swer

474
B10(a)

. .

. .

or rawi i e a cu s rou a . .

B10(b) .  . or rawi
acce era io
.  . a e a
| . ms a e i
ra ie
coor i a es
or a swer

B10(c)(i) maximum resis i e orce W



u u
.

B10(c)(ii) W  FR m
ma
. u  FR . u .
FR | .

B10(d)(i) v u
a
t
 .

t
t . s

B10(d)(ii) is a ce ra e e
area u er v t rap uri ece era io
u . u .
. m

Either e e ma e ro a es ere is a i uce e.m. . e era e a


B11(a) e coi s pro uci a curre i e amp ue o e c a i
ma e ic ux i a e pro uce a e coi s.

475
B11(b) i more ur s o e coi or a
o a e e ma e as er a era io s
se a s ro er ma e

B11(c) e so iro core wi co ce ra e e ma e ic ie pro uce


e coi

B11(d) e amp wi s i ow u wea ecause ere wi s i e


c a i ma e ic ux i a e u poor ux i a e a e coi as
woo is o a so ma e ic ma eria .

B11(e)(i) s e po e mo es awa rom par e e o e par or e


ac uires po ari . correc po a ari
s e po e approac es is par i remai s or po e. w e mo i
s po e o e ma e mo es awa rom par e e o pa
parr awa or
ac uires po ari u i po e o e ma e reac es p
par
ar approac i
a ai . or e u
escrip io o
o e rro
o a io o
e ma e

B11(e)(ii)
1(e)(ii)
e.m. or corr
correc
s ape a
s mme r wi w
c a e i
irec io a er
a o e
um er o ro a io s
um ro a io

OR
R e e ru
ru e i rraa es e air par ic es are se i osci a io wi
1(a)
B11(a) e i ra io o air air par
par ic
ic es ear o e ru er a cause osci a io
i ea a ace
ce p par
ar ic
ic es.
ue o e i rra a io o par ic es i a irec io para e o e wa e
ra e some
some re re io s experie ce a i er e si a a i s a
w ieo e err re io s experie ce a ower e si .
is se s up re io s o compressio a rare ac io a sou
e er is ra smi e o e ears ears.

B(11)(b)(i) T . s
f
T .

476
B11(b)(ii) orrec i ica io o amp i u e i i . .

2 3 4 5 6 7 8 9 10

Initial position of particles before vibration begins

amp i u e
amp i u e

1 2 3 4 5 6 7 8 9
(a) time 0.00 s

2 3 4 5 6 7 8 9
(b) time 0.01 s

3 4 5 6 7 8 9 10

(c) time 0.02 s

3 4 5 6 7 8 9 10 11
(d) ttime
(d) ime 0.03 s

Line
Lin
ne X 3 5 7 9
(e) time 0.04 s

Scale: 4 cm

477
B11(b)(iii) a ee par ic e o par ic e s ows a wa e e
. m . m or cm

B11(b)(iv) e oci re ue c wa e e . . m s or cm s

B11(b)(v) orrec posi io s o par ic es a or a


correc
posi io s

B11(b)(vi) correc
isp aceme cm wa e orm

correc
a e i o
o axes
. . . ime s

478
479
NAME CLASS INDEX NO.

ST. PATRICK S SCHOOL


PRELIMINARY EXAMINATIONS 2018

SUB ECT : PHYSICS DATE : 11 SEPTEMBER 2018


6091/01

LEVEL : SECONDARY 4 EXPRESS DURATION : 1H

INSTRUCTIONS TO CANDIDATES:

DO NOT OPEN THIS BOOKLET UNTIL YOU ARE TOLD TO DO SO.

ere are forty ues io s i is paper. swer all ues io s. or eac ues io ere
are our possi e a swers A B C a D. oose e one ou co si er correc a
recor our c oice i soft pencil o e separa e OPTICAL ANSWER SHEET.

INFORMATION FOR CANDIDATES:

Eac correc a swer wi score o e mar . ar s wi o e e uc e or wro a swers.


rou wor i s ou e o ei is oo e . our o a score or Paper wi e e
um er o correc a swers i e .

This paper consists of 14 printed pages including the cover page.

480
1 Diagram I s ows e sca e o a microme er screw au e w e e ap is c ose .

Diagram II s ows e same microme er screw au e use o measure e iame er


o a rou pe e .

Diagram I
Diagram II

a is e iame er o e pe e

A . mm B . mm
C . mm D . mm

2 ow ca e perio ic ime o a simp e pe u um e si i ica re uce

A i creasi e mass o e pe u um.


B ecreasi e mass o e pe u um.
C i creasi e e o e pe u um.
D ecreasi e e o e pe u um.

3 ic o e o owi s a eme s co cer i mass a wei is are rue

I ass is co s a e er w ere.
II ei is a orce a pu s owar s e ce re o e ear .
III ei c a es rom p ace o p ace epe i o e acce era io ue o
ra i .

A Io B I a III o
C II a III o D I II a III

481
4 e mass o a paper c ip is . a e e si o i s ma eria is . cm . e
o a o ume o a um er o paper c ips is cm . ow ma paper c ips are ere

A B
C D

5 e ia ram s ows e spee


ime rap o a ru er. ms

a is e a era e spee o e
ru er e wee poi s X a Y

A . ms B . ms
C . ms D . ms

6 ow wi e acce era io ue o ra i e a ec e i a ea o ec is re ease rom


a i er posi io a i s mass is a e

Higher Position Halved its mass


A oc a e oc a e
B oc a e ecreases
C creases ecreases
D creases oc a e

7 o ec mo es i a s rai
ori o a i e u er a si e orce.
e ma i u e a irec io o e
orce c a e uri e mo io .
e e oci ime rap o e
o ec is s ow . ic is e
i er a w ere e orce i e
mos wor o e o ec

482
8 o u i orm ro o u ow wei W is suspe e wo s ri s as s ow i e
ia ram. e e sio i o e o e s ri s is .

a is e e sio T i e o er s ri

A . B .
C . D .

9 our o es A B C a D are ma e o a u i orm ami a. e ce re o ra i o e


ami a is a G. ic o e o e o owi s ows correc e ami a a i ree
a ou eac o e o es

A B

483
10 ic o e o owi i o es ac a e i e er o

I mass o raise er ica cm.


II mass o i creasi i s spee rom ms o m s.
III c ar e par ic e o acce era e usi a po e ia i ere ce o .

A Io B II o
C Ia III o D I II a III

11 rap is p o e o s ow e re a io s ip e wee e pressure exer e o e a e


a e ase area o e oc .

ic o e o e o owi rap s ows is re a io s ip

12 e a mosp eric pressure is Pa. a is e a era e orce experie ce a e


o om o e wa er a wi e . m wi . ma ep . m e si o
wa er is m.

A B
C D

13 ma ome er is p ace o a ramp


co ec i o a as supp wi
pressure P. e a mosp eric
pressure is H me res o mercur .

e e s Q a R are
measure i me res w a is e
pressure P i me res o mercur

A Exac R + H B Exac R Q H
C esser a R + H D esser a R Q H

484
14 ic o e o e o owi ia rams es represe s e o ser e pa s o c a
pow er suspe e i s i wa er

A B C D

15 room empera ure we ee a a me a oc is coo er a a woo e oor.

ic o e o owi s a eme s is are rue

I e empera ure o e me a oc is ower.


II e a is a e er co uc or o erma e er a woo .
III e ra s er o erma e er i me a is as er a woo .

A Io B I a III o
C II a III o D I II a III

16 a mass o o wa er a mus e mixe wi wa er a room empera ure o


i or er o o ai o wa er a

A . B
C D

17 poi source o i is oca e cm a o e a p a e mirror a is re ec e ra is


e ec e cm a o e e mirror a a ori o a isp aceme o cm.
source

detector

3 cm
2 cm

A 12 cm B

a is e is a ce e wee A a e poi o i ci e ce

A cm B cm
C . cm D . cm

485
18 e ia ram s ows wo ra s o i e eri a ass oc .

ic o e o e o owi e ua io s is correc

A w y B in w in y
x z in x in z

C w x y z D w x y z

19 a o ec is p ace cm rom a co er i e s e ima e orme is s i


sma er a e o ec . a is e approxima e oca e o e e s

A cm B cm
C cm D cm

20 e a sou wa e passes a air mo ecu e i e irec io s ow w ic ia ram


s ows e correc mo eme o e air mo ecu e

irec io o
sou wa e
air mo ecu e

A B

C D

486
21 e ia ram s ows a s aps o o a ra e i wa e a is mo i rom e o ri .
e re ue c o e wa e s ow is .

a i eac ri o e cm cm w ic o e o owi correc s a es e


amp i u e a e spee o e wa e

Amplitude / mm Speed / m/s


A .
B .
C .
D .

22 ripp e a is se up so a ere are i ere e e s o wa er as s ow i e


ia ram.

R
shallowest
vibrator

deep shallow shallower

e i ra or pro uces s rai wa es w ic o e o e o owi ia rams correc


s ows a ripp e approac i e re io R

A B

C D

487
23 ic o e o e o owi s a eme s a ou e isi e spec rum is not true

A i rom e su oes o pro uce a pure spec rum.


B e isi e spec rum i c u es i ra re a u ra io e ra s.
C rai ow is ac ua a spec rum cause e ispersio o i rom wa er
rop e s.
D is see w e w i e i passi rou a prism is isperse i o i s
compo e co ours.

24 e is p a er i s a a ar a ears a ec o rom a wa . s a er. e spee


o sou i air is m s. ow ar awa is e p a er rom e wa

A m B m
C m D m

25 ic o e o owi is are app ica io s o u rasou

Pre a a sca i o oe us
ea i o sur ica i s rume s
si a microp o e uri ec ure

A Io B II o
C Ia II o D I II a III

26 Figure I s ows wo me a ic sp eres X a Y p ace i co ac . posi i e c ar e


ro Z is e rou ear em.

X Y Z X Y Z

Figure I Figure II

Y is ear e wi ou remo i Z as s ow i Figure II w ic o e o e o owi


s a eme s is correc

A o X a Y wi e e a i e c ar e .
B o X a Y wi e posi i e c ar e .
C X wi e posi i e c ar e a Y wi carr o c ar e.
D X wi carr o c ar e a Y wi e e a i e c ar e .

488
27 a er ri es o c ar e rou a circui . e o a wor o e is . a is
e e ec romo i e orce o e a er

A . B
C D

28 esis ors o a are


co ec e i series o a
supp as s ow i e circui .

a is e e ec o c osi e
swi c o e curre raw rom
e supp a e po e ia
i ere ce across e

Current Potential Difference


A ecreases . ecreases
B c a e creases
C creases . creases
D creases . creases

29 ic o e o e o owi ia rams s ows e correc co ec io s or a swi c a


amp i a i i circui Ke i e eu ra E Ear a me a casi

A B

C D

489
30 e ec ric e e a e e is i e wi a p u co ai i a use.

is co ec e o a mai s supp . e wa er i i a es a ew mi u es o oi .
e e p u is co ec e o a mai s supp

A e e e oes o wor .
B e use i epu ows.
C e wa er a es a o er ime o oi .
D e ea i e eme o e e e me s.

31 e ec rica ea er is use o e ermi e e speci ic ea capaci o a me a a e


o owi rea i s are o ai e

mass o me a
ime or w ic ea is supp ie s
empera ure rise o me a

is ou a e speci ic ea capaci o e me a is .

ssumi a o e e er supp ie e ea er is os o e surrou i


w a is e power ra i o e ea er

A B
C D

32 e es wa o ema e ise a ma e is o

A row i o e rou se era imes.


B p ace i ao eE axis a ammer i .
C p ace i i a so e oi carr i irec curre a s ow pu i i ou .
D p ace i i a so e oi carr i a er a i curre a s ow pu i i ou .

490
33 e ia ram s ows a s ie esi e o pro ec a ca o e ra u e rom e e ec s
o ex er a ma e ic ie s.

ic o e o e o owi is use o ma e e s ie

A ee B opper
C ica D o iro

34 eu ro e ers a u i orm ma e ic ie perpe icu ar as s ow i e ia ram.


xxxxxxxxxxxxxxxx
xxxxxxxxxxxxxxxx
xxxxxxxxxxxxxxxx
x x x x x x x x x x x x x x x x u i orm ma e ic ie
eu ro xxxxxxxxxxxxxxxx
xxxxxxxxxxxxxxxx
xxxxxxxxxxxxxxxx

e pa a is race ou wi e ecause .

A s rai ere is o orce ac i


B circu ar e orce is a wa s orwar
C spira e orce is a wa s perpe icu ar o i s mo io
D para o ic e orce is a wa s perpe icu ar o i s mo io

35 wo o i su a e wires carr i equal


curre s are p ace perpe icu ar o
eac o er as s ow .

e poi s P Q R a S are a a e ua
is a ces rom e wires.

w ic poi s is are e resu i


ma e ic ie zero

A Po B Qo
C Pa R D Pa S

491
36 ic par o a simp e .c. mo or re erses e irec io o curre rou e coi
e er a c c e

A e arma ure B e rus es


C e sp i ri commu a or D e s ip ri s

37 ar ma e is e a o e a oop o wire i e posi io s ow e ow. o e oop


a ma e are mo e si ewa s wi e same u i orm spee .

ic o e o owi rap s best represe s e aria io o e ma iu eo e


e.m. . i uce i e oop

A e.m.f. / V B e.m.f. / V

t/s

t/s

C e.m.f. / V D e.m.f. / V

t/s

t/s

38 e ia ram s ows a i su a e copper


wire coi e arou a so iro ro .
copper ri R is p ace a o e e o e
ro . a wi appe o e copper ri
Rw e e swi c S is c ose

A e ri ecomes ea e up.
B e ri ecomes ma e i e .
C e ri wi e a rac e o e coi .
D e ri wi e repe e rom e coi .

492
39 Diagram I s ows e osci oscope race pro uce a i pu o a .

Diagram I

Diagram II s ows e race rom a i ere i pu o e same osci oscope.

Diagram II

a is e a ue o e ew i pu

A a B a
C a D a

40 ic pairs o ua i ies are not propor io a

A a e i empera ure a ua i o ea supp ie o a o ec


B pee a ime or a o ec a i ree rom res rou a acuum
C urre a po e ia i ere ce or a i u w e empera ure rises
D esu a orce ac i o a o ec a o a s rai i e a i s acce era io

END OF PAPER

493
NAME CLASS INDEX NO.

ST. PATRICK S SCHOOL


PRELIMINARY EXAMINATIONS 2018

SUB ECT : PHYSICS DATE : 20 AUGUST 2018


6091/02

LEVEL : SECONDARY 4 EXPRESS DURATION : 1 H 45 MIN

INSTRUCTIONS TO CANDIDATES:

DO NOT OPEN THIS BOOKLET UNTIL YOU ARE TOLD TO DO SO.

. is paper co sis s o Two (2) Sections Section A a Section B.


. Section A: swer ALL ues io s. ri e our a swers i e spaces pro i e .
. Section B: swer ALL ues io s.
ues io 11 is EITHER / OR QUESTION. SELECT ONLY ONE PART OF THIS
QUESTION.
. a cu a ors ma e use w ere ecessar .
P PE
. DO NOT DETACH a sec io s rom is paper.
P PE
INFORMATION FOR CANDIDATES:
E
rou ou e paper e acce era io ue o ra i
o Ear is a e as unless stated otherwise. E

e um er o mar s is i e i rac e s a e ues io


e o eac ues io .
ues io
ou are a ise o spe o o er a o e our o
ec io a o o er a mi u es o ec io . ues io E

TOTAL
80 ei a e
Paper
20 ei a e
Paper

GRADE
Pare s i a ure
TARGET GRADE

This paper consists of 18 printed pages including the cover page.

494
SECTION A : 50 marks

Answer ALL questions in this section. Show your working and write your answers in the
space provided.

1 Diagram I s ows a a e w ic c oses au oma ica a er use.

Diagram I Diagram II

ea s o e is a ac e c ai s K a KL o e op ar o e a e
a o e op o a ear er ica pos . pe i e a e raises e s o e.
e e a e is re ease e orce exer e e c ai s K a KL
c oses e a e.

(a) Diagram II s ows e i s a w e e a e is c ose a eac


c ai is a o e er ica . e p a e co ai i e c ai s a
e s o e is a ri a es o e a e. mea s o a sca e ec or
ia ram e ermi e e e sio i ca e K i e wei o eso e
is . ou are o i ica e e sca e use .

ca e use e sio i ca e K 4

495
(b) Diagram III s ows e op iew o e a e.

Diagram III

ue o e e sio i c ai K ere is a ori o a orce o


a o s e a e c ose as s ow i e ia ram. orce F is
app ie o e a e o ope i .

(i) mea s o a arrow i ica e i Diagram III w ere e


orce F s ou e app ie so a i s ma i u e is a mi imum.
a e e orce as F. 1

(ii) a cu a e is mi imum orce F i (b)(i) ee e o us ope


e a e.

i imum orce F 2

496
2 (a) e i e work a i e e ame a s m o or a u i i w ic i
is measure .

i o wor m o 2

(b) o pus s ar a car o a wi er s mor i i s a er is a wo


peop e eac pus es wi a orce o . er pus i or m
e car s e i e s ar s. a e mome i s i e ic e er is
ca cu a e

(i) e wor o e e pus o e peop e o e car a

or o e 2

(ii) e a era e ric io a orces o e car.

era e ric io a orces 3

497
3 (a) (i) is o ser e a so i s a e ixe s ape w i e i ui s o
o . si e proper o mo ecu es exp ai e o ser a io .

(ii) a proper o mo ecu es ma es ases i a e space


a ai a e o em

(b) Exp ai ow mo ecu es escape rom e sur ace o a e apora i


i ui .

(c) Exp ai ow e apora io a ec s e i er a e er o e i ui .

498
4 e wa e e o our e ec roma e ic wa es i c u i i rare wa e is
i e i e o owi a e.

Type of Wave Wavelength

rare wa e . mm

A . m

B cm

C m

(a) ic o e wa es A B or C is isi e i

(b) s a io roa cas s i s pro rammes a .

(i) ame e e ec roma e ic wa e use or e roa cas .

(ii) a cu a e e wa e e o e wa e.

a ee 2

(c) ars emi a pes o e ec roma e ic wa es. e escopes a


mo i or ra s are mou e o sa e i es i space. wou a
ra e escope ase o Ear not e a e o e ec ra s emi e
rom is a s ars

(d) a e o e simi ar proper ies o a e ec roma e ic wa es.

499
5 e ia ram s ows e pa o a ra o ue i as i passes rou a
ass prism.

(a) si e a es s ow i e ia ram ca cu a e e re rac i e


i ex o e ass.

e rac i e i ex 2

(b) Exp ai w e ra oes o emer e rom e prism a B.

(c) e o owi ia ram a seco ra o ue i s ri es e ass


prism a C. is ra is para e o e ra a s ri es e ass prism
a A. is ia ram co i ue e pa o e seco ra rou
a ou o e ass prism.

500
6 e ia ram s ows our s a es o ow a p o ocopier wor s.
Posi i e c ar e
paper

Posi i e c ar e
ima e

e our s a es are a e e rom A o D u are o arra e i or er.

e o owi a e s ows e irs s a e wi a rie escrip io o w a


appe s uri is s a e wi i e p o ocopier.

Stage Brief Description of the stage


i is s o e o a ocume a e w i e areas
re ec i o o e rum w ic is posi i e c ar e .
D reas o e rum a are s ruc e i are
isc ar e a ea es a posi i e c ar e ima e o
e rum.

(a) omp e e e a e

(i) arra i e o er s a es i correc or er. 1

(ii) pro i i a rie escrip io o eac s a e. 3

(b) Exp ai w e paper i s a e A is more posi i e c ar e a


e ima e orme i s a e D.

501
7 e rap s ows ow e curre I i a io e aries wi e po e ia
i ere ce V across i .

(a) escri e ow I aries wi Vw e

(i) V is e a i e

(ii) V is posi i e.

(b) a cu a e e resis a ce o e io e w e V is . .

esis a ce 2

(c) s u e s a es w e V is e a i e e resis a ce is ero . ae


wi a reaso w e er ou a ree wi esu e .

502
8 e ia ram s ows a power s a io w ic e era es o power.

e o a e is s eppe up o a e a io a ri ra smi s e
power o er a ar e is a ce. e o a e is e s eppe ow e ore e
power is use i us ries a omes i a ow .

(a) i e a is e i o e i e a ca cu a e e
curre owi i e ra smissio i e.

urre 2

(b) a cu a e e power oss a o e ra smissio i e i e a e


o a resis a ce o e i e is .

Power 2

(c) ow wou ou mo i e circui so a e power oss i e circui


ca e re uce Exp ai our a swer.

503
SECTION B : 30 marks

Each question is worth 10 marks. Answer ALL questions in this section. Question 11 is
an EITHER / OR QUESTION. SELECT ONLY ONE PART OF THIS QUESTION. Show
your working and write your answers in the spaces provided.

9 e ia ram s ows e p a o a e room a par o e mai room o a


ouse. er rooms are o s ow .

e empera ures o e mai room e e room a e ou si e o e


ouse are s ow .

e o owi a e s ows a e erma e er i pu s o e e room i


o e our.

Thermal energy input to bedroom Energy /

rou e oor a wa s rom mai room .

rou e wa s rom ou si e o ouse .

rou e wi ow .

rom e perso s eepi i e room .

(a) Exp ai w more erma e er e ers e e room rom e


ou si e o e ouse a rom e mai room.

504
(b) air co i io er eeps e empera ure co s a i e e room
remo i e er .

e i a sui a e oca io o e air co i io er i e e room a


exp ai ow i coo s e e room e icie .

(c) e i si e wa o e e room ra ia es erma e er . Exp ai


ow e co our o e i si e wa a ec s e amou o e er
ra ia e .

(d) (i) a is e ra e o erma e er remo e e air


co i io er i e empera ure o e room is o e mai ai e
a

ae 3

(ii) a e a assump io ma e i e ca cu a io or d(i).

505
10 e ia ram s ows a e ec ric s a er.

co sis s o e s a er a a c ar i u i. o e s a er a e
c ar i u i are comp e e co ere p as ic cases a ere is o
me a co ac e wee em. si e e s a er ere is a
rec ar ea e ce or ri i a mo or.

(a) e e s a er is i opera io e curre owi rou e


mo or is . . a cu a e e power co sume e mo or.

Power 2

(b) e e e er s ore i e ce is use up e s a er is p ace


o e c ar i u i o rec ar e e ce . e c ar i u i is
co ec e o e mai s supp a i s a e is s ow i e ia ram.

a cu a e e e er i ou es raw e c ar i u i rom e
mai s supp i o e our.

E er 2

506
(c) o c ar e e rec ar ea e ce e s a er wi e p ace o e
c ar i u i. e o om o e s a er is coi X w ic is
co ec e o e rec ar ea e ce . o er coi Y is oca e
i si e e c ar i u i wi a so iro ar ixe i si e i . e e
s a er is p ace o e c ar i u i e so iro ar ies i si e
coi X.

(i) Exp ai ow a e ec romo i e orce e.m. . is pro uce i e


s a er o rec ar e e ce .

(ii) ou pu o a e o a.c. is pro uce a coi X o c ar e


e ce . a cu a e e um er o ur s i coi X i ere is
ur s i coi Y.

um er o ur s 2

(d) e c ar i u i is co ec e o e mai s wi a wo pi p u .
a e a reaso a exp ai s w i is sa e o use a wo pi p u .

507
11 EITHER

e ia ram s ows a o o mass ri i o a s a e scoo er.

e pus es o wi is rear oo mome ari cruises or a w i e a e


pus es o wi rear oo a ai . e c c e is repea e . ere is prese ce
o resis i e orces.

e o owi rap s ows ow e e oci o e o c a es o er e


irs . s o is our e .

(a) escri e e o s acce era io o er e irs . so is our e .

508
(b) ic par o e rap s ows a ere is prese ce o resis i e
orces Exp ai our a swer.

(c) . s ca cu a e

(i) e oa isp aceme o e o

oa isp aceme 1

(ii) e acce era io o e o

cce era io 1

(iii) e orwar ri i orce app ie e o i e oa


resis i e orce ac i o im is . .

orwar ri i orce 3

(d) a e e o er orce a is par o a ac io reac io pair wi e


orwar ri i orce ca cu a e i c(iii).

509
11 OR

e ia ram s ows a ma e ic re a use o opera e e swi c o a


mac i e i a ac or .
i o a e

ac i e

(a) Exp ai ow e ma e ic re a wor s w e e empera ure arou


e ermis or decreases.

(b) (i) e ia ram i ica e e N-pole o e e ec roma e. 1

(ii) u es a sui a e ma eria or e pi o ar S. i e a reaso


or our c oice.

510
(c) (i) Exp ai c ear e a a a e o usi a ma e ic re a o
opera e e mac i e

(ii) e mac i e opera es a a ru s a a era e


o ours ai . a cu a e e cos o opera i e mac i e
i a mo a s i eac u i o e ec rici cos s . .

os 2

END OF PAPER

511
512
PHYSICS 6091 20 AUG
AU 2018 (P2
AU (P2)
11 SEPT
SEPT 2018 (P
(P1)
SECONDARY 4 EXPRESS

PAPER 1 40 marks (DURATION: 1 HOUR)

C D D D A A B A A C

D D D A C D C B B C

A B B A A C C C B C

C D D A C C A D C C

513
PAPER 2 (DURATION: 1 HOUR 45 MIN)

Section A 50 marks

u es e swers u
1a
a 4(M) 7

. .

ui a e sca e
ape o ec or ia ram
a e i orces
e sio i caa e K . .

1bi
bi 1(E)
F

Diagram III

1bii a i mome s a ou i e 2 (M)

o a c oc wise mome s o a a i c oc wise mome s


. .

514
u es e swers u
2a or is e e er ra s erre w e a orce mo e a 2 (E)
(E) 7
o ec o er a is a ce i e irec io o e orce.

i o wor ou es m o

2bi orce is a ce mo e 2 (M)


( )
(M

2bii a ai s ric io 3 (D)

orce is a ce mo e

orces
era e ric io a orces
rces .

515
u es e swers u
3ai o i s a e ixe s ape as e mo ecu es are i ra i 2 (M) 7
a ou eir ixe posi io .
i ui s o o a e ixe s ape as e mo ecu es are a e
o s i e pas o e a o er.

3aii e wea i ermo ecu ar orce e wee e as mo ecu es. 1 (E)

3b e mo ecu es are a co s a ra om mo io a e 2 (E)


co i e wi o e a o er. imes some mo ecu es
ai e e ou e er o rea e i ermo ecu ar o s
e wee em a escape o e surrou i as
apour.

3c uri e apora io some mo ecu es wi more e er 2 (M)


(M)
escape rom e sur ace o e i ui . e remai i
mo ecu es as ess i e ic e er . us e i eerr a
e er o e i ui wi ecrease .

u es e swers u
4a 1 (E)
(E
E) 6

4bi a iowa es 1 (E)

4bii 2 (M)
m or a or
o
m

4c ra s are a sor e e Ear


ar s a mo
mosp ere. 1 (M)

4d o e o owi
ow 1 (E)
e are ra s erse e wa
w es.
es.
e ra e a e samesam
amee sp
spee o msi acuum.
e o o re e uire
uirre a me ium o ra e .
u
e o o carr
c rr a c ar e.
ca
e are o a ec e e ec ric ie .

516
u es e swers u
5a 2 (E) 6

. s

5b i is ra e i rom op ica e ser me ium ass o 2 (M)


( )
(M
a op ica ess e se me ium air a e a e o
i ci e ce is rea er a e cri ica a e o a i er a
re ec io a es p ace .

5c 2 (D)
(D
D)

e rac io a wi ra para e o
a re rac io

orma s ou e raw a e su
ssurr aces
ace e uc m i
o o o e

u es e swers u
6ai DBAC 1 (M) 5

6aii 3 (D)
e a i e c ar e o er par ic es are e a rac e o
e posi i e c ar e ima e.

e o er is ra s erre a rac e o e paper w e e


rum ro s o e posi i e c ar e paper.

ea is app ie o e o er pow er o ix e ima e o o


e paper.

6b o a e e ai e c ar e o er is a rac e o e 1 (D)
paper i s ea o s ic i o e rum.

517
u es e swers u
7ai e is e a i e e curre is ero. 1 (E) 6

7aii e is e wee o . e curre is ero. 2 (D)


e is rea er a . e curre aries rom m
o m .

7b rom e rap w e is . is m . 2 (M)

esis a ce o e io e

. x
.

7c o. e is e ai e a is ero i mea s a e 1 (M)


(M)
resis a ce mus e er i suc a o cu
ccurre
urr
rre c
ca
a
pass rou .

u es e swers u
8a P 2 (E) 6
I

100 0000 000


1
400 000
250A 1

8b Plo I2R 2 (M)

250 2 u 100
100 1
6 25 1

8c ep up e o a e o more a o re uce e 2 (D)


curre owi
ow
wi rou e ra smissio i e. s power oss
i e circui is wi re uce curre power oss is
re uce .

518
Section B 30 marks

u es e swers u
9a ere is rea er empera ure i ere ce o q e wee 1 (M) 10
ou si e e ouse a e e room a a e wee e
mai room a e e room w ic is a empera ure
i ere ce o q .

9b ee s o e p ace a e op o e e room. 3 (S)

e air arou e air co i io er wi e coo e ecomes


more e se a si s o e o om o e room.

e warmer air ei ess e se wi rise o e op o e


room o e coo e .

co ec io curre is crea e w ic coo


e ps o co
oo e
room e icie .

9c e i si e wa o e e room s ou e pai
aii e w i e . 2 (M
(M)
i e is a poor emi er o ra ia ea .

9di our o a ea e eri e e room 3 (D)


. . . .
.

aeo erma e er remo e air co


e air c i io er
. s
.

9dii reasoo a e assump


assum
umpp io 1 (D)
x ere
e is o ea ea e eriei
er e room o er a w a is
sae .
x e wi ow a oor is ep c ose rou ou .
x e empera
emp
e m er
mperaa ure ou si e e e room remai s as s a e .

519
u es e swers u
10a Power co sume e mo or 2 (E) 10

.
.

10b E P 2 (E)
(E)

10ci e e c ar i u i is co ec e o e m mai
ai s supp a 3 (D)
a er a i curre ows rou coi . is a er a i
curre wi pro uce a c a i ma e ic ie . ccor i
o ara a s aw is c a i ma e ic ie i i coi
wi pro uce a e.m. . i coicoi w iic
c w wii pro uce a
i uce curre o rec ar e e ce e .

10cii s p s p 2 (M)
s
s urr s

10d e c ar i u i as
as o
ou e i su a io or as comp e e 1 (S)
i su
u a e p as ic.
ic.

520
Either

u es e swers u
11a rom s o . s Posi i e u i orm co s a acce era io 2 ((M)
M) 10
rom . s o . s e a i e a o u i orm acce era iio
o .

11b rom . s o . s . s o . s a . s o . s ep par


ar 2 (M
(M))
w e e o is cruisi . uri ese imi s e o is
s owi ow s owi a pre
ese
ere is prese e cce
e o rresis
esis
es is i e
orces i e ric io a air resis a ce o oppose
oppos
osse isis mo
mo io
io .

11ci o a isp aceme 1 (M)


(M)
u . u
. m

11cii cce era io 1 (M)


u
.
ms

11ciii esu a orce


orc
rce
e 3 (D)
ma
u

.
.
o s

11d e orce app iie e o o e rou . 1 (M)

521
Or

u es e swers u
11a e e empera ure o e ermis or ecreases i s 3 (M) 10
resis a ce i creases.
e o a e across e ermis or i creases a caus
ca uses
us
causeses a
ar er curre o ow rou e so e oi .
e e ec roma e ecomes s ro ma
a e is
iise
se a
a rac s o c ose e swi c .

11bi op o e iro core is po e. 1 (M


(M)

11bii ro 2 (E)
ca e ma e ise a ema e ise easi .

11ci e ma e ic re a pre e s e us
user romm iire
irec
rec
c co
co ac wi 2 (M)
i o a e o power supp .
e ce i pre e s mi imises e rris
is o e e
ec
c ric s oc .

11cii o a e er use 2 (M)


u u u

os o e er
u .

522
523
ame ass ex um er

UNITY SECONDARY SCHOOL


PRELIMINARY EXAMINATION 2018

SECONDARY FOUR EXPRESS

PHYSICS 6091/01 12 SEPTEMBER 2018

PAPER 1 1 HOUR

Additional Materials : Optical Answer Sheet

READ THESE INSTRUCTIONS FIRST


. is paper co sis s o 40 u ip e oice ues io s.
. swer a ues io s o e p ica swer ee OAS .
. ri e our ame c ass a s a e our re is er um er i e spaces o
e OAS
. o o o or use a correc io ui o e OAS. ea e i s ruc io s o
e OAS care u .
. e o a mar or is paper is mar s.

is paper co sis s o 16 pri e pa es i c u i is co er pa e.

ec E P sics P Pa e o Pre im

524
Section A

swer ALL e ues io s i is sec io .

1 e iame er o a c i rica pe ci is measure usi a microme er screw au e.


s u e i i ia a es e ero error rea i ia ram o e microme er o owe
e rea i o e iame er ia ram .

Diagram A Diagram B
a is e ac ua iame er o ec i rica pe ci

A . mm B . mm C . mm D . mm

2 e ia ram s ows a simp e pe u um. swi s e wee A a B. e perio s o


osci a io or i ere e s o pe u um are recor e i e a e e ow.

e o pe u um m . . . .
perio o osci a io s . . . .

A B

e e o e pe u um is . m w a is e approxima e ime a e or
osci a io s
A . s
B . s
C . s
D . s

ec E P sics P Pa e o Pre im

525
3 e i ure e ow s ows wo orces ac i a a poi Y w ic are represe e YX
a YZ respec i e .

W Z

X Y

ic op io represe s a ir orce a is re uire o mai ai e ui i rium


A YW
B WY
C XZ
D ZX

4 a is row er ica upwar s rom P. reac es e rea es ei a Qa e


a s ac o P w ere i is cau .

e ec i air resis a ce w ic o e o owi s a eme s is CORRECT


A cce era io a Q is ero.
B e ime o rise rom P o Q is rea er a e ime o a rom Q o P.
C e oa isp aceme o e a is ero.
D e a experie ces a s ea ecreasi upwar orce w e i rises rom P o
Q.

ec E P sics P Pa e o Pre im

526
5 car o a o a mass o is mo i a o a roa a co s a spee o m s.
su e ra es or s e ore i comes o a s op i ro o a ra ic i .
ssumi a e ra i orce a e ece era io o e car are co s a s w a is
e ra i orce exer e o e car
A
B
C
D

6 cm mo e copper o e si . cm is mixe orou wi o mo e


i o e si . cm o orm a ro e a o .
a is e a era e e si o e ro ea o
A . cm
B . cm
C . cm
D . cm

7 roc is ei u ree o e oo . e a as ro au pus es e roc


upwar s e wi ee a .
A e roc is easier o e pus e a o e Ear
B e roc is more i icu o e pus e a o e Ear
C e roc re uires as muc e or o e pus e as o e Ear
D e roc re uires o e or o e pus e

8 orce is app ie o pus a ox a o a m ramp o reac a ei o


. m a o e i s s ar i posi io . ssume ra i a io a ie s re g .

. m

a is e wor o e a ai s ric io
A
B
C
D
ec E P sics P Pa e o Pre im

527
9 rec a u ar oc o ime sio s mx mx m exer s pressure o o a ar
rou . ssume ra i a io a ie s re g .
a is e LEAST pressure a e rec a u ar oc ca exer o e rou
A . Pa
B . Pa
C . Pa
D . Pa

10 e ia rams e ow s ow air rappe i wo capi ar u es A a B a sma co um


o mercur as s ow . e e o e mercur co um is cm. e a mosp eric
pressure is cm .

rappe air
cm co um
o mercur

cm co um
o mercur
rappe air
capi ar u eA capi ar u e B
a is e pressure o e rappe air i capi ar u es A a B

air pressure in air pressure in


capillary tube A capillary tube B
A cm cm
B cm cm
C cm cm
D cm cm

11 m u i orm woo e ru e wei i is ei i e er ica up a orce F a


e cm mar as s ow .

cm F
cm
mar mar

cm
mar
a is e MINIMUM orce F re uire o i e ru e
A . B .
C . D .
ec E P sics P Pa e o Pre im

528
12 e ia ram s ows a a a ci o pi o e o a s a . e o is i e s i i
oes o opp e u re ur s o i s ori i a posi io .

pi o

wei

ere is e posi io o e ce re o ra i o e o
A a o e e pi o
B e ow e pi o
C exac a e pi o
D i si e e wei

13 as o e same o ume as a i ui expa s as er w e ea e


ic o e o owi reaso s is CORRECT
A e as mo ecu es are i er.
B e as mo ecu es expa as er.
C e orces e wee as mo ecu es are wea er a ose e wee e i ui
mo ecu es.
D e as mo ecu es co i e more re ue wi eac o er.

14 e row ia mo io experime i o i smo e par ic es ea par ic es se e


uic u er sma par ic es remai suspe e or o perio s o ime.
ic o e o owi reaso s is CORRECT
A e air pressure as a rea er e ec o er sma par ic es.
B er sma par ic es a e ow i er ia a are easi a ec e e
om ar me s o e air par ic es.
C e Ear s ra i a io a ie oes o ac o e er sma par ic es.
D e er sma smo e par ic es as e same e si as e air.

15 i ui i ass ermome er as a mercur e e o . cm a q a a mercur


e e o . cm a q .
a is e mercur e e w e e empera ure is q .
A . cm B . cm
C . cm D . cm

ec E P sics P Pa e o Pre im

529
16 e ermocoup e is a e o measure a empera ure as i as q .
ic o e o owi reaso s is are CORRECT
i speci ic ea capaci
ow ea capaci
i me i poi
A o
B o
C a o
D a o

17 ic o e o owi s a eme s a ou e acuum as is INCORRECT


A ea oss ra ia io is mi imise eepi e o wa er i a ou e wa e
co ai er.
B ea oss is mi imise p aci a cor or p as ic s opper o c ose up e ec
o e co ai er.
C e acuum i e ou e wa co ai er e ec i e pre e s co uc io a
co ec io .
D e wa s o e co ai er are si ere o re uce ra ia io .

18 ic o e o owi s a eme s is are CORRECT


E ec rica co uc ors are usua oo co uc ors o ea .
o uc ors o ea a e ree e ec ro s o ra smi ea uic .
E ec roma e ic wa es ee e ec ro s o ra smi e er .
A o
B a o
C a o
D a

19 e speci ic ea capaci or ice a wa er is i e as . a .


respec i e . e speci ic a e ea o usio o wa er is i e o e .
a is e o a amou o e er ee e o raise . o ice rom  o

A
B
C
D
ec E P sics P Pa e o Pre im

530
20 E apora io is a wa s accompa ie coo i .
ic o e o owi reaso s is CORRECT
A e air mo ecu es coo e i ui sur ace.
B e more e er e ic mo ecu es ea e e i ui .
C ere are ewer mo ecu es e i e i ui .
D e escape mo ecu es re ur e o e i ui .

21 ra s isi e i a u ra io e ra ia io are a par o e e ec roma e ic


spec rum.
ic o e o owi escri es e CORRECT or er i i creasi wa e e
A ra s isi e i a u ra io e ra ia io
B ra s u ra io e ra ia io a isi e i
C isi e i ra s a u ra io e ra ia io
D u ra io e ra ia io ra s a isi e i

22 a are e CORRECT c a es o e re ue c wa e e a spee o e wa er


wa es w e e mo e rom s a ow wa ers o eep wa ers

frequency wavelength speed


A i creases remai s e same i creases
B remai s e same ecreases ecreases
C remai s e same i creases i creases
D ecreases remai s e same ecreases

23 i sec ma es a sou a is i er a e maximum au i e re ue c o a


orma perso .
i e a e spee o e sou i air is m s w ic o ese wa e e so e
sou is ma e e i sec
A . m
B . m
C m
D m

ec E P sics P Pa e o Pre im

531
24 a experime o measure e spee o sou i air a ir s a s m awa rom
a wa a c aps er a s o e er o pro uce a sou . e i s a w e s e ears
a ec o s e c aps er a s a ai . e oes is or c aps. e o a ime a e
or c aps is s.
ic o ese ca cu a io s i es e spee o sou i air

25 ra o i ra e s rom acuum i o ass.

spee

pq

acuum qq

ass sq

rq

spee

a is e re rac i e i ex o e ass
A si p q si s q
B si q q si r q
C
D

ec E P sics P Pa e o Pre im

532
26 ra o i is i ci e o o e si e o a rec a e ass oc . ea eo
re rac io is i e ass. e cri ica a e or i i ass is .
ic ia ram s ows e pa o is ra

A B

C D

27 e e o ec is p ace a cm rom e op ica ce re o a co er i e s e


ima e orme is rea i er e a ma i ie . e e o ec is p ace a cm rom
e op ica ce re o e same co er i e s e ima e orme is rea i er e a
imi is e .
a is e oca e o e co er i e s

A cm
B cm
C cm
D ca o e e ermi e

ec E P sics P Pa e o Pre im

533
28 wo eu ra co uc i a s X a Y are suspe e i su a i rea s rom e
cei i as s ow . e are separa e a s ee o i su a or. X is ouc e a ro
w ic carries posi i e c ar es.
i su a or

X Y
ic o e o owi ia rams es represe s e c ar e is ri u io o em

A B

X Y X Y

C D

X Y X Y

29 P Q R a S are our i e ica i su a e me a sp eres. e are arra e as s ow


e ow. posi i e c ar e ro is rou ear o P. p ere S is ear e mome ari .

P Q R S

e ro is remo e w a wou e e c ar e o sp eres R a S

R S
A posi i e posi i e
B posi i e e ai e
C e ai e posi i e
D e ai e e ai e

ec E P sics P Pa e o Pre im

534
30 e rap e ow s ows e re a io s ips e wee e po e ia i ere ce a e
curre o our i ere co uc ors W X Y a Z.

o a e
W
X
Y
Z

curre
ic co uc or as e i es resis a ce
A W
B X
C Y
D Z

31 a is e e.m. . o e ce i a e ec rica circui i i uses o e er o se


o curre arou e circui or mi u es
A .
B .
C
D

ec E P sics P Pa e o Pre im

535
32 e ia ram s ows a circui wi a po e ia i i er oi e i series wi a ixe resis or.

a are e mi imum a maximum rea i s w ic ca e o ai e o e


o me er

minimum voltage/ V maximum voltage/ V


A
B
C
D

33 ree i e ica i ame u s are co ec e o a .c. supp as s ow i e i ure


e ow. Eac u opera es a orma ri ess a e amme er re is ers a s ea
curre . i ame i o e o e u s i e circui rea s.

a appe s o e amme er rea i a e ri ess o e remai i u s

ammeter reading / A brightness/ V


A i creases i creases
B i creases u c a e
C ecreases i creases
D ecreases u c a e

ec E P sics P Pa e o Pre im

536
34 ome e ec ro ic compo e s are co ec e as s ow i e circui .

ic o ese circums a ces wi i up e amp i e circui


A ircui p ace u er a ri i
B ircui p ace i a ar room
C a co a
D a o a

35 e i ure e ow s ows e co ec io o a me a case e ec ric e e o e a.c.


mai s.

. .

E
. i ame

ic o e o owi s a eme s a ou e arra eme is TRUE


A e swi c s ou e p ace a o e eu ra wire.
B e ra i o e use is oo sma .
C e use s ou e p ace a o e eu ra wire.
D e Ear wire s ou o e co ec e o e casi .

ec E P sics P Pa e o Pre im

537
36 E X o a me a ro a rac s e po e o a compass ee e. a oes is s ow
a ou e ro
A cou e ma e o copper u is o perma e ma e ise .
B cou e ma e o copper wi a po e a X.
C cou e ma e o s ee u is o ma e ise .
D cou e ma e o s ee wi a po e a X.

37 eam o e ec ro s ra e s e wee e po es o a ma e. e eam s ar s o mo e


i e irec io s ow .
e ec ro
eam
irec io o
mo eme

ic is e mos i e irec io o e ma e ic ie
A i o e paper
B ou o paper
C o e ri
D o e e

38 si a is isp a e o a . . . scree as s ow .

a is e se i o e ime ase o e . . .
A ms i
B ms i
C ms i
D ms i

ec E P sics P Pa e o Pre im

538
39 sma coi is co ec e o a se si i e a a ome er. e e ma e is a owe o
a owar s e coi e a a ome er ee e mo es uic o e ri o e ero
posi io . e ma e mo es rou e coi .

ow oes e a a ome er ee e mo e as e ma e a s awa rom e coi


A oes o mo e.
B i es a s ea rea i o e ri .
C mo es uic o e e o e ero posi io a e re ur s o ero
D mo es uic o e ri o e ero posi io a e re ur s o ero.

40 pupi se s up a mo e ra s ormer as s ow . is co ec e oa . .c. supp .


o amps a e a o a e o . .

ur s ur s
primar coi seco ar coi

.
.
amp P
amp Q

a oes e pupi o ice a ou e amps

lamp P lamp Q
A orma ri ess o i
B er ri im
C im er ri
D ecreases u c a e

END OF PAPER
ec E P sics P Pa e o Pre im

539
ame ass ex um er

UNITY SECONDARY SCHOOL


PRELIMINARY EXAMINATION 2018

SECONDARY FOUR EXPRESS

PHYSICS 6091/02 12 SEPTEMBER 2018

PAPER 2 1 HOUR 45 MINUTES

Additional Materials : NIL

READ THESE INSTRUCTIONS FIRST


. swer ALL ues io s i ec io A o e ues io paper.
. Section B a swer ues io s 9 a 10 a either part o ues io 11. ri e
our a swers i e spaces pro i e o e ues io paper.
. wor i s a co s ruc io s mus e s ow c ear . Omission of essential
working will result in loss of marks
. e um er o mar s is i e i rac e s a ee o eac ues io or par
ues io .
. ou are expec e o use a e ec ro ic ca cu a or o e a ua e exp ici umerica
expressio .
. e o a mar or is paper is mar s.

is paper co sis s o 17 pri e pa es i c u i is co er pa e.

ec E P sics P Pa e o Pre im

540
Section A 50 Marks

swer ALL e ues io s i is sec io . ri e our a swers i e spaces pro i e o e


ues io paper.

1 a i apore i race a race car ri er approac es a s ape e care u


a e e i i o e race as o ows
(i) e race car approac es e irs e a m a s.
(ii) e race car s ows ow wi ecreasi ece era io o m o a i ae e
irs e rom s i s.
(iii) e race car co i ues a m o a i ae e seco e rom s i
s.
(iv) e race car spee s up a o a s rai roa wi i creasi acce era io o
m a er e e rom s i s.

(a) a e w a is mea u i orm acce era io .

. .. ....

. .

(b) si e i orma io rom (i) o (iv) s e c e spee ime rap o e race car
rom s i s. a e our rap .

ec E P sics P Pa e o Pre im

541
2 i . . e ow represe s a orce ime rap o e orce app ie o a s a io ar
ox o pus i o er a rou sur ace. e ox acce era es u i orm a . m s uri
s o s. ssume e ric io a orce is co s a rou ou e mo io o e
ox.
orce

ime s

Fig. 2.1
(a) a cu a e e ric io a orce ac i o e ox.

ric io a orce .................................


(b) e ermi e e acce era io rom s o s.

acce era io .................................

(c) er seco s a ra i orce is ei app ie o e ox o s ow i ow .


e ermi e e a era e ra i orce app ie rom s o s op i a s.

a era e ra i orce .................................

ec E P sics P Pa e o Pre im

542
3 u i orm ro PQ o e . cm a wei . is u as s ow i e i . . .
wo spri a a ces A a B are a ac e o poi s P a Q o e ro respec i e .
oa o . is p ace . cm rom e spri a a ce A.

spri spri
a a ce A a a ce B

P Q
cm . cm cm
mar mar
.
Fig. 3.1
(a) i . . mar a a e e posi io o e wei o e ro .
(b) a i mome s a ou poi Q ca cu a e e rea i o e spri a a ce A i
or er or e ro o a a ce ori o a .

rea i .................................

(c) e ce or o erwise e ermi e e rea i o e spri a a ce B i or er or e


ro o a a ce ori o a .

rea i .................................

(d) e . wei is ra ua mo e a o e ro owar s poi P w ie e


ro is ei ep i e ui i rium.
aea exp ai ec a ei e rea i o e spri a a ce B.

..

.. .......

.. ....

ec E P sics P Pa e o Pre im

543
4 i . . s ows a pe o ma ome er use o measure e pressure i ere ce e wee
e pressure exer e e orce o o a p a orm area o . m a a as wi i
a as esse . e o ume o as is . m. i e a e e si o wa er is
m a e ra i a io a co s a is g .

orce
ru er p a orm o
as i esse area . m

ma ome er

wa er

Fig. 4.1
(a) a e i i . . wi A e oca io wi e i es pressure.
(b) e ermi e e pressure exer e e orce o e p a orm.

pressure

(c) e ermi e e as pressure exer e e as i e esse .

as pressure ..
(d) er e remo a o e orce o e p a orm e as expa s o a ew
o ume o m.
ssume a e empera ure remai s co s a rou ou e expa sio o e
as e ermi e e ew as pressure exer e e as.

ew as pressure ..
ec E P sics P Pa e o Pre im

544
5 eac er emo s ra es e sou pro uce a ou spea er co ec i a
microp o e o a ca o e ra osci oscope . . . . e eac er i i ia o ai s e
. . . race s ow i i . . . Fig. 5.2 i isio
Fig. 5.1 i isio
i isio
i isio

ai ime ase
i isio ms i isio

s i s i

e eac er e a us s e co ro s o o ai e . . . race s ow i i . . .
(a) aea rie exp ai e a us me s e eac er ma e o e co ro s o o ai
race o i . . .
. ..
...
.. ....
.. .. .

(b) e ime ase s ow i i . . is se o ms i isio .


e ermi e e re ue c emi e e ou spea er.

re ue c ...
(c) e ou spea er was a us e o pro uce ou er sou o a ower pi c .
escri e a exp ai w a appe s o e race o e . . . scree .
.. ....
.. ....
.. .
.. .. .
ec E P sics P Pa e o Pre im

545
6 i . . s ows a posi i e c ar e ro a u c ar e me a sp ere o a i su a e
s a a a co ec io o ear .
u c ar e
me a sp ere
co ec io o
c ar e ro ear

Fig. 6.1
(a) escri e ow is appara us is use o i e e me a sp ere a e a i e c ar e
i uc io .

. ..

...

.. ....

.. .

.. .. .
(b) a e a exp ai w a appe s o e ree e ec ro s i e me a sp ere uri
e c ar i process.
...

.. ....

.. .

.. .. .

(c) ee o e c ar i process e me a sp ere as a e a i e c ar e o


. . e c ar i process oo . s.
a cu a e e a era e curre uri is ime.

curre ..

ec E P sics P Pa e o Pre im

546
7 e e o o power s a io e era es e ec rica e er a a . e
e era or i e power s a io is co ec e o e primar coi o a i ea ra s ormer.
e ra s ormer c a es e o a e e ore e e ec rica e er is ra smi e across
e cou r rou ra smissio ca es a .

power s a io primar seco ar ou pu


coi coi

Fig. 7.1

(a) Exp ai ow e curre i e primar coi ca pro uce a ou pu o a ei e


seco ar coi .

...........

...........

....

......

(b) a cu a e e ra io o e um er o ur s i e primar coi o e um er o ur s


i e seco ar coi .

ra io .................................

(c) e o a resis a ce o e ra smissio ca es is a e e ec rica power


ra smi e rou e ra smissio ca es is . x .
e ermi e e power oss i e ra smissio ca e.

power oss .................................

ec E P sics P Pa e o Pre im

547
8 i . . is raw o u sca e. o ec O is p ace i ro o a co er i e s L. e
e s orms a ima e o e o ec O.

Fig.
Fi
ig.
g. 8.1
Fig. 8.1
1 ((full)
full
fu
ull
ll)
l)

O
F F

Fig. 8.1
(a) i . .

(i) raw wo ra s rom e op o e o ec o oca e e op o ima e.

(ii) raw a a e e ima e .

(b) a e ow e ra s s ow a e ima e is ir ua .

...........

.......

......

(c) e o ec is mo e s i awa rom e co er i e s. a e wo c a es a


is wi cause o e ima e.

. ..

..

. ..

ec E P sics P Pa e o Pre im

548
Section B 30 Marks
swer a ues io s i is sec io . swer o o e o e wo a er a i e ues io s i
Question 11. ri e our a swers i e spaces pro i e o e ues io paper.

9 a experime o i ou e coo i e ec o mo i air o wa er was e


ea e a e o coo . e e wa er reac e a empera ure o q i s empera ure
a eac mi u e was measure a recor e or e ex mi .
e irs ru ere was o wi . e seco ru e wa er was p ace i ro o a
a w ic was swi c e o ow spee . e ir a our ru s e a was
su se ue swi c e o me ium a i spee s respec i e . e resu s o e
experime are s ow i e a e e ow.
a er empera ure q
i spee
mi mi mi mi mi
o wi
ow spee
e ium spee
i spee

(a) Exp ai w e empera ures ee o e recor e w e ere is o wi .

.. .

. .
(b) a es o e er o i crease or ecrease e empera ure o o wa er
. q .
us ca cu a e e rop i e empera ure a e ea ra s erre rom e
o wa er o e surrou i air o er e perio o mi u es. ecor our a swers
i e a e e ow.

empera ure rop ea ra s erre


i spee
o er mi q rom wa er o air
o wi

ow spee

e ium spee

i spee

ec E P sics P Pa e o Pre im

549
(c) e e a was swi c e o i spee ca cu a e ow muc more ea is os
o er e mi u es perio ue o e mo i air as compare o o wi or
o wa er.

ea os .................................
(d) ssume e spee o e mo i air a i spee is m .
si e i orma io rom e experime es ima e e o a ea oss e wa er
i a our i e wi spee is i crease o m .

ea os .................................
(e) a e w e er e ea oss ca cu a e i 9(d) is a o eres ima e or a
u eres ima e a rie exp ai e reaso s .

.. .

.. .

.. .

.. .

. .

ec E P sics P Pa e o Pre im

550
10 i . . s ows a ower u i co ec e o a ermis or a a .c. power source. e
ower u i as a co s a resis a ce o . e resis a ce o e ermis or is
s ow i e rap i i . . .

ower
u i

Fig. 10.1
esis a ce

empera ure

Fig. 10.2

(a) se e rap o e ermi e e resis a ce o e ermis or w e e room


empera ure is a .

.. .

(b) a cu a e e curre rou e ower u i w e e room empera ure is a


.

curre rou e ower u i .................................

ec E P sics P Pa e o Pre im

551
(c) Exp ai ow e spee o e ower u i c a es w e e room empera ure
i creases.

.. ..

.. ..

.. ..

.. .

..
(d) e ower u i ee s a mi imum o po e ia i ere ce o o opera e.
e ermi e e mi imum room empera ure a e ower u i wi s ar o opera e.

mi imum empera ure .................................

(e) Exp ai ow is circui is sui a e i opera i a ou oor ower u i or o a


a i opera io s.

.. ..

.. ..

.. ..

.. ..

.. ..

..

ec E P sics P Pa e o Pre im

552
11 EITHER
i . . s ows a coi i a ma e ic ie . e coi is a e o ro a e a ou e axis. e
e s X a Y o e coi is co ec e irec o a .c. power supp . e arrows o e
si es o e coi s ows e irec io o e curre i e coi .

S N

X Y
Fig. 11.1
(a) i . . raw arrows o s ow e irec io s o e orces ac i o o si es
o e coi .
(b) escri e e mo io o e coi u i i comes o res .

...........

......

S N

X Y

Fig. 11.2

ec E P sics P Pa e o Pre im

553
(c) i . . raw a sp i ri commu a or a rus es co ec e o X a Y.
omp e e e ia ram o s ow a e ec ric circui usi s m o s w ic i c u e a
irec curre supp a a swi c .
(d) Exp ai w e coi ro a es co i uous w e e sp i ri commu a or is use .

...........

ec E P sics P Pa e o Pre im

554
11 OR
i . . s ows a ma e wo compasses a wo ai s.

N S

Fig. 11.3

(a) i . . raw a arrow i eac compass o s ow e irec io o e ma e ic


ie o e ma e ic ie i wo posi io s.
(b) e ma e causes e ai s o ecome ma e ise i uc io . o e so
eac ai ecomes ma e ic po es.
i . . mar N or S o o e s o eac ai o s ow e ma e ic po es.

(c) e e ma e is remo e e ai s are s i ma e ise .


escri e a me o o es w e er e ai s are s i ma e ise w e e are
awa rom e ma e.

...........

......

ec E P sics P Pa e o Pre im

555
i . . s ows a so e oi carr i a curre . e curre i e so e oi crea e a
ma e ic ie . K

arrow s owi e
irec io o e
curre i e

˜A ˜B

L
Fig. 11.4

(d) ma e ic ie i e passes rou Aa B.


i . . raw is ma e ic ie i e o i si e a ou si e e so e oi .
raw a arrow o e i e o s ow e irec io o e ma e ic ie .
(e) a e ow e pa er o e ma e ic ie i es i si e e so e oi c a es w e
e s re o e ma e ic ie i creases.

...........

......

(f) i . . s ows a er ica wire KL ex o e e o e so e oi . e wire is


co ec e o a circui a ere is curre ow war s i e wire rom K o L. e
curre i e so e oi is s ow i i . . .
escri e ow e irec io o e orce o wire KL ca e e ermi e .

...........

ec E P sics P Pa e o Pre im

556
......
End of Paper

ec E P sics P Pa e o Pre im

557
ec E Express P sics Pre im Exam ar i c eme

P1 MCQ:

Qn Ans Qn Ans Qn Ans Qn Ans Qn Ans Qn Ans Qn Ans Qn Ans


C A A B B A A C
D A B A C A A A
B B C B A D D B
C C B C C D A C
D B D B C A B A

P2 Section A
1(a) co s a ra e o c a eo e oci or co s a c a eo e oci per u i ime A1
1(b)

Correct
e oci m pts A1 A
A11

A1
A1

ime s

2(a) ma x . .
ric io rom s . .

2(b) e . . o e orce
e so o ac
acce
c era io m s B1

2(c) spee o ox a s x . . m s A1
cce era io . . ms
ra i orc
rce
orcee ric
ric io . x . A1
ra i orc
rce
orcee . . . A1

3(a) . a cm m ar o i ure A1
mar
3(b) si pr ri cip
pri cip e o mome s a ou poi
um o a ic ic oc w ise mome s sum o c oc wise mome s
wise
. x . m . x . m x m A1
. . A1
3(c) . . A1 ecf 3b
3(d) e spri orce a wi ecrease. A1 oc wise mome a ou poi P cause
e mo i owar s P ecreases wi a esser mome arm. e a i c oc wise
mome pro uce e spri orce o a a ce e ro wi a so ecrease. A1

4(a) Poi is oca e a w ere a o e ase o e a i i ure A1


4(b) . Pa A1
4(c) p . x x Pa A1
o a pressure 120 Pa A1 ecf 4b

558
4(d) P P
. P . A1
P Pa A1 s

5(a) eac er uses s i o mo e e race i isio s rom e o om o scree o e


mi e o scree . A1
eac er a us e ai rom i o i o s re c e wa e i e er ica
irec io . A1
eac er a us e ime ase ms i o ms i o s re c e wa e i ori o a
irec io . A1
5(b) perio i isio x ms i ms A1
. A1
5(c) e amp i u e o e sou o scree wi i crease. A1
e re ue c wi ecrease ess re ue as pi c ower A1

6(a) ar e ro rou ear o u c ar e me a sp ere u o ouc i i . A1


e co ec io o ear is co ec e o e opposi e e o e me a sp ere a i is
e remo e . A1 e c ar e ro is ow remo e e me a sp ere
re ecomes
e a i e c ar e . A1

6(b) e ree e ec ro s are i i ia a rac e e posi i e c ar e ro w e i is is rou


ear o e me a sp ere. A1 e e co ec io o e arr is
a
ear is co ec
ec e e
e ec ro s wi mo e rom e ear o e me a sp p ere
erre resu
e res
esu i i excess
exce
excess
cess e ec ro s o
e me a sp ere. A1 ai s e a i e c ar
e me a sp ere remai ar e a er a
co ec io a ro are se ue ia remo e .
6(c) ar e . x ime . s
. x . . x A1
A1

7(a) e a er a i curre e era e e power s a io


io a e primarimar coi s. A1
prim
pr
is wi i uce a i creasi mama e ic c ie i ep rima
rim r coi .
primar
e i creasi a c a i ma m e icc ie i uces
uces e.m.
e.m. . a a curre i e
seco ar coi s i i A1
A1
7(b) ra io p s p s A
A1
7(c) P x . A1
A1
Power oss . x . A1

8(a)
A1
A 1
Correct
rays
A1 ea

8(b) e ra s i er e a ca o co er e o orm a ima i e. A1


8(c) e ima e wi e ar er A1 e ima e wi s i i s posi io e e ur er awa rom e
o ec a e co er i e s. A1

559
Section B
9(a) is is o e ermi e e mi imum amou o ea oss coo i e ec c a ei
empera ure w e ere is o wi . A1
9(b)
emp rop q ea ra s erre

A1 per row
9(c) A1
9(d) ssumi e coo i e ec is ou e or m e amou o ea os i mi
x ue o wi a x or our A1
x ue o o wi . o a ea os
A1
9(e) is is a o eres ima e as ac ua ea os s. A1
ess
ss
wa er is ess. A1 e mo i air mi
o e owi o er e wa er a a e e ma er a ere
er e or
oree i iss o ic
e icie
coo i s s em A1 a a so sa a e empera ure
urre i er re ce e we
ere wee
e wa er a
e iro me ecome ess as more coo i a es p a
ac
ce ere
ace erere e ra e o coo i is
a so rea re uce .

10(a) A1
10(b) q ermis or .
o a e across ermis or x . A1
A1
urre . . A1

10(c) e e surrou i empera a ure


u i c cr
reases e re
creases rresis
sis
sis a ce o e ermis or rops
ur er. A1 ccor i o e po e iaa i i ere ru
u e e ower o u i wi a e a i er po e ia
i ere ce acrosss i resu i i a i e err curre
cu a a i er spee . A1
so accep o era resis a ce ecrea ase
ses
ecreases s ere
ere o era
e a curre i creases.
er

10(d) e ower is a ermi


er mis
ermiss or
o as a po e ia i ere ce o across.
esis a ce o ermis
errmi
mis orr x
.
A1
A1 cco
or i
ccor o rap a occurs a q A1
10(e) a w e empera
empe
em perara ure
ure is
is i er resis a ce o e ermis or is ower. e po e ia
i ere ce across
acr
cros
osss ower
o er u i is i er ere i creasi
ow ower spee . A1

i w e empera
emp
em er
mpe a ure is ower a a e ow q e resis a ce o e ermis or
i creases e o a po e ia i ere ce across i rises a o e A1 . e po e ia
i ere ce across e ower u i a s e ow opera i o a eo A1 ower is o
opera i .

11 EITHER
11(a) A1

560
11(b) e coi ro a es i a c oc wise irec io a ou i s axis A1 or a ou a maximum o q.
s e curre is o re erse a is u c ure e coi co i ues o s a i e er ica
posi io . A1
11(c)

orrec s ape o ommu a or a rus es A1 circui a er a swi swwi c A1A e ec rica


co ec io s arma ure i correc o commu a or A1
11(d) e sp i ri commu a or c a es e irec io o e curre cur
urre
re e er er a a re o u io
A1 sec io o e coi ro a es upwar s ue o a upwar orce orcee cause
cau
ause ma e ic ie
e ma
a e curre . e e coi is a e er ica posi io cur
urre
re is
e curre is i s a a eous cu o
as e sp i ri commu a or is o i co ac wi e car
caar o rus rus es
ru e A1 . e m mome
ome um
carries e coi pas e er ica posi io e sp i ri commu
com
co mm mu a oror owow coco aac
c s e car o
rus es o e o er si e a re erses e curre o . A1
irecc iio A1 s si e emi
emi s e
a ru e ere wi e a ow war orce o e coi causi e coi
co
oi o ro a e a ou i s axis
oi
co i uous . A1

11 OR
(a b) N S N S
N S

A1 ea ai ma e s a comp pas
a s iirec
compass rec
c io s
c) P ace a o ma
11(c) a e i e iro o i i s iro paper c ips ear o e ai . e ere is
a rac io e ai is s i ma
m e i e A1 or p ace a ma e ear o e ai . es o
or a sou po e ess owar
owa
warr s e a aii . a o e po es is ei repe e e ai is
ma e ise .
Do not accept
accep
pt using
usin
usi g nails
in na s themselves
them
the selves to test each other.
11(d)

A1 ea an arrow and line inside and outside


outs

11(e) e um er o ma e ic ie i es i creases a e ie i es ecomes c oser o o e


a o er A1
11(f) si emi s e a ru e e irec io o e ma e ic ie represe e e
i ex i er is perpe icu ar o e curre ow represe e e mi e i er A1 e
orce e era e as represe e e um is perpe icu ar o o e irec io o e
ma e ic ie a e curre . A1

561
562

S-ar putea să vă placă și